Download as pdf or txt
Download as pdf or txt
You are on page 1of 171

ALGEBRA AND CALCULUS

(19A54101)
COMMON TO ALL BRANCHES

I B. Tech I semester

PREPARED BY
DEPARTMENT OF MATHEMATICS

S.V. COLLEGE OF ENGINEERING


KARAKAMBADI ROAD, TIRUPATI

Page | 1
UNIT-I

MATRICES

Page | 2
Matrix: A system of mn numbers real (or) complex arranged in the form of an ordered set of ‘m’
rows, each row consisting of an ordered set of ‘n’ numbers between [ ] (or) ( ) (or) || || is called a
matrix of order mxn.
a11 a12 ......... a1n 
a 
 21 a12 ......... a 2 n 
Eg: .......... .......... .....  = [aij ]mxn where 1≤ i≤m, 1≤j≤n.
 
.......... .......... ... 
a m1 a m 2 ....... a mn 
 mxn

Types of matrices:
1. Square matrix : A square matrix A of order n x n is sometimes called as a n- rowed matrix A (or)
simply a square matrix of order n

1 1  nd
Eg :   is 2 order matrix
 2 2 
2. Rectangular matrix: A matrix which is not a square matrix is called a rectangular matrix,

1 − 1 2 
Eg:   is a 2x3 matrix
 2 3 4
3. Row matrix: A matrix of order 1xm is called a row matrix
Eg: 1 2 31x 3

4. Column matrix: A matrix of order nx1 is called a column matrix


1 
Eg: 1 
 
 2  3 x1

5. Unit matrix: if A= [aij] nxn such that aij = 1 for i = j and aij = 0 for i≠j, then A is called a unit matrx.

1 0 0
1 0
Eg: I2 =   I3= 0 1 0
0 1  0 0 1 

6. Zero matrix : it A = [aij] mxn such that aij = 0  I and j then A is called a zero matrix (or) null matrix
0 0 0
Eg: O2x3= 
0 0 0

7. Diagonal elements in a matrix: A= [aij]nxn, the elements aij of A for which i = j. i.e.
(a11, a22….ann) are called the diagonal elements of A

1 2 3 
Eg: A= 4 5 6 diagonal elements are 1, 5, 9
7 8 9 

Page | 3
Note: the line along which the diagonal elements lie is called the principle diagonal of A
8. Diagonal matrix: A square matrix all of whose elements except those in leading diagonal are zero
is called diagonal matrix.
If d1, d2….. dn are diagonal elements of a diagonal matrix A, then A is written as A =
diag (d1,d2….dn)

3 0 0 
E.g. : A = diag (3, 1,-2)= 0 1 0 
0 0 − 2

9. Scalar matrix: A diagonal matrix whose leading diagonal elements are equal is called a scalar
matrix.

2 0 0
Eg : A= 0 2 0
0 0 2 

10. Equal Matrices : Two matrices A = [aij] and b= [bij] are said to be equal if and only if (i) A and B
are of the same type(order) (ii) aij = bij for every i & j
11. The transpose of a matrix: The matrix obtained from any given matrix A, by interchanging its
rows and columns is called the transpose of A. It is denoted by A 1 (or) AT.
If A = [aij] m x n then the transpose of A is A1 = [bji] n x m, where b ji = a ij Also (A1)1 = A
Note: A1 and B1 be the transposes of A and B respectively, then
(i) (A1)1 = A
(ii) (A+B)1 = A1+B1
(iii) (KA)1 = KA1, K is a scalar
(iv) (AB)1= B1A1
12. The conjugate of a matrix: The matrix obtained from any given matrix A, on replacing its
elements by corresponding conjugate complex numbers is called the conjugate of A and is
denoted by A
− −
Note: if A and B be the conjugates of A and B respectively then,

(i) ( A ) = A

(ii) A + B = A + B

(iii) ( KA) = K A (K is a any complex number)

(iv) AB = AB
2 3i 2 − 5i  2 − 3i 2 + 5i 
Eg; if A=   = i − 4i + 3 2 x 3
then A
− i 0 4i + 3  2 x 3  0

Page | 4
13. The conjugate Transpose of a matrix
The conjugate of the transpose of the matrix A is called the conjugate transpose of A and is
denoted by Aθ
− −
Thus Aθ = ( A1 ) where A1 is the transpose of A. Now A = [aij] m×n  Aθ =[bij] n×m , where bij = a ij
i.e. the (i,j)th element of Aθ conjugate complex of the (j, i)th element of A.
5 3 − i − 2i 
Eg: if A =  
0 1 + i 4 − i  2X3

5 0 
then Aθ = 3 + i 1 − i 
2i 4+i 
 
  3x2

14.
(i) Upper Triangular matrix: A square matrix all of whose elements below the leading diagonal are
zero is called an Upper triangular matrix.
1 3 8 
Eg; 0 4 − 5 is an upper triangular matrix
0 0 2 
(ii) Lower triangular matrix: A square matrix all of whose elements above the leading diagonal are
zero is called a lower triangular matrix. i.e, aij=0 for i< j
4 0 0 
Eg: 5 2 0
7 3 6 
(iii) Triangular matrix: A matrix is said to be triangular matrix it is either an upper triangular matrix
or a lower triangular matrix
15. Symmetric matrix: A square matrix A =[aij] is said to be symmetric if aij = aji for every i and j
Thus A is a symmetric matrix if AT= A
a h g 
Eg: h b f  is a symmetric matrix
 g f c 
16. Skew – Symmetric: A square matrix A = [aij] is said to be skew – symmetric if aij = – aji for every i
and j.
 0 a − b

E.g : − a 0 c  is a skew – symmetric matrix
 b − c 0 
Thus A is a skew – symmetric iff A= -A1 or -A= A1
Note: Every diagonal element of a skew – symmetric matrix is necessarily zero.
Since aij = -aij  aij = 0

Page | 5
17. Multiplication of a matrix by a scalar.
Let ‘A’ be a matrix. The matrix obtain by multiplying every element of A by a scalar K, is called
the product of A by K and is denoted by KA (or) AK
Thus: A + [aij] m×n then KA = [kaij] m×n = k[aij] m×n
18. Sum of matrices:
Let A = [aij] m×n ,B = [bij] m×n be two matrices. The matrix C = [cij] m×n where cij = aij+bij is called
the sum of the matrices A and B.
The sum of A and B is denoted by A+B.
Thus [aij] m×n + [bij] m×n = [aij+bij] m×n and [aij+bij] m×n = [aij] m×n + [bij] m×n
19. The difference of two matrices: If A, B are two matrices of the same type then A+ (-B) is taken as
A–B
20. Matrix multiplication: Let A = [ai k ]m×n , B = [bkj]nxp then the matrix C = [cij]mxp where cij is called the
product of the matrices A and B in that order and we write C = AB.
The matrix A is called the pre-factor & B is called the post – factor
Note: If the number of columns of A is equal to the number of rows in B then the matrices are said
to be conformable for multiplication in that order.
21. Positive integral powers of a square matrix:
Let A be a square matrix. Then A2 is defined A.A
Now, by associative law A3 = A2.A = (AA)A
= A(AA) = AA2
Similarly we have Am-1A = A Am-1 = Am where m is a positive integer
Note 1: Multiplication of matrices is distributive w.r.t. addition of matrices.
i.e, A (B+C)=AB + AC
(B+C)A =BA+CA
Note 2: If A is a matrix of order mxn then AIn = InA = A
n
22. Trace of A square matrix : Let A = [aij] n×n the trace of the square matrix A is defined as a
i =1
ii .

and is denoted by ‘tr A’


n
Thus trA = a
i =1
ii = a11+a22+ …….ann

a h g
Eg : A = h b f  then trA = a+b+c
 g f c 

Page | 6
Properties: If A and B are square matrices of order n and λ is any scalar, then
(i) tr (λ A) = λ tr A
(ii) tr (A+B) = trA + tr B
(iii) tr(AB) = tr(BA)
23. Idempotent matrix: If A is a square matrix such that A2 = A then ‘A’ is called idempotent matrix
24. Nilpotent Matrix: If A is a square matrix such that Am=0 where m is a +ve integer then A is called
nilpotent matrix.
Note: If m is least positive integer such that Am = 0 then A is called nilpotent of index m
25. Involuntary: If A is a square matrix such that A2 = I then A is called involuntary matrix.

26. Orthogonal Matrix: A square matrix A is said to be orthogonal if AAT = AT A = I


Examples:
cos sin  
1. Show that A =   is orthogonal.
− sin  cos 
cos sin  
Sol: Given A =  
− sin  cos 

cos − sin  
AT = 
sin  cos 

cos sin   cos − sin  


Consider A.AT = 
− sin  cos  sin  cos 

 cos2  + sin 2  − cos sin  + cos sin  


= 
− sin  cos + cos sin  cos2  + sin 2  

1 0
A AT =  = I
0 1 
A is orthogonal matrix.

− 1 2 2 
2. Prove that the matrix 2 − 1 2  is orthogonal.
1
3
2 2 − 1
− 1 2 2 
Sol: Given A = 2 − 1 2 
1
3
2 2 − 1

Page | 7
− 1 2 2 
Then AT = 2 − 1 2 
1
3
2 2 − 1

− 1 2 2  − 1 2 2 
Consider A .AT = 2 −1 2  2 − 1 2 
1
9  
2 2 − 1 2 2 − 1
9 0 0 1 0 0
= 0 9 0  = 0 1 0 
1
9
0 0 9 0 0 1 
A.AT = I
Similarly AT.A = I
Hence A is orthogonal Matrix
0 2b c 
3. Determine the values of a, b, c when a b − c  is orthogonal.
a − b c 
Sol: - For orthogonal matrix AAT =I

0 2b c  0 a a 
So AAT = a b − c  2b b − b  = I
a − b c  c − c c 
 4b 2 + c 2 2b 2 − c 2 − 2b 2 + c 2  1 0 0
 2  0 1 0 
2b − c a2 + b2 + c2 a2 − b2 − c2
2
 =I=  
− 2b 2 + c 2 a 2 − b 2 − c 2 a 2 + b 2 + c 2  0 0 1 
 
Solving 2b2-c2 =0, a2-b2-c2 =0
We get c =  2b a2 =b2+2b2 =3b2

 a =  3b
From the diagonal elements of I
4b2+c2= 1  4b2+2b2=1 (c2=2b2)
1
b= 
6

a=  3b
1
=
2
1
b= 
6
c =  2b
1
=
3

Page | 8
27. Determinant of a square matrix:

a11 a12 ......... a1n  a11 a12 ......... a1n


a  a 21 a 22 ......... a 2 n
 21 a 22 ......... a 2 n 
If A = .......... .......... .....  then A = .......... .......... .....
 
a i1 a i 2 ......... a in  a i1 a i 2 ......... a in
a  a n1 a n 2 ....... a nn
 n1 a n 2 ....... a nn  nxn

28. Minors and cofactors of a square matrix


Let A =[aij] n×n be a square matrix when form A the elements of ith row and jth column are
deleted the determinant of (n-1) rowed matrix [Mij] is called the minor of aij of A and is denoted
by |Mij|
The signed minor (-1) i+j |Mij| is called the cofactor of aij and is denoted by Aij..

a11 a12 a13 


 
If A = a21 a22 a23  then
a31 a32 a33 

| A | = a11 |M11| + a12 |M12 | +a13 |M13| (or)


= a11 A11 +a12 A12 +a13 A13

1 1 3

E.g.: Find Determinant of 1 3 − 3 by using minors and co-factors.
− 2 − 4 − 4 
1 1 3

Sol: A = 1 3 − 3
− 2 − 4 − 4 

3 −31 −3 1 3
det A = 1 −1 +3
−4 −4 −2 −4 −2 −4
=1(-12-12)-1(-4-6)+3(-4+6)
= -24+10+6 = -8
Similarly, we find det A by using co-factors also.
Note 1: If A is a square matrix of order n then KA = K n A , where k is a scalar.

Note 2: If A is a square matrix of order n, then A = AT

Note 3: If A and B be two square matrices of the same order, then AB = A B

Page | 9
29. Inverse of a Matrix: Let A be any square matrix, then a matrix B, if exists such that AB = BA =I
then B is called inverse of A and is denoted by A-1.
Note:1 (A-1)-1 = A
Note 2: I-1 = I
30. Adjoint of a matrix:
Let A be a square matrix of order n. The transpose of the matrix got from A by replacing the
elements of A by the corresponding co-factors is called the adjoint of A and is denoted by adj A.
Note: For any scalar k, adj(kA) = kn-1 adj A
Note: The necessary and sufficient condition for a square matrix to posses’ inverse is that A  0

−1 1
Note: if A  0 then A = (adj A)
A

31. Singular and Non-singular Matrices:


A square matrix A is said to be singular if A = 0 .

If A  0 then ‘A’ is said to be non-singular.

Note: 1. only non-singular matrices possess inverses.


2. The product of non-singular matrices is also non-singular.
Theorem: If A, B are invertible matrices of the same order, then
(i). (AB)-1 = B-1A-1
(ii). (A1)-1 = (A-1)1
Proof:
(i). we have (B-1A-1) (AB) = B-1(A-1A)B
= B-1(I B)
= B-1B
=I
(AB)-1 = B-1A-1
(ii). A-1A = AA-1 = I
Consider A-1A =I
 (A-1 A) 1 = I1
 A1. (A-1)1 = I
 (A1)-1 = (A-1)1
Unitary matrix:
( ) −1
T
A square matrix A such that A = A

( ) ( )
T T
i.e A A=A A =I

If A A=I then A is called Unitary matrix

Page | 10
Note: The transpose of a unitary matrix is unitary.

PROBLEMS
 3 7 − 4i −2 + 5i 
 7 + 4i −2 3 + i  then show that A is Hermitian and iA is skew-Hermitian.
1) If A= 
 −2 − 5i 3 − i 4 

 3 7 − 4i −2 + 5i 
 7 + 4i −2 3 + i  then
Sol: Given A= 
 −2 − 5i 3 − i 4 

 3 7 + 4i −2 − 5i   3 7 − 4i −2 + 5i 

A =  7 − 4i −2  
3 − i  And ( A ) =  7 + 4i
T
−2 3 + i 
 −2 + 5i 3 + i 4   −2 − 5i 3 − i 4 

( )
T
A= A Hence A is Hermitian matrix.

Let B= iA

 3i 4 + 7i −5 − 2i 
 
i.e B=  −4 + 7i −2i −1 + 3i  then
 5 − 2i 1 + 3i 4i 

 −3i 4 − 7i −5 + 2i 

B =  −4 − 7i 2i −1 − 3i 
 5 + 2i 1 − 3i −4i 

 −3i −4 − 7i 5 + 2i   3i 4 + 7i − 5 − 2i 
( B)  1 − 3i  = (−1) − 4 + 7i − 2i − 1 + 3i  = − B
 
T
=  4 − 7i 2i
 −5 + 2i −1 − 3i −4i   5 − 2i 1 + 3i 4i 

 ( B ) =-B
T

B= iA is a skew Hermitian matrix.


2) If A and B are Hermitian matrices, prove that AB-BA is a skew-Hermitian matrix.
Sol: Given A and B are Hermitian matrices

 ( A ) = A And ( B ) = B ------------- (1)


T T

Now ( AB − BA ) = AB − BA ( )
T T

( )
T
= AB − B A

( ) − ( B A) = ( B ) ( A) − ( A) ( B )
T T T T T T
= AB

Page | 11
= BA − AB (By (1))
= − ( AB − BA)

Hence AB-BA is a skew-Hermitian matrix.

 a + ic −b + id 
3) Show that A=  is unitary if and only if a2+b2+c2+d2=1
b + id a − ic 

 a + ic −b + id 
Sol: Given A= 
b + id a − ic 

 a − ic −b − id 
Then A = 
b − id a + ic 

 a − ic b − id 
 T
Hence A = A =  () a + ic 
− b − id
 a + ic − b + id   a − ic b − id 
 AA =   
b + id a − ic  − b − id a + ic 
 a 2 + b2 + c2 + d 2 0
= 
 0 a +b +c +d 
2 2 2 2

 AA = I If and only if a 2 + b 2 + c 2 + d 2 = 1

4). Show that every square matrix is uniquely expressible as the sum of a Hermitian matrix and a

skew- Hermitian matrix.

Sol. Let A be any square matrix

( ) = A + ( A )
  
Now A + A

= A + A

(A+ A )  
= A + A  A + A is a Hermitian matrix.


1
2
( )
A + A is also a Hermitian matrix

( ) = A − ( A )
  
Now A − A

= A − A = − ( A − A)

Hence A − A is a skew-Hermitian matrix


1
2
( A − A ) is also a skew –Hermitian matrix.

Page | 12
Uniqueness:

Let A =R+S be another such representation of A

Where R is Hermitian and S is skew-Hermitian


Then A = ( R + S )

= R + S 
= R−S ( R = R, S  = −S )

R =
1
2
( ) 1
A + A = P and S = A − A = Q
2
( )
Hence P=R and Q=S
Thus the representation is unique.

 0 1 + 2i 
, show that (I − A)(I + A) is a unitary matrix.
−1
5).Given that A=  
 −1 + 2i 0 
1 0  0 1 + 2i 
Sol: We have I − A =   −
0 1 − 1 + 2i 0 
 1 −1 − 2i 
=
1 
and
1 − 2i
1 0  0 1 + 2i 
I + A=  +
0 1 − 1 + 2i 0 
 1 1 + 2i 
=
 −1 + 2i 1 
1  1 − 1 − 2i 
 ( I + A) −1 =
(  )
1 − 4i − 1 1 − 2i
2
1 
1 1 −1 − 2i 
=
6 1 − 2i 1 

Let B = (I − A)(I + A)
−1

1 1 − 1 − 2i   1 − 1 − 2i  1 1 + (1 − 2i )(−1 − 2i ) − 1 − 2i − 1 − 2i 
B=     = 
6 1 − 2i 1  1 − 2i 1  6  1 − 2i + 1 − 2i (−1 − 2i )(1 − 2i ) + 1
1  − 4 − 2 − 4i 
B= 
6 2 − 4i − 4 

1  −4 −2 + 4i  1  −4 2 + 4i 
and ( B ) = 
T
Now B =  
6  2 + 4i −4  6  −2 + 4i −4 

1  −4 −2 − 4i   −4 2 + 4i 
( )
T
= − 2 + 4i − 4 
36  2 − 4i −4 
B B
 

1 36 0  1 0 
= = =I
36  0 36  0 1 

Page | 13
(B)
T
= B −1

i.e., B is unitary matrix.

 (I − A)(I + A) is a unitary matrix.


−1

6) Show that the inverse of a unitary matrix is unitary.

Sol: Let A be a unitary matrix. Then AA = I

i.e ( AA ) −1
= I −1

 ( A ) A−1 = I
−1

 ( A −1 ) A −1 = I

Thus A−1 is unitary.

Problems
1). Express the matrix A as sum of symmetric and skew – symmetric matrices. Where

3 −2 6
A = 2 7 − 1
5 4 0 
3 −2 6
Sol: Given A = 2 7 − 1
5 4 0 

 3 2 5 
Then AT = − 2 7 4 

 6 − 1 0 

Matrix A can be written as A = ½ (A+AT)+ ½ (A-AT)

3 −2 6  3 + 2 5 
1  
P= ½ (A+AT) =  2 7 − 1 + − 2 7 4  
2 
5 4 0  6 −1 0 

6 0 11  3 0 11 / 2
1
= 0 14 3  = 0
 7 3 / 2 
2
11 3 0 11 / 2 3/ 2 0 

Page | 14
Q= ½ (A-AT)

3 −2 6  3 2 5  0 −4 1
1   
  4   = 4 − 5
1
=  2 7 − 1 − − 2 7 0
2  2
5 4 0  6 − 1 0 
 − 1 5 0

0 −2 1 / 2
= 2 0 − 5 / 2 
− 1 / 2 5 / 2 0 
A = P+Q where ‘P’ is symmetric matrix
‘Q’ is skew-symmetric matrix.
Sub – Matrix: Any matrix obtained by deleting some rows or columns or both of a given matrix is
called is sub Matrix.
1 5 6 7 
8 9 10
E.g.: Let A = 8 9 10 5  . Then   is a sub matrix of A obtained by deleting first row
3 4 5 −1 3 4 5  2 x3

and 4th column of A.


Minor of a Matrix: Let A be an m x n matrix. The determinant of a square sub matrix of A is called a
minor of the matrix.
Note: If the order of the square sub matrix is‘t’ then its determinant is called a minor of order is‘t’.

2 1 1
3 1 2 
Eg: A = 
1 2 3
 
5 6 7  4X3

2 1
→B =  is a sub-matrix of order ‘2’
3 1
B = 2-3 = -1 is a minor of order ‘2’
2 1 1 
→ C = 3 1 2 is a sub-matrix of order ‘3’
5 6 7 
detC = 2(7-12)-1(21-10) +(18-5)
= 2(-5)-1(11) +1(13)
= -10-11+13 = -8 is a minor of order ‘3’

Page | 15
*Rank of a Matrix:
Let A be m x n matrix. If A is a null matrix, we define its rank to be ‘0’. If A is a non-null matrix,
we say that r is the rank of A if
(i) Every (r+1)th order minor of A is ‘0’ (zero) &
(ii) At least one rth order minor of A which is not zero.
Note:
1. It is denoted by ρ (A)
2. Rank of a matrix is unique.
3. Every matrix will have a rank.
4. If A is a matrix of order mxn,
Rank of A ≤ min (m,n)
5. If ρ(A) = r then every minor of A of order r+1, or more is zero.
6. Rank of the Identity matrix In is n.
7. If A is a matrix of order n and A is non-singular then ρ(A) = n
Important Note:
1. The rank of a matrix is ≤r if all minors of (r+1)th order are zero.
2. The rank of a matrix is ≥r, if there is at least one minor of order ‘r’ which is not equal to zero.
PROBLEMS

1 2 3 
1. Find the rank of the given matrix 3 4 4 
7 10 12
1 2 3 
Sol: Given matrix A = 3 4 4 
7 10 12

→ det A = 1(48-40)-2(36-28) +3(30-28)


= 8-16+6 = -2 ≠ 0
We have minor of order 3
ρ(A) =3
1 2 3 4
2. Find the rank of the matrix 5 6 7 8 
8 7 0 5 
Sol: Given the matrix is of order 3x4
Its Rank ≤ min (3,4) = 3
Highest order of the minor will be 3.

Page | 16
1 2 3 
Let us consider the minor 5 6 7 

8 7 0 

Determinant of minor is = 1(-49)-2(-56) +3(35-48)


= -49+112-39 = 24 ≠ 0.
Hence rank of the given matrix is ‘3’.
* Elementary Transformations on a Matrix:
i). Interchange of ith row and jth row is denoted by Ri ↔ Rj
(ii). If ith row is multiplied with k then it is denoted by Ri →K Ri
(iii). If all the elements of ith row are multiplied with k and added to the corresponding elements of j th
row then it is denoted by Rj → Rj +KRi
Note: 1. The corresponding column transformations will be denoted by writing ‘c’. i.e
ci ↔cj, ci → k cj cj → cj + kci
2. The elementary operations on a matrix do not change its rank.
Equivalence of Matrices: If B is obtained from A after a finite number of elementary transformations
on A, then B is said to be equivalent to A and it is denoted as B~A.
Note : 1. If A and B are two equivalent matrices, then rank A = rank B.
2. If A and B have the same size and the same rank, then the two matrices are equivalent.
Echelon form of a matrix: ROW OPERATIONS
A matrix is said to be in Echelon form, if
(i). Zero rows, if any exists, they should be below the non-zero row.
(ii). the first non-zero entry in each non-zero row is equal to ‘1’.
(iii). the number of zeros before the first non-zero element in a row is less than the number of such
zeros in the next row.
Note:
1. The number of non-zero rows in echelon form of A is the rank of ‘A’.
2. The rank of the transpose of a matrix is the same as that of original matrix.
3. The condition (ii) is optional.
E.g.:
1 0 0 0
0 1 0 0
1.  is a row echelon form.
0 0 1 1
 
0 0 0 0 

Page | 17
1 0 0 0 0 0
2. 0 1 0 0 0 0  is a row echelon form.
0 0 0 0 0 0

PROBLEMS
2 3 7 
1. Find the rank of the matrix A = 3 − 2 4  by reducing it to Echelon form.
1 − 3 − 1 
2 3 7 
Sol: Given A = 3 − 2 4 
1 − 3 − 1 
Applying row transformations on A
1 − 3 − 1
A ~ 3 − 2 4 R1 ↔ R3
2 3 7 
1 − 3 − 1 
~ 0 7 7  R2 → R2 –3R1, R3→ R3 -2R1
0 9 9 
1 − 3 − 1
~ 0 1 1  R2 → R2/7, R3→ R3/9
0 1 1 
1 − 3 − 1
~ 0 1 1  R3 → R3 –R2
0 0 0 
This is the Echelon form of matrix A.
The rank of a matrix A = Number of non-zero rows =2
4 4 − 3 1 
1 1 −1 0 
2. For what values of k the matrix   has rank ‘3’.
k 2 2 − 2
 
9 9 k 3 
Sol: The given matrix is of the order 4x4
If its rank is 3  det A =0
4 4 − 3 1 
1 1 −1 0 
A=  
k 2 2 − 2
 
9 9 k 3 
By applying R2 → 4R2-R1, R3 →4R3 – kR1, R4 → 4R4 – 9R1

Page | 18
4 4 −3 1 
0 0 −1 − 1 
We get A ~ 
0 8 − 4k 8 + 3k 8 − k 
 
0 0 4k + 27 3
Since Rank A = 3  det A =0
0 −1 −1
 4 8 − 4k 8 + 3k 8−k = 0
0 4k + 27 3
 1[(8-4k)3]-1(8-4k) (4k+27)] = 0
 (8-4k) (3-4k-27) = 0
 (8-4k) (-24-4k) =0
 (2-k) (6+k) =0
 k =2 or k = -6
2 1 3 5
4 2 1 3
3. Find the rank of the matrix by using Echelon form 𝐴 = [ ]
8 4 7 13
8 4 −3 −1
2 1 3 5
4 2 1 3
Sol: Given 𝐴 = [ ]
8 4 7 13
8 4 −3 −1
By applying𝑅2 → 𝑅2 − 2𝑅1 ; 𝑅3 → 𝑅3 − 4𝑅1 ; 𝑅4 → 𝑅4 − 4𝑅1
2 1 3 5
0 0 −5 −7
𝐴~ [ ]
0 0 −5 −7
0 0 −15 −21
𝑅 𝑅 𝑅
By applying𝑅2 → −12 ; 𝑅3 → −13 ; 𝑅4 → −34
2 1 3 5
0 0 5 7
𝐴~ [ ]
0 0 5 7
0 0 5 7
By applying 𝑅3 → 𝑅3 − 𝑅2 ; 𝑅4 → 𝑅4 − 𝑅2
2 1 3 5
0 0 5 7
𝐴~ [ ]
0 0 0 0
0 0 0 0
∴A is in Echelon form
Rank of A= Number of non zero rows
∴ 𝜌(𝐴) = 2

Page | 19
𝟏 −𝟐 𝟎 𝟏
𝟐 −𝟏 𝟏 𝟎
4. Find the rank of the matrix 𝑨 = [ ] by reducing it to Echelon form.
𝟑 −𝟑 𝟏 𝟏
−𝟏 −𝟏 −𝟏 𝟏
1 −2 0 1
2 −1 1 0
Sol: Given 𝐴 = [ ]
3 −3 1 1
−1 −1 −1 1
By applying𝑅2 → 𝑅2 − 2𝑅1 ; 𝑅3 → 𝑅3 − 3𝑅1 ; 𝑅4 → 𝑅4 + 𝑅1
1 −2 0 1
0 3 1 −2
𝐴~ [ ]
0 3 1 −2
0 −3 −1 2

By applying 𝑅3 → 𝑅3 − 𝑅2 ; 𝑅4 → 𝑅4 + 𝑅2
1 −2 0 1
0 3 1 −2
𝐴~ [ ]
0 0 0 0
0 0 0 0

∴ A is in Echelon form
Rank of A= Number of non zero rows
∴ 𝜌(𝐴) = 2
−𝟏 𝟐 𝟏 𝟖
5. Find the rank of the matrix 𝑨 = [ 𝟐 𝟏 −𝟏 𝟎] by reducing it to Echelon form.
𝟑 𝟐 𝟏 𝟕
−1 2 1 8
Sol: Given 𝐴 = [ 2 1 −1 0]
3 2 1 7
By applying 𝑅1 → −𝑅1
1 −2 −1 −8
𝐴~ [2 1 −1 0]
3 2 1 7
By applying 𝑅2 → 𝑅2 − 2𝑅1 ; 𝑅3 → 𝑅3 − 3𝑅1
1 −2 −1 −8
𝐴~ [0 5 1 16]
0 8 4 31
By applying 𝑅2 → 8𝑅2
1 −2 −1 −8
𝐴~ [0 40 8 128]
0 8 4 31
By applying 𝑅2 ↔ 𝑅3
1 −2 −1 −8
𝐴~ [0 8 4 31]
0 40 8 128
By applying 𝑅3 → 𝑅3 − 5𝑅2
1 −2 −1 −8
𝐴~ [0 8 4 31]
0 0 −12 −27
∴ A is in Echelon form
Rank of A= Number of non-zero rows
∴ 𝜌(𝐴) = 3

Page | 20
𝟏 𝟒 𝟑 −𝟐 𝟏
−𝟐 −𝟑 −𝟏 𝟒 𝟑
6. Find the rank of the matrix 𝑨 = [ ] by reducing it to Echelon form.
−𝟏 𝟔 𝟕 𝟐 𝟗
−𝟑 𝟑 𝟔 𝟔 𝟏𝟐
1 4 3 −2 1
−2 −3 −1 4 3
Sol: Given 𝐴 = [ ]
−1 6 7 2 9
−3 3 6 6 12
By applying 𝑅2 → 𝑅2 + 2𝑅1 ; 𝑅3 → 𝑅3 + 𝑅1 ; 𝑅4 → 𝑅4 + 3𝑅1
1 4 3 −2 1
0 5 5 0 5
𝐴~ [ ]
0 10 10 0 10
0 15 15 0 15
By applying 𝑅3 → 𝑅3 − 2𝑅3 ; 𝑅4 → 𝑅4 − 3𝑅2
1 4 3 −2 1
0 5 5 0 5
𝐴~ [ ]
0 0 0 0 0
0 0 0 0 0
∴ A is in Echelon form
Rank of A= Number of non-zero rows
∴ 𝜌(𝐴) = 2

Normal Form:
 I r 0
Every mxn matrix of rank r can be reduced to the form   (or) (Ir ) (or)
0 0 
Ir   I 0
  (or)  r  by a finite number of elementary transformations, where Ir is the r – rowed
0   
unit matrix.

Note: 1. If A is an mxn matrix of rank r, there exists non-singular matrices P and Q such that PAQ
=
 I r 0
 
0 0 
Normal form another name is “canonical form”
1 2 3 4 
E.g.: By reducing the matrix 2 1 4 3  into normal form, find its rank.
3 0 5 −10
1 2 3 4 
Sol: Given A = 2 1 4 3 
3 0 5 −10
By applying R2 → R2 – 2R1 ; R3 → R3 – 3R1
1 2 3 4 

A ~ 0 − 3 − 2 5 
0 − 6 − 4 − 22

Page | 21
By applying R3 → R3/-2
1 2 3 4 

A ~ 0 − 3 − 2 − 5
0 3 2 11 
By applying R3 → R3+R2
1 2 3 4 

A ~ 0 − 3 − 2 − 5
0 0 0 6 
By applying c2→ c2 - 2c1, c3→c3-3c1, c4→c4-4c1
0 0 0 0 

A ~ 0 − 3 − 2 − 5
0 0 0 6 
By applying c3 → 3 c3 -2c2, c4→3c4-5c2
1 0 0 0
A ~ 0 − 30 0 
0 0 0 18
By applying c2→c2/-3, c4→c4/18
1 0 0 0
A ~ 0 1 0 0 
0 0 0 1 
By applying c4 ↔ c3
1 0 0 0
A~ 0 1 0 0 
00 1 0 
This is in normal form [I3 0]
Hence Rank of A is ‘3’.
Gauss – Jordan method
• The inverse of a matrix by elementary Transformations: (Gauss – Jordan method)
1. suppose A is a non-singular matrix of order ‘n’ then we write A = In A
2. Now we apply elementary row-operations only to the matrix A and the pre-factor In of
the R.H.S
3. We will do this till we get In = BA then obviously B is the inverse of A.

1 6 4
1. Find the inverse of the matrix A using elementary operations where A= 0 2 3 
0 1 2 
1 6 4
Sol: Given A = 0 2 3 
0 1 2 

Page | 22
We can write A = I3 A
1 6 4 1 0 0
0 2 3  = 0 1 0  A
   
0 1 2  0 0 1 
Applying R3 →2R3-R2, we get
1 6 4 1 0 0 
0 2 3  = 0 1 0  A

0 0 1  0 − 1 2
Applying R1→R1-3R2, we get
1 0 − 5 1 − 3 0 
 0 2 3  = 0 1 0  A
   
0 0 1  0 − 1 2
Applying R1 → R1+5R3, R2 → R2-3R3 , we get
1 00 1 − 8 10 
0 20  = 0 4 − 6  A

1  0 − 1
0 0 2
Applying R2 →R2/2, we get
1 0 0 1 − 8 10 
0 1 0  = 0 2 − 3  A
  
0 0 1  0 − 1 2
I3 = BA
B is the inverse of A.

SYSTEM OF HOMOGENEOUS AND NON HOMOGENEOUS LINEAR EQUATIONS

An equation of the form a1 x1 + a 2 x2 + .......... .... + a n xn = b where x1, x2 ,………,xn are


unknowns and a1 , a2 ,……………….,an , b are constants is called linear equation in n unknowns .
Definition: Consider the system of m linear equations in n unknowns x1, x2 ,………,xn as given below:
a11 x1 + a1 2 x2 + .......... .... + a1 n xn = b1
a21 x1 + a2 2 x2 + .......... .... + a2 n xn = b2
………………………………………………………………………………………………

ai 1 x1 + ai 2 x2 + .......... .... + ai n xn = bi
……………………………………………………………………………………………….

am1 x1 + am 2 x2 + .......... .... + am n xn = bm


The number aij’s are known as coefficient and b1 , b2 , …………….bm are constants. An ordered n-tuple
(x1, x2 ,………,xn ) satisfying all the equations simultaneously is called a solution of system.

Homogeneous system:
If all bi = 0 for i = 1, 2, …………………,m.

Page | 23
Non-Homogeneous system:
If all bi  0 i.e.at least one bi  0.
Matrix Representation:
The above system of linear non Homogeneous equations can be written in Matrix form as AX=B
 a11 a12 .......... . a1n   x1   b1 
a a 22..................... a 2 n   x 2  =  b2 
 21
a m1 a m 2...................... a mn   x n  bm 

Augmented Matrix:
It is denoted by [A/B] or [A B] is obtained by Augmenting A by the column B.

 a11 a12 .......... . a1n b1 


 
∴[A /B] =  a 21 a 22..................... a 2 n b2 
a m1 a m 2...................... a mn bm 

By reducing [A /B] into its row echelon form the existence and uniqueness of solution AX = B exists.
NOTE:
Given a system, we do not know in general whether it has a solution or not .If there is atleast one
solution , then the system is said to be consistent .If does not have any solution then the system is
inconsistent.
1. CONSISTENT: A system is said to be consistent if it has at least one solution
a) If  ( AB ) =  ( A) = r = n (total number of unknowns) then the system is consistent and it has
unique solution.
b) If  ( AB ) =  ( A) = r  n (total number of unknowns) then the system is consistent and it has
Infinitely many solutions.
NOTE: To obtain solutions set (n – r) variables any arbitrary value & solve for remaining unknowns.
2. INCONSISTENT: If  ( AB )   ( A) then the system is inconsistent and it has no solution.
NOTE: Here rank is denoted by 
For Non Homogeneous System, The system AX = B is consistent i.e it has a solution.
The system is inconsistent i.e. it has no solution.
NOTE: Find the rank A and rank [A /B] by reducing the augmented matrix [A /B] to Echelon form by
elementary row operations . Then the matrix A will be reduced to Echelon form.
This procedure is illustrated through the following examples.
Example 1: Find whether the following equations are consistent, if so solve them
x + y + 2z= 4; 2x – y + 3z = 9; 3x – y - z = 2.

1 1 2  x  4
2 − 1 3   y  9 
Solution: The given equations can be written in the matrix form as     = 
3 − 1 − 1  z   2

i.e .AX = B

Page | 24
1 1 2 4
2 − 1 3 9 
The Augmented matrix [A/ B] =  
3 − 1 − 1 2

Applying R2 → R2 – 2R1 and R3 → R3 – 3R1

1 1 2 4 
 1 
[A / B] = 0 − 3 − 1
0 − 4 − 7 − 10

Applying R2 → R2 – 2R1 and R3 → R3 – 3R1

1 1 2 4 
0 − 3 − 1 1 
[A / B] = 
0 0 − 17 − 34

Since Rank of A = 3 & Rank of [A / B] = 3


Since the number of non- zero rows of matrix A is 3
Since the number of non- zero rows of matrix [A /B] is 3
∴ Rank of A = Rank of [A B]
i.e.  ( A) =  ( AB )
The given system is consistent. So, it has a solution.
Since Rank of A = Rank of [A /B] = Number of unknowns (n)
∴ The given system has a unique solution.

1 1 2   x  4 
0 − 3 − 1   y  1 
    = 
0 0 − 17  z  − 34

 x + y + 2z= 4 → (1)
-3y – z = 1 → (2)
-17z = -34 → (3)
From (3) z = 2
Substituting z = 2 in eq ( 2) , we get y = -1
Substituting z = 2 & y = -1 in eq (1) , we get x = 1.
∴ x = 1, y = -1 , z = 2 is the solution.
Example 2: Find whether the following system of equations is consistent. If so solve them.
x + 2y+ 2z = 2; 3x - 2y - z = 5: 2x - 5y + 3z = -4; x + 4y + 6z = 0.
Solution: The given equations can be written in the matrix form as AX = B

Page | 25
1 2 2  2 
3 − 2 − 1  x   5 
i.e.    y =  
2 − 5 3     − 4 
   z   
 1 4 6   0 
1 2 2 2
3 − 2 − 1 5 
The Augmented matrix [A/ B] =  
 2 − 5 3 − 4
 
1 4 6 0

Applying R2 → R2 – 3R1; R3 → R3 – 2R1; R4 → R4 – R1


1 2 2 2
0 − 8 − 7 − 1 
[A /B] =  
0 − 9 − 1 − 8 
 
0 2 4 − 2

Applying R3 → 8R3 – 9R2;; R4 → 4R4 +R2 ,we get


1 2 2 2 
0 − 8 − 7 − 1 
[A/B] =  
0 0 55 − 55
 
0 0 9 −9 

Applying R3 → R3/55; R4 → R4/9


1 2 2 2
0 − 8 − 7 − 1
[A /B]   
0 0 1 − 1
 
0 0 1 − 1

Applying R4 → R4 -R3
1 2 2 2
0 − 8 − 7 − 1
[A /B]   
0 0 1 − 1
 
0 0 0 0

Since Rank of A = 3 & Rank of [A/ B] = 3


∴ Rank of A = Rank of [A /B]
i.e.  ( A) =  ( AB )
The given system is consistent, so it has a solution.
Since Rank of A = Rank of [A /B] = Number of unknowns (n)
∴ The given system has a unique solution.

Page | 26
1 2 2 2
0 − 8 − 7   x   
We have    y  =  − 1
0 0 1    − 1
   z   
0 0 0 0

 x + 2y + 2z = 2 → (1)
-8y - 7z = -1 → (2)
z = -1 → (3)
From (3) z = -1
Substituting z = -1 in eq ( 2) , we get y = 1
Substituting z = -1 & y = 1 in eq (1) , we get x = 2.
∴ x = 2 , y = 1 , z = -1 is the solution.
Example3: Show that the equations x + y + z = 4 , 2x + 5y -2z = 3 , x + 7y – 7z = 5 are not consistent.
Solution: The given equations can be written in the matrix form as AX = B

1 1 1   x 4
   y 3
Where A = 2 5 − 2 ,X =   B=  
1 7 − 7   z  5 

1 1 1   x   4 
2 5 − 2  y   3 
   = 
1 7 − 7   z  5 

1 1 1 4 
 
The Augmented matrix [A /B] = 2 5 − 2 3
1 7 − 7 5

Applying R2 → R2 – 2R1 and R3 → R3 – R1

1 1 1 4
 0 3 − 4 − 5
[A /B] =  
0 6 − 8 1 

Applying R3 → R3 – 2R2 , we get

1 1 1 4
 0 3 − 4 − 5
[A /B] =  
0 0 0 11 

We can see  ( AB ) = 3, since the number of non- zero rows is 3

Applying the same row operations on A, we get from above

Page | 27
1 1 1 
 
A = 0 3 − 4 
0 0 0 

Here the number of non- zero rows is 2 so the rank of A = 2


Here we have  ( AB )   ( A) .

 The given system is not consistent.


Example 4: Discuss for what values of  ,  the simultaneous equations x + y + z = 6 ;

x + 2y + 3z =10; x + 2y +  z =  have
i) no solution
ii) a unique solution
iii) an infinite number of solutions.
Solution: The given equations can be written in the matrix form as AX = B

1 1 1   x  6 
   y  10
AX = 1 2 3    =   =B
1 2    z    

1 1 1 6 
 
We have the Augmented matrix [A /B] = 1 2 3 10
1 2   

Applying R2 → R2 – R1 and R3 → R3 – R1 we get

1 1 1 6 
0 1 4 
[A/B] =  2
0 1  − 1  − 6

Applying R3 → R3 – R2 we get

1 1 1 6 
0 1 4 
[A/B] =  2
0 0  − 3  − 10

Case i): Let   3 then rank of A =3 and rank of [A/B]=3, So that they have same rank.
Then the system of equations is consistent. Here the number of unknowns is 3 which is
same as the rank of A. The system of equations will have a unique solution. This is true
for any value of .
Thus if   3 and  has any value, the given system of equations will have a
Unique solution.

Page | 28
Case ii): Suppose  = 3 &   10 , then we can see that rank of A = 2 and rank of [A/B] =3.
Since the ranks of A and [A/B] are not equal, we say that the system of equations has no
solution (inconsistent).
Case iii): Suppose  = 3 &  = 10 .Then we have rank of A = rank of [A/B] = 2

 The given system of equations will be consistent.


But here the number of unknowns = 3 > rank of A
Hence the system has infinitely many solutions.
Some more problems:
1. Test for the consistency of x + y + z = 1; x - y + 2z = 1; x – y + 2z = 5; 2x - 2y + 3z = 1
Solution: Not consistent
2. Find whether the following system of equations is consistent. If so solve them.
X + y + 2z = 9; x - 2y + 2z = 3; 2x - y + z = 3 ; 3x – y + z = 4.
Solution: Not consistent
3. Determine whether the following equations will have a solution, if so solve them
x1 + 2x2+ x3 = 2 ; 3x1 + x2 - 2x3 = 1 ; 4x1 - 3x2 - x3 = 3; 2x1 + 4x2+ 2x3 = 4.
Solution: x1 = 1 ; x2 = 0 ; x3=1.
4. Find the values of a & b for which the equations
x + y + z = 3; x + 2y + 2z = 6 ; x + 9y + a z = b have
i) No Solution ii) A unique solution iii) Infinite number of solutions.
Solution : i) Let a = 17 ; b  27 ii) Let a  17 ;b  27 ; iii) Let a = 17 ; b = 27.

Homogeneous System of Linear Equations:


Definition:
Consider the system of m homogeneous linear equations in n unknowns x1, x2 ,………,xn as given below:

a11 x1 + a12 x2 + .......... .... + a1n xn = 0


a 21 x1 + a2 2 x2 + .......... .... + a2 n xn = 0
………………………………………………………………………………………………

ai 1 x1 + ai 2 x2 + .......... .... + ai n xn = 0
……………………………………………………………………………………………….

a m 1 x1 + a m 2 x 2 + .......... .... + a m n x n = 0 → (1 )

Page | 29
Matrix Representation:
The above system of linear Homogeneous equations can be written in Matrix form as AX = 0

 a11 a12 .......... . a1n   x1  0


a a 22..................... a 2 n   x 2  = 0
 21
a m1 a m 2...................... a mn   x n  0

Here A is called coefficient Matrix. It is clear that x1 = 0 = x2 = x3 = …………….. = xn is a solution


of equation (1)
This is called Trivial solution of AX = 0.
Definitions:
1. The system AX = 0 is always consistent since X = 0 i.e.(x1 = 0 = x2 = x3 = …………….. = xn ) is a
Solution of AX = 0. This solution is called a trivial solution of the system. The Trivial solution is
called zero solution.
2. Given AX = 0, we try to decide whether it has a solution X  0. Such a solution, if exists, is called a
non -Trivial solution.
Note: 1. If A is non-Singular matrix i.e.(det A  0) then the linear system AX = 0 has only the zero
solution.
2. The system AX = 0 possesses a non – zero solution  A is singular matrix.
Working Rule For Finding The Solutions Of The Equation AX = 0 :
I .i) If r = n (number of variables)  the system of equations have only Trivial solution (Zero
solution)
ii) If r< n  the system of equations have an infinite number of Non – Trivial solutions, we shall
have n – r linearly independent solutions.
To obtain infinite solutions, set (n – r) variables any arbitrary value and solve for the
remaining Unknowns.
II. If the number of equations is less than number of variables, the solution is always other
than a Trivial Solution.
III. If the number of equations is equal to number of variables, the necessary and sufficient
condition for the solutions other than a Trivial solution is that the determinant of the
coefficient matrix is zero.
Example 1: Solve completely the system of equations:
x +2y + 3z = 0; 3x + 4y + 4z = 0 ; 7x + 10y + 12z =0.

1 2 3   x 0 
   y  
Solution: Taking A = 3 4 4  , X =   and O= 0
7 10 12  z  0

Page | 30
We get the system of equations as AX = 0

1 2 3 
 
Consider A = 3 4 4 
7 10 12

Applying R2 → R2 – 3R1 and R3 → R3 – 7R1),we get

1 2 3
 
~ 0 − 2 − 5
0 − 4 − 9

Applying R3 → R3 – 2R2

1 2 3
 0 − 2 − 5
~ 
0 0 1 

This is in Echelon form. Number of non -zero rows is 3.


 The rank of A = 3
 No. of variables = 3
 Number of non-zero solutions is = n - r =3 - 3 = 0.
 x = 0, y = 0, z = 0 is the only solution.
Example2: Solve completely the system of equations :
x + y + w = 0; y + z = 0 ; x + y + z + w = 0 ; x + y + 2z = 0.
Solution: The given equations can be written in the matrix form as AX = 0

1 1 0 1 x 0 
0 1 1 0  y 0 
Where A =
 X=
  0=  
1 1 1 1 z 0 
     
1 1 2 0  w 0 
1 1 0 1
0 1 1 0
Consider A =

1 1 1 1
 
1 1 2 0
Applying R3 → R3 – R1 , R4 → R4 – R1 ,we get

1 1 0 1
0 1 1 0 
~

0 0 1 0
 
0 0 2 −1

Page | 31
Applying R4 → R4 – 2 R3 , we get

1 1 0 1
0 1 1 0 
~

0 0 1 0
 
0 0 0 −1
 Rank of A =4 and Number of variables = 4
 There is no non -zero solution
Hence x = y = z = w = 0 is the only solution.
Example 3: Find all the solutions of system of equations.
x + 2y - z = 0; 2x + y + z = 0; x - 4y + 5z = 0.

1 2 − 1
 1  .Then det A = 1 (5 +4) -2(10-1)-1(-8 - 1)
Solution: Let A = 2 1
1 − 4 5 

=9 -18+9 = 0
 The rank of A = 2 < 3(no. of variables)
Applying R2 → R2 – 2R1 , R3 → R3 – R1 , we get

1 2 − 1
 
A = 0 − 3 3 
0 − 6 6 

Applying R3 → R3 – 2R2 , we get

1 2 − 1
 
A = 0 − 3 3 
0 0 0 

Thus, the rank of A = 2


 Number of non – zero solutions = n – r = 3 - 2 = 1.
From the above matrix , -3y + 3z = 0
y = z
Let z = k . Then y = k
From the above matrix , x + 2y – z = 0
 x = z – 2y
 x = k – 2k = -k
 The solutions are given by x = -k, y = k, z = k .

Page | 32
Example 4: Solve completely the system of equations:
x + y - 2z + 3w = 0; x - 2y + z – w = 0; 4x + y - 5z + 8w = 0; 5x - 7y + 2z - w = 0.
Solution: The given equations can be written in the matrix form as AX = 0
1 1 − 2 3  x 0 
1 − 2 1 − 1  y  
AX =     = 0 
4 1 − 5 8  z 0 
     
5 − 7 2 − 1  w 0 
Applying R2 → R2 – R1 , R3 → R3 – 4R1, R4 → R4 – 5R1, ),we get
1 1 −2 3 
0 − 3 3 − 4 
A ~ 
0 − 3 3 −4
 
0 − 12 12 − 16
Applying R4 → R4/4 , we get
1 1 − 2 3 
0 − 3 3 − 4 
A ~  
0 − 3 3 − 4 
 
0 − 3 3 − 4 
Applying R3 → R3– R2 , R4 → R4 – R2, we get
1 1 − 2 3 
0 − 3 3 − 4 
A ~  
0 0 0 0 
 
0 0 0 0 

This is in Echelon form. We have


Rank A = The number of non -zero rows in this echelon form = 2(r) and number of unknowns = 4
Since r <n. The given system has infinite number of non – trivial solutions.
 Number of independent solutions = 4 – 2 = 2 Now we shall assign arbitrary values to 2
variables and the remaining 2 variables shall be found in terms of these. The given system is
equivalent to

1 1 − 2 3 
    x 0
0 − 3 3 − 4   y   0 
   =  
0 0 0 0   z  0 
    
0 0 0 0   w  0 

This gives the equations x + y - 2z +3w = 0; -3y + 3z - 4w = 0.


5 4
Taking z = k1 and w = k2 We see that x = k1 - k2, Y = k1 - k2 , Z = k1 , w = k2
3 3
constitutes the general solution of the given system.

Page | 33
Example 5: show that the only real number  for which the system
x+ 2y + 3z =  x; 3x + y + 2z =  y; 2x + 3y + z =  z has non -zero solution is 6 and
solve them when  =6.
Solution: The given equations can be written in the matrix form as AX = 0

1 −  2 3   x 0 
 3 1−  2  , X =  y  0 =
 0 
Where A =   
 2 3 1 −    z  0

Here the number of variables = n = 3


The given system of equations possesses a non-zero (non-trivial) solution, if
rank of A < no. of unknowns i.e. Rank of A < 3
For this we must have detA = 0
1−  2 3
3 1−  2 =0
2 3 1− 

6− 6− 6−


Applying R1 → R1+R2+R3 , we get 3 1−  2 =0
2 3 1− 

1 1 1
 (6 -  ) 3 1 −  2 =0
2 3 1− 

Applying C2 → C2- C1 , C3 → C3- C1 we get


1 0 0
(6 -  ) 3 − 2 −  −1 =0
2 1 −1− 

 (6 -  ) [(-2 -  )(-1-  ) +1] = 0


 (6 -  ) ( 2 + 3 + 3) = 0
  = 6 is the only real value and other values are complex.
When  = 6, the given system becomes

− 5 2 3  x 0 
 3 −5 2   y  
    = 0 
 2 3 − 5  z  0

Page | 34
Applying R2 → 5R2 +3R1 , R3 → 5R3 + 2R1,we get

− 5 2 3   x  0 
 0 − 19 19   y  0 
~    = 
 0 19 − 19  z  0

Applying R3 → R3 + R2, we get

− 5 2 3   x 0 
 0 − 19 19  y   
~    = 0 
 0 0 0   z  0

 -5x + 2y + 3z = 0
 -19y + 19z = 0
y = z
Since Rank of A < Number of unknowns. i.e. r = 2 < n =3
 The given system has infinite number of non-trivial solutions.
Set n - r = 3 -2 = 1. Let z = k  y = k and from the above equations -5x +2k + 3k = 0
 x=k
 x = k , y = k , z = k is the solution.

Eigen Values & Eigen Vectors


Def: Characteristic vector of a matrix:
Let A= [ aij] be an n x n matrix. A non-zero vector X is said to be a Characteristic Vector of A if
there exists a scalar such that AX=λX.
Note: If AX=λX (X≠0), then we say ‘λ’ is the Eigen value (or) characteristic root of ‘A’.
5 4 1
Eg: Let 𝐴 = [ ] 𝑋 =[ ]
1 3 1
5 4 1 1 1
𝐴𝑋 = [ ] [ ] = [ ] = 1. [ ]
1 3 1 1 1
𝐴𝑋 = 1. 𝑋
1
Here Characteristic vector of A is [ ] and Characteristic root of A is “1”.
1
Note: We notice that an Eigen value of a square matrix A can be 0. But a zero vector cannot be an
Eigen vector of A.
Method of finding the Eigen vectors of a matrix.
Let A = [aij] be a nxn matrix. Let X be an Eigen vector of A corresponding to the Eigen value λ.
Then by definition AX = λX.
AX = λIX
AX –λIX = 0

Page | 35
(A-λI)X = 0 ------- (1)
This is a homogeneous system of n equations in n unknowns. Will have a non-zero solution X if and
only |A-λI| = 0
A-λI is called characteristic matrix of A
|A-λI| is a polynomial in λ of degree n and is called the characteristic polynomial of A
|A-λI|=0 is called the characteristic equation
Solving characteristic equation of A, we get the roots, these are called the
characteristic roots or Eigen values of the matrix.
Corresponding to each one of these n Eigen values, we can find the characteristic vectors.
Procedure to find Eigen values and Eigen vectors
𝑎11 𝑎12 … 𝑎1𝑛
𝑎21 𝑎22 … 𝑎2𝑛
Let 𝐴 = [ … … … … ] be a given square matrix
𝑎𝑛1 𝑎𝑛2 … 𝑎𝑛𝑛
Characteristic matrix of A is (A-λI )
 a11 −  a12 a1n 
 a a22 −  a2 n 
i.e., A −  I =  21
 
 
 an1 an 2 ann −  
Then the characteristic polynomial is A − I
a11 −  a12 ... a1n
a21 a22 −  ... a2 n
say ( ) = A − I =
... ... ... ...
an1 an 2 ... ann − 
The characteristic equation is |A- we solve the we get n roots, these
are called Eigen values or latent values or proper values.
Let each one of these Eigen values say λ their Eigen vector X corresponding the given value
λ is obtained by solving Homogeneous system
 a11 −  a12 a1n   x1  0
 a a22 −  a2 n   x2  0
 21 = and determining the non-trivial solution.
    
    
 an1 an 2 ann −    xn  0

PROBLEMS
8 −4
1. Find the Eigen values and the corresponding Eigen vectors of [ ]
2 2
8 −4
Sol: Let 𝐴 = [ ]
2 2
Characteristic equation of A is A −  I = 0

Page | 36
8−λ −4
| |=0
2 2−λ

8 − λ −4 𝑥1
Consider the system [ ][ ] = 0
2 2 − λ 𝑥2
Eigen vector corresponding to λ=4:
Put in the above system, we get
4 −4 𝑥1 0
 [ ][ ] = [ ]
2 −2 𝑥2 0

 x    1
Let x1 =  , Eigen vector is  1  =   =   
 x2    1
1
[ ] is a Eigen vector of matrix A corresponding to the Eigen value λ=4
1
Eigen vector corresponding to λ=6:

2 −4 𝑥1 0
[ ][ ] = [ ]
2 −4 𝑥2 0

from (1) and (2) we have x1 = 2x2


Say x2 =   x1 = 2

 2   2
Eigen vector =   =   
  1 
2
[ ] is a Eigen vector of matrix A corresponding to the Eigen value λ=6
1
𝟐 𝟎 𝟏
2. Find the Eigen values and the corresponding Eigen vectors of the matrix [𝟎 𝟐 𝟎]
𝟏 𝟎 𝟐
2 0 1
Sol: Let 𝐴 = [0 2 0]
1 0 2
The characteristic equation of matrix A is |A-λI|=0

Page | 37
2−λ 0 1
i.e. |A- λ I| =| 0 2−λ 0 |=0
1 0 2−λ

∴ λ=1, 2, 3
The Eigen values of A is 1, 2, 3
To find the Eigen vector consider the system (𝐴 − λI)X = 0
2−λ 0 1 𝑥1 0
 [ 0 2−λ 0 ] [𝑥2 ] = [0]
1 0 2 − λ 𝑥3 0
Eigen vector corresponding to λ=1:
2−1 0 1 𝑥1 0
[ 0 2−1 0 ] [𝑥2 ] = [0]
1 0 2 − 1 𝑥3 0
1 0 1 1 𝑥 0
 [0 1 0] [𝑥2 ] = [0]
1 0 1 𝑥3 0


𝑥1 −𝛼 −1
[𝑥2 ] = [ 0 ] = 𝛼 [ 0 ]
𝑥3 𝛼 1
−1
[ 0 ] is the Eigenvector to the corresponding Eigen value λ=1.
1
Eigen vector corresponding to λ=2:

2−λ 0 1 𝑥1 0
 [ 0 2−λ 𝑥
0 ] [ 2 ] = [0]
1 0 2 − λ 𝑥3 0

Page | 38
2−2 0 1 𝑥1 0
[ 0 2−2 𝑥
0 ] [ 2 ] = [0]
1 0 2 − 2 𝑥3 0
0 0 1 𝑥1 0
 [0 0 0] [𝑥2 ] = [0]
1 0 0 𝑥3 0

𝑥1 0 0
𝑥
[ 2 ] = [𝛼 ] = 𝛼 [1]
𝑥3 0 0
0
The Eigen vector is[1]
0
Eigen vector corresponding to λ=3:
2−λ 0 1 𝑥1 0
 [ 0 2−λ 0 ] [𝑥2 ] = [0]
1 0 2 − λ 𝑥3 0

2−3 0 1 𝑥1 0
[ 0 2−3 𝑥
0 ] [ 2 ] = [0]
1 0 2 − 3 𝑥3 0
−1 0 1 𝑥1 0
 [ 0 −1 0 ] [𝑥2 ] = [0]
1 0 −1 𝑥3 0

 x1    1
 x  =  0  =  0 
 2    
 x3    1

1
The Eigen vector is [0]
1
𝟔 −𝟐 𝟐
3. Find the Eigen values and the corresponding Eigen vectors of the matrix [−𝟐 𝟑 −𝟏]
𝟐 −𝟏 𝟑
6 −2 2
Sol: Given 𝐴 = [−2 3 −1]
2 −1 3
The characteristic equation of matrix A is |A-λI|=0
6 − λ −2 2
i.e. |A- λ I| =| −2 3 − λ −1 | = 0 ≡ 𝜆3 − 𝑆1 𝜆2 + 𝑆2 𝜆 − 𝑆3 = 0
2 −1 3−λ
Page | 39
𝑆1= SUM OF DIAGONAL ELEMENTS OF A 6+3 +3=12
3 −1 6 2 6 −2
𝑆2 = SUM OF MINORS OF DIAGONAL ELEMENTS OF A= | |+| |+| |=8+14+14=36
−1 3 2 3 −2 3
𝑆3 = DET A = 32
 (6 − λ)[(3 − λ)2 − 1] + 2[−2(3 − λ) + 2] + 2[2 − 2(3 − λ)] = 0
 (6 − λ)[λ2 − 6λ + 8] + 2[2λ − 4] + 2[2λ − 4] = 0
 λ3 − 12λ2 + 36λ − 32 = 0
λ = 2, 2, 8
The Eigen values of A is 2, 2, 8
The Eigen vector of matrix A corresponding to𝛌 = 𝟐;
To find the Eigen vector consider the system (𝐴 − λI)X = 0 (𝐴 − 2I)X = 0

6−2 −2 2 𝑥1 0
 [ −2 3 − 2 𝑥
−1 ] [ 2 ] = [0]
2 −1 3 − 2 𝑥3 0
4 −2 2 𝑥1 0
 [−2 1 −1] [𝑥2 ] = [0]
2 −1 1 𝑥3 0
 4𝑥1 − 2𝑥2 + 2𝑥3 = 0
−2𝑥1 + 𝑥2 − 𝑥3 = 0
2𝑥1 − 𝑥2 + 𝑥3 = 0
By observing that three equations are identical
Therefore, we have to take two arbitrary constants for any two variables
Let 𝑥1 = 𝑘1 , 𝑥2 = 𝑘2
From third equation 2𝑘1 − 𝑘2 + 𝑥3 = 0
𝑥3 = −2𝑘1 + 𝑘2
𝑥1 𝑘1 𝑘1 0 1 0
𝑥
X = [ 2] = [ 𝑘2 ] = [ 0 ] + [𝑘2 ] = 𝑘1 [ 0 ] + 𝑘2 [1]
𝑥3 −2𝑘1 + 𝑘2 −2𝑘1 𝑘2 −2 1
1 0
∴ 𝑋1 = [ 0 ] 𝑋2 = [1]
−2 1
The Eigen vector of matrix A corresponding to𝛌 = 𝟖;
To find the Eigen vector consider the system (𝐴 − λI)X = 0 (𝐴 − 8I)X = 0
6−8 −2 2 𝑥1 0
 [ −2 3 − 8 𝑥
−1 ] [ 2 ] = [0]
2 −1 3 − 8 𝑥3 0
−2 −2 2 𝑥 1 0
 [−2 −5 −1] [𝑥2 ] = [0]
2 −1 −5 𝑥3 0

Page | 40
 −2𝑥1 − 2𝑥2 + 2𝑥3 = 0
−2𝑥1 − 5𝑥2 − 𝑥3 = 0
2𝑥1 − 𝑥2 − 5𝑥3 = 0
By solving above three equations we get 𝑥1 = 2, 𝑥2 = −1, 𝑥3 = 1
𝑥1 2
𝑥
X3 = [ 2 ] = [−1]
𝑥3 1
1 0 2
The Eigen vectors are 𝑋1 = [ 0 ] 𝑋2 = [1] X3 = [−1]
−2 1 1
𝟖 −𝟔 𝟐
4. Find the Eigen values and the corresponding Eigen vectors of the matrix [−𝟔 𝟕 −𝟒]
𝟐 −𝟒 𝟑
8 −6 2
Sol: Given 𝐴 = [−6 7 −4]
2 −4 3
The characteristic equation of matrix A is |A-λI|=0
8 − λ −6 2
i.e. |A- λ I| =| −6 7 − λ −4 | = 0
2 −4 3−λ
 (8 − λ)[(7 − λ)(3 − λ) − 16] + 6[−6(3 − λ) + 8] + 2[24 − 2(7 − λ)] = 0
 (8 − λ)[λ2 − 10λ + 5] + 6[6λ − 10] + 2[2λ − 10] = 0
 λ3 − 18λ2 + 45λ = 0
λ = 0, 3, 15
The Eigen values of A is 0, 3, 15
The Eigen vector of matrix A corresponding to𝛌 = 𝟎;
To find the Eigen vector consider the system (𝐴 − λI)X = 0 (𝐴 − 0I)X = 0
8−0 −6 2 𝑥1 0
 [ −6 7 − 0 𝑥
−4 ] [ 2 ] = [0]
2 −4 3 − 0 𝑥3 0
8 −6 2 𝑥1 0
𝑥
 [−6 7 −4] [ 2 ] = [0]
2 −4 3 𝑥3 0
 8𝑥1 − 6𝑥2 + 2𝑥3 = 0
−6𝑥1 + 7𝑥2 − 4𝑥3 = 0
2𝑥1 − 4𝑥2 + 3𝑥3 = 0
By solving above any of the two equations we get 𝑥1 = 1, 𝑥2 = 2, 𝑥3 = 2
1
Eigen vector 𝑋1 = [2]
2
The Eigen vector of matrix A corresponding to𝛌 = 𝟑;
To find the Eigen vector consider the system (𝐴 − λI)X = 0 (𝐴 − 3I)X = 0

Page | 41
8−3 −6 2 𝑥1 0
 [ −6 7 − 3 𝑥
−4 ] [ 2 ] = [0]
2 −4 3 − 3 𝑥3 0
5 −6 2 𝑥1 0
 [−6 4 −4] [𝑥2 ] = [0]
2 −4 0 𝑥3 0
 5𝑥1 − 6𝑥2 + 2𝑥3 = 0
−6𝑥1 + 4𝑥2 − 4𝑥3 = 0
2𝑥1 − 4𝑥2 = 0
By solving above any of the two equations we get 𝑥1 = −2, 𝑥2 = −1, 𝑥3 = 2
−2
Eigen vector 𝑋2 = [−1]
2
The Eigen vector of matrix A corresponding to 𝛌 = 𝟏𝟓;
To find the Eigen vector consider the system (𝐴 − λI)X = 0 (𝐴 − 15I)X = 0
8 − 15 −6 2 𝑥1 0
 [ −6 7 − 15 −4 ] [𝑥2 ] = [0]
2 −4 3 − 15 𝑥3 0
−7 −6 2 𝑥1 0
 [−6 −8 −4 ] [𝑥2 ] = [0]
2 −4 −12 𝑥3 0
 −7𝑥1 − 6𝑥2 + 2𝑥3 = 0
−6𝑥1 − 8𝑥2 − 4𝑥3 = 0
2𝑥1 − 4𝑥2 − 12𝑥3 = 0
By solving above any of the two equations we get 𝑥1 = 2, 𝑥2 = −2, 𝑥3 = 1
2
Eigen vector 𝑋3 = [−2]
1
1 −2 2
The Eigen vectors are 𝑋1 = [2] 𝑋2 = [−1] X3 = [−2]
2 2 1
Properties of Eigen Values:
1. The sum of the Eigen values of a matrix A is same as trace of the matrix A.

Proof: To prove this theorem, we consider third order square matrix


𝑎11 𝑎12 𝑎13
A= [ 21 𝑎22 𝑎23 ]
𝑎
𝑎31 𝑎32 𝑎33
The characteristic polynomial of A is
= |𝐴 − 𝜆𝐼|
𝑎11 − 𝜆 𝑎12 𝑎13
= | 𝑎21 𝑎22−𝜆 𝑎23 |
𝑎31 𝑎32 𝑎33−𝜆
3 2 (𝑎
= −𝜆 + 𝜆 11 + 𝑎22 + 𝑎33 ) − 𝜆 (𝑎21 𝑎12 + 𝑎13 𝑎31 + ⋯ … ) + (𝑎11 𝑎23 𝑎32 + 𝑎12 𝑎23 𝑎32 + ⋯ )
We have
−𝜆2 𝑐𝑜𝑒𝑓𝑓𝑖𝑐𝑖𝑒𝑛𝑡
Sum of the roots = 𝜆3 𝑐𝑜𝑒𝑓𝑓𝑖𝑐𝑖𝑒𝑛𝑡

Page | 42
−(𝑎11 +𝑎22 +𝑎33 )
= −1
= 𝑎11 + 𝑎22 + 𝑎33
= Trace of A
Hence the result.
2. The product of the Eigen values of a matrix A is equal to its determinant.

Proof: Let 𝜆1 , 𝜆2 … … . . 𝜆𝑛 be the Eigen values of 𝐴𝑛×𝑛 .Then


|𝐴 − 𝜆𝐼| = (−1)𝑛 (𝜆 − 𝜆1 )(𝜆 − 𝜆2 ) … … … . (𝜆 − 𝜆𝑛 ) = 0
Put 𝜆 = 0
|𝐴| = (−1)𝑛 (−𝜆1 )(−𝜆2 ) … … … . (−𝜆𝑛 )
= (−1)𝑛 (−1)𝑛 (𝜆1 𝜆2 … … … . 𝜆𝑛 )

= (−1)2𝑛 (𝜆1 𝜆2 … … … . 𝜆𝑛 )
|𝐴| = 𝜆1 𝜆2 … … … . 𝜆𝑛
Hence the result.
3. If 𝝀 is an Eigen value of A corresponding to the Eigen vector X then 𝝀𝒏 is the Eigen value of
𝑨𝒏 corresponding to the Eigen vector X.
Proof:
Since 𝜆 is an Eigen value of A corresponding to the Eigen vector X, we have
𝐴𝑋 = 𝜆 𝑋 -----------------------------1
Pre multiplying equation 1 by A
𝐴(𝐴𝑋) = A (𝜆 𝑋)
(𝐴𝐴)𝑋 = 𝜆 (𝐴 𝑋)
𝐴2 𝑋 = 𝜆 (𝜆 𝑋)
𝐴2 𝑋 = 𝜆2 𝑋
Hence 𝜆2 is Eigen value of 𝐴2 with 𝑋 itself as the corresponding Eigen Vector.
Thus the theorem is true for 𝑛 = 2.
Let the result be true for 𝑛 = 𝑘.
Then 𝐴𝑘 𝑋 = 𝜆𝑘 𝑋
Pre multiplying this by A
𝐴 (𝐴𝑘 𝑋) = 𝐴(𝜆𝑘 𝑋)
𝐴𝑘+1 𝑋 = 𝜆𝑘 (𝐴𝑋)
= 𝜆𝑘 (𝜆𝑋) (Since 𝑨𝑿 = 𝝀 𝑿 )
𝑘+1
=𝜆 𝑋
𝐴𝑘+1 𝑋 = 𝜆𝑘+1 𝑋
Which implies that 𝜆𝑘+1 is Eigen value of 𝐴𝑘+1 with X itself as the corresponding Eigen
Vector.
Hence by the principle of mathematical induction, the theorem is true for all positive integers n.

4. A square matrix A and it’s transpose 𝑨𝑻 have the same Eigen values.
Proof:
We have (𝐴 − 𝜆𝐼)𝑇 = 𝐴𝑇 − 𝜆𝐼 𝑇
= 𝐴𝑇 − 𝜆𝐼
|(𝐴 − 𝜆𝐼)𝑇 | =|𝐴𝑇 − 𝜆𝐼| or
𝑇
|𝐴 − 𝜆𝐼| =|𝐴 − 𝜆𝐼| [Since |𝑨𝑻 | = |𝑨| ]
Page | 43
∴ |𝐴 − 𝜆𝐼| = 0 if and only if |𝐴𝑇 − 𝜆𝐼| = 0
i.e., 𝜆 is an Eigen value of A if and only if 𝜆 is an Eigen value of 𝐴𝑇 .
Thus the Eigen values of A and 𝐴𝑇 are same.

5. If 𝝀 is an Eigen value of the matrix A corresponding to the Eigen vector X then 𝒌 + 𝝀 is an


Eigen value of the matrix 𝑨 + 𝒌𝑰.
Proof:
Let 𝜆 be an Eigen value of the matrix A corresponding to the Eigen vector X then
𝐴𝑋 = 𝜆 𝑋 -----------------------------1
Now (𝐴 + 𝑘𝐼) 𝑋 = 𝐴𝑋 + 𝑘𝐼𝑋

= 𝜆𝑋 + 𝑘𝑋 (Since 𝑨𝑿 = 𝝀 𝑿 )
= (𝜆 + 𝑘)𝑋
(𝐴 + 𝑘𝐼) 𝑋 = (𝜆 + 𝑘)𝑋 -----------------
From Equation 2 , we see that that the scalar 𝑘 + 𝜆 is an 2 Eigen value of the matrix
𝐴 + 𝑘𝐼 and X is a corresponding Eigen vector.
6. If 𝝀𝟏 , 𝝀𝟐 … … . . 𝝀𝒏 are the Eigen values of a matrix A, then 𝒌𝝀𝟏 , 𝒌𝝀𝟐 … … . . 𝒌𝝀𝒏 are the Eigen
values of the matrix KA, where k is a non-zero scalar.
Proof:
Let A be a square matrix of order n.
Then |𝑘𝐴 − 𝜆𝑘𝐼| = |𝑘(𝐴 − 𝜆𝐼)|
= 𝑘 𝑛 |𝐴 − 𝜆𝐼|
Since 𝑘 ≠ 0, therefore |𝑘𝐴 − 𝜆𝑘𝐼| = 0 if and only if |𝐴 − 𝜆𝐼| = 0
i.e., 𝑘𝜆 is an Eigen value of 𝑘𝐴 if and only if 𝜆 is an Eigen value of A.
Thus 𝑘𝜆1 , 𝑘𝜆2 … … . . 𝑘𝜆𝑛 are the Eigen Values of the Matrix 𝑘𝐴 if 𝜆1 , 𝜆2 … … . . 𝜆𝑛 are the
Eigen values of the matrix A.

7. If 𝝀𝟏 , 𝝀𝟐 … … . . 𝝀𝒏 are the Eigen Values of A, then (𝝀𝟏 − 𝒌)(𝝀𝟐 − 𝒌) … … … . (𝝀𝒏 − 𝒌) are the
Eigen Values of the matrix (𝑨 − 𝒌𝑰), where k is a non-zero scalar.
Proof:
Since 𝜆1 , 𝜆2 … … . . 𝜆𝑛 are the Eigen values of A, then the characteristic polynomial of A is
|𝐴 − 𝜆𝐼| = (𝜆1 − 𝜆)(𝜆2 − 𝜆) … … … . (𝜆2 − 𝜆) -----------------------------
1
Thus the Characteristic polynomial of (𝐴 − 𝑘𝐼) is
|𝐴 − 𝑘𝐼 − 𝜆𝐼| = |𝐴 − (𝑘 + 𝜆)𝐼|
= [𝜆1 − (𝜆 + 𝑘)] [𝜆2 − (𝜆 + 𝑘)] ……[𝜆𝑛 − (𝜆 + 𝑘)] (Since from 1 )
= [(𝜆1 − 𝑘) − 𝜆] [(𝜆2 − 𝑘) − 𝜆] ………………….. [(𝜆𝑛 − 𝑘) − 𝜆]
This shows that the Eigen values of (𝐴 − 𝑘𝐼) are(𝜆1 − 𝑘)(𝜆2 − 𝑘) … … … . (𝜆𝑛 − 𝑘).
8. Prove that the Eigen values of 𝑨−𝟏 are the reciprocals of the Eigen values of A.
(Or)
If 𝝀 is an Eigen value of a non-singular matrix A corresponding to the Eigen vector X. Then
𝝀−𝟏 is an Eigen value of 𝑨−𝟏 and corresponding Eigen vector X itself.

Proof:
Since A is non-singular and product of the Eigen values is equal to |𝐴| , it follows that none of
the Eigen values of A is 0.

Page | 44
If 𝜆 is an Eigen value of the non-singular matrix A and X is the corresponding Eigen vector,
𝜆 ≠ 0 and 𝐴𝑋 = 𝜆 𝑋
Pre multiplying this with 𝐴−1 , we get
𝐴−1 (𝐴𝑋) = 𝐴−1 (𝜆 𝑋)
(𝐴−1 𝐴)𝑋 = 𝜆(𝐴−1 𝑋)
𝐼𝑋 = 𝜆(𝐴−1 𝑋)
𝑋 = 𝜆(𝐴−1 𝑋)
𝜆−1 𝑋 = 𝐴−1 𝑋
𝐴−1 𝑋 = 𝜆−1 𝑋 Since 𝝀 ≠ 𝟎

Hence by the definition, it follows that 𝜆−1 is an Eigen value of 𝐴−1 and X is the corresponding Eigen
vector.

|𝑨|
9. If 𝝀 is an Eigen value of a non-singular matrix A, then is an Eigen value of the matrix
𝝀
𝒂𝒅𝒋 𝑨.
Proof:
Since 𝜆 is an Eigen value of a non-singular matrix, therefore 𝜆 ≠ 0.
Also 𝜆 is an Eigen value of A implies that there exist a non-zero vector X such that
𝐴𝑋 = 𝜆 𝑋 ------------------1
Pre multiplying 1 by 𝑎𝑑𝑗 𝐴
(𝑎𝑑𝑗 𝐴) 𝐴𝑋 = (𝑎𝑑𝑗 𝐴) 𝜆𝑋
[(𝑎𝑑𝑗 𝐴) 𝐴]𝑋 = 𝜆[(𝑎𝑑𝑗 𝐴) 𝑋]
|𝐴| 𝐼 𝑋 = 𝜆(𝑎𝑑𝑗 𝐴)𝑋 [Since (𝒂𝒅𝒋 𝑨)𝑨 = |𝑨| 𝑰 ]
|𝐴| 𝑋 = 𝜆(𝑎𝑑𝑗 𝐴)𝑋
|𝐴|
𝑋 = (𝑎𝑑𝑗 𝐴)𝑋
𝜆
|𝐴|
(𝑎𝑑𝑗 𝐴)𝑋 = 𝑋
𝜆
|𝐴|
Since X is a non-zero vector, therefore from the relation 1 . It is clear that is an Eigen
𝜆
value of the matrix 𝑎𝑑𝑗 𝐴.
𝟏
10. If 𝝀 is an Eigen value of an orthogonal matrix, then 𝝀 is also an Eigen value.
Proof:
1
We know that if 𝜆 is an Eigen value of a matrix A, then 𝜆 is an Eigen value of 𝐴−1 .
Since A is an orthogonal matrix, therefore
𝐴−1 = 𝐴𝑇
1
is an Eigen value of 𝐴𝑇 .
𝜆
But the matrices A and 𝐴𝑇 have the same Eigen values. Since the determinants |𝐴 − 𝜆𝐼|
and |𝐴𝑇 − 𝜆𝐼|are same.
1
Hence 𝜆 is also an Eigen value of A.
11. The Eigen values of a diagonal matrix are its diagonal elements.
Proof: Let

Page | 45
𝑎11 0 0 …… 0
0 𝑎22 0 …… 0
0 0 𝑎33 … … 0
𝐷=
. . . …… .
. . . …… .
[ 0 0 0 …… 𝑎𝑛𝑛 ]
The characteristic equation of D is
|𝐷 − 𝜆𝐼| = 0
𝑎11 − 𝜆 0 0 …… 0
0 𝑎22 − 𝜆 0 …… 0
| |
0 0 𝑎33 − 𝜆 … … 0
=0
| . . . …… . |
. . . …… .
0 0 0 …… 𝑎𝑛𝑛−𝜆

⇒ (𝑎11 − 𝜆)(𝑎22 − 𝜆)(𝑎33 − 𝜆) … … … . (𝑎𝑛𝑛 − 𝜆) = 0


⇒ 𝜆 = 𝑎11 , 𝑎22 , 𝑎33 … … … … 𝑎𝑛𝑛 are the Eigen values of D.

The Eigen values of a triangular matrix are the diagonal elements.


Proof:
𝑎11 𝑎12 𝑎13 … … 𝑎1𝑛
0 𝑎22 𝑎23 … … 𝑎2𝑛
0 0 𝑎33 … … 𝑎3𝑛
Let 𝐴= . . . …… . be the upper triangular matrix.
. . . …… .
[ 0 0 0 … … 𝑎𝑛𝑛 ]
The Characteristic equation of A is
|𝐴 − 𝜆𝐼| = 0
𝑎11 − 𝜆 𝑎12 𝑎13 …… 𝑎1𝑛
0 𝑎22 − 𝜆 𝑎23 …… 𝑎2𝑛
| |
0 0 𝑎33 − 𝜆 … … 𝑎3𝑛
=0
| . . . … … . |
. . . …… .
0 0 0 … … 𝑎𝑛𝑛−𝜆
⇒ (𝑎11 − 𝜆)(𝑎22 − 𝜆)(𝑎33 − 𝜆) … … … . (𝑎𝑛𝑛 − 𝜆) = 0
⇒ 𝜆 = 𝑎11 , 𝑎22 𝑎33 … … … … 𝑎𝑛𝑛
Hence the Eigen values are the diagonal elements.
12. Eigen values of two similar matrices are same.
Proof:
Let A and B be two similar matrices so that
𝐵 = 𝑃−1 𝐴𝑃
To prove that A and 𝑃−1 𝐴𝑃 have the same Eigen values.
Let 𝜆 be Eigen value of B then,
|𝐵 − 𝜆𝐼| = 0
|𝑃 𝐴𝑃 − 𝜆𝑃−1 𝑃| = 0
−1

|𝑃−1 𝑃(𝐴 − 𝜆𝐼)| = 0


|𝑃−1 ||𝑃| |𝐴 − 𝜆𝐼| = 0
|𝐴 − 𝜆𝐼| = 0 𝒔𝒊𝒏𝒄𝒆 |𝑷−𝟏 ||𝑷| = 𝟏
|𝐵 − 𝜆𝐼| = 0 ⇒ |𝐴 − 𝜆𝐼| = 0
Page | 46
Hence A and B have the same Eigen values.

13. If A and B are square matrices and if A is invertible then the matrices 𝑨−𝟏 𝑩 and 𝑩𝑨−𝟏 have
the same Eigen values.
Proof:
Given A is invertible, then 𝐴−1 exists.
Now 𝐴−1 𝐵 = 𝐴−1 𝐵 𝐼
= 𝐴−1 𝐵 (𝐴−1 𝐴)
= 𝐴−1 (𝐵𝐴−1 )𝐴
⇒ 𝐴−1 𝐵 = 𝐴−1 (𝐵𝐴−1 )𝐴 ---------------- 1
By using the above property, matrices 𝐵𝐴−1 and 𝐴−1 (𝐵𝐴−1 )𝐴 have the same Eigen
values.
Now by 1 , the matrices 𝐴−1 𝐵 and 𝐵𝐴−1 have the same Eigen values.

14. The Eigen values of a real symmetric matrix are always real.
Proof:
Let 𝜆 be an Eigen value of a real symmetric matrix A and let X be the corresponding Eigen
vector. Then 𝐴𝑋 = 𝜆 𝑋 ------------------ 1
Taking the Conjugate of 1
𝐴̅ 𝑋̅ = 𝜆̅ 𝑋̅
Taking the transpose on both sides
(𝐴̅ 𝑋̅)𝑇 = (𝜆̅ 𝑋̅ )𝑇
(𝑋̅)𝑇 (𝐴̅ )𝑇 = 𝜆̅ (𝑋̅)𝑇
Since A is symmetric, we have
𝐴̅ = 𝐴 and 𝐴𝑇 = 𝐴
∴ (𝑋̅)𝑇 𝐴 = 𝜆̅ (𝑋̅)𝑇
Post multiplying by X, we get
(𝑋̅)𝑇 𝐴 𝑋 = 𝜆̅ (𝑋̅)𝑇 𝑋 ------------2
Pre multiplying 1 with(𝑋̅)𝑇 , we get
(𝑋̅)𝑇 𝐴𝑋 = (𝑋̅)𝑇 𝜆 𝑋 ------------ 3
-
2 3

⇒ (𝜆 − 𝜆̅ ) (𝑋̅)𝑇 𝑋 = 0
⇒ (𝜆 − 𝜆̅ ) = 0 ̅ )𝑻 𝑿 ≠ 𝟎
𝒔𝒊𝒏𝒄𝒆 (𝑿
⇒ 𝜆 = 𝜆̅
⇒ 𝜆 is real
Hence the result.
15. If 𝝀 is an Eigen value of A,then prove that the Eigen value of 𝑩 = 𝒂𝟎 𝑨𝟐 + 𝒂𝟏 𝑨 + 𝒂𝟐 𝑰 is
𝒂𝟎 𝝀𝟐 + 𝒂𝟏 𝝀 + 𝒂𝟐 .
Proof:
If X be the Eigen vector corresponding to the Eigen value , then
𝐴𝑋 = 𝜆 𝑋 ------------------1
1
Page | 47
Pre multiplying by A on both sides of
𝐴(𝐴𝑋) = 𝐴(𝜆 𝑋)
(𝐴𝐴)𝑋 = 𝜆 (𝐴 𝑋)
𝐴2 𝑋 = 𝜆 (𝜆 𝑋) (Since 𝑨𝑿 = 𝝀 𝑿)
2 2
𝐴 𝑋 =𝜆 𝑋
Hence 𝜆 is Eigen value of 𝐴2 with 𝑋 itself as the corresponding Eigen Vector.
2

We have
𝐵 = 𝑎0 𝐴2 + 𝑎1 𝐴 + 𝑎2 𝐼
𝐵𝑋 = (𝑎0 𝐴2 + 𝑎1 𝐴 + 𝑎2 𝐼)𝑋
𝐵𝑋 = 𝑎0 𝐴2 𝑋 + 𝑎1 𝐴𝑋 + 𝑎2 𝑋
𝐵𝑋 = 𝑎0 𝜆2 𝑋 + 𝑎1 𝜆𝑋 + 𝑎2 𝑋
𝐵𝑋 = (𝑎0 𝜆2 + 𝑎1 𝜆 + 𝑎2 )𝑋

This shows that 𝑎0 𝜆2 + 𝑎1 𝜆 + 𝑎2 is an Eigen value of B and the corresponding Eigen Vector of
B is X.

16. Prove that the two Eigen vectors corresponding to the two different Eigen Values are
linearly independent.
Proof:
Let A be a square matrix. Let 𝑋1and 𝑋2 be the two Eigen vectors of A corresponding to two
distinct Eigen
values 𝜆1 and 𝜆2 .Then
𝐴𝑋1 = 𝜆1 𝑋1And 𝐴𝑋2 = 𝜆2 𝑋2 ---------------------1
Now we shall prove that the Eigen vectors 𝑋1and 𝑋2 are linearly independent.
Let us assume that the 𝑋1 and 𝑋2 are linearly dependent.
Then for two scalars 𝑘1 and 𝑘2 not both zeroes such that 𝑘1 𝑋1 + 𝑘2 𝑋2 = 0 ------------------ 2
Multiplying both sides of 2 by A, we get
𝐴 ( 𝑘1 𝑋1 + 𝑘2 𝑋2 ) = 𝐴(0)
𝐴 ( 𝑘1 𝑋1 ) + 𝐴(𝑘2 𝑋2 ) = 0
𝑘1 (𝐴𝑋1 ) + 𝑘2 (𝐴𝑋2 ) = 0
𝑘1 (𝜆1 𝑋1 ) + 𝑘2 (𝜆2 𝑋2 ) = 0 --------------------3 (Since from 1 )

3 - 𝜆2 1 ⇒ 𝑘1 (𝜆1 − 𝜆2 )𝑋1 = 0
⇒ 𝑘1 = 0 Since 𝝀𝟏 ≠ 𝝀𝟐 & 𝑿𝟏 ≠ 𝟎
Similarly 𝑘2 = 0
But this contradicts our assumption that 𝑘1 , 𝑘2 are not zeroes. Hence our assumption that
𝑋1 & 𝑋2 are Linearly independent is wrong.
Hence the statement is true.
17. For a real symmetric matrix, the Eigen vectors corresponding to two distinct Eigen values
are orthogonal.
Proof :
Let 𝜆1 and 𝜆2 be Eigen values of a real symmetric matrix A and let 𝑋1 & 𝑋2 be the
corresponding Eigen vectors.
Let 𝜆1 ≠ 𝜆2 , we have to show that 𝑋1 𝑇 𝑋2 = 0
1
Page | 48
We have 𝐴𝑋1 = 𝜆1 𝑋1---------------------
𝐴𝑋2 = 𝜆2 𝑋2 ---------------------
2
Pre multiplying 1 by 𝑋2 𝑇
𝑋2 𝑇 𝐴𝑋1 = 𝑋2 𝑇 𝜆1 𝑋1
Taking Transpose, we have
(𝑋2 𝑇 𝐴𝑋1 )𝑇 = (𝑋2 𝑇 𝜆1 𝑋1 )𝑇
⇒ 𝑋1 𝑇 𝐴𝑇 𝑋2 = 𝜆1 𝑋1 𝑇 𝑋2
⇒ 𝑋1 𝑇 𝐴𝑋2 = 𝜆1 𝑋1 𝑇 𝑋2 --------------------3 (Since 𝑨𝑻 = 𝑨)
Pre multiplying 2 by 𝑋1 𝑇
𝑋1 𝑇 𝐴𝑋2 = 𝜆2 𝑋1 𝑇 𝑋2 -----------------------
4
4
From 3 and 4
4
(𝜆1 − 𝜆2 )𝑋1 𝑇 𝑋2 = 0

⇒ 𝑋1 𝑇 𝑋2 = 0 Since 𝝀𝟏 ≠ 𝝀𝟐
∴ 𝑋1 is orthogonal to 𝑋2

5. Find the Eigen values and Eigen vectors of the matrix A and it’s inverse, where A =

Sol: Given A =

The characteristic equation of A is given by |A-λI| = 0

Eigen vector corresponding to λ=1

and x1 = 

  1
X =  0  =  0 is the solution where  is arbitrary constant
 
 0  0

Page | 49
1 
 X = 0 Is the Eigen vector corresponding to  = 1
0

Eigen vector corresponding to λ=2

5 x3 = 0  x3 = 0

− x1 + 3x2 = 0  x1 = 3x2
Let x2 = k
x1 = 3k
3k  3
X =  k  = k 1
 
 0  0

 3
 X = 1 is the Eigen vector corresponding to  = 2
0

Eigen vector corresponding to λ=3

− 2 3 4  x1  0
For  = 3, becomes  0 − 1 5  x2  = 0
 0 0 0  x3  0

X=

19
 X = 10 is the Eigen vector corresponding to  = 3
 2 
Page | 50
Eigen values of A –1 are

1 1
 Eigen values of A−1 are 1, ,
2 3
We know Eigen vectors of A –1 are same as Eigen vectors of A.

6.

Let f(A) =
Then Eigen values of f(A) are f(1), f(3) and f(-2)
f(1) = 3(1)3+5(1)2-6(1)+2(1) = 4
f(3) = 3(3)3+5(3)2-6(3)+2(1) = 110
f(-2) = 3(-2)3+5(-2)2-6(-2)+2(1) = 10
Eigen values of are 4,110,10

 3i 2 + i
7. Find the Eigen values of A=  
 −2 + i − i 
 3i 2 + i
Sol: we have A=  
 −2 + i − i 
 −3i 2 − i   3i − 2 + i 
and A = 
T
So A = 
 −2 − i i 
2 + i − i 

 A = − AT
Thus A is a skew-Hermitian matrix.
Page | 51
The characteristic equation of A is A −  I = 0
3i −  − 2 + i
A = =0
T

−2+i −i −
 − 2i + 8 = 0
2

  = 4i, −2i are the Eigen values of A

1 3
 i 
8. Find the Eigen values of A =  2 2 
 3 1i
 2 2 
 1 3
− i 
Now A =  2 2  and
 3 − 1 i
 2 2 
 1 3
− i 
( ) T
A = 2 2 
 3 − 1 i
 2 2 
T 1 0 
We can see that A . A =  =I
0 1 
Thus A is a unitary matrix
 The characteristic equation is A −  I = 0

1 3
i−
 2 2 =0
3 1
i−
2 2

3 1 − 3 1
Which gives  = + i and + i and
2 2 2 2

 = 1/ 2 3 + 1/ 2i
Hence above  values are Eigen values of A.

Cayley - Hamilton Theorem:


Statement: Every square matrix satisfies its own characteristic equation

Page | 52
PROBLEMS

1. Show that the matrix A = satisfies its characteristic equation Hence find A-1

Sol: The Characteristic equation of A is |𝐴 − λI| = 0

C2 → C2+C3

By Cayley – Hamilton theorem, we have A3-A2+A-I=0------(1)

1 − 2 2 − 1 0 0  − 1 2 − 2
A = 1 − 2 3 A2 = − 1 − 1 2 A3 = − 2 2 − 1
0 − 1 2 − 1 0 1  − 1 1 0 

 − 1 2 − 2 − 1 0 0 1 − 2 2 1 0 0
A3 − A2 + A − I = − 2 2 − 1 − − 1 − 1 2 + 1 − 2 3 − 0 1 0
 − 1 1 0  − 1 0 1 0 − 1 2 0 0 1

Multiplying equation (1) with A–1 we get A2 – A + I =A–1

− 1 0 0 1 − 2 2 1 0 0  − 1 2 − 2
A−1 = − 1 − 1 2 − 1 − 2 3 + 0 1 0 = − 2 2 − 1
− 1 0 1 0 − 1 2 0 0 1  − 1 1 0 

2) Using Cayley - Hamilton Theorem find the inverse and A4 of the matrix A =

Sol: Let A =

The characteristic equation is given by |A-λI|=0

Page | 53
7− 2 −2
i.e., − 6 −1−  2 =0
6 2 −1 − 

By Cayley – Hamilton theorem we have A3-5A2+7A-3I=0…..(1)


Multiply equation (1) with A-1 we get

A-1

 25 8 − 8  79 26 − 26

A = − 24 − 7 8  A = − 78 − 25 26 
2  3

 24 8 − 7  78 26 − 25

− 3 − 2 2 
A =  6 5 − 2
1
−1

3
− 6 − 2 5 

Multiply equation (1) with A , we get

𝐴4 = 5𝐴3 − 7𝐴2 + 3𝐴
395 130 −130 175 56 −56 21 6 −6
𝐴4 = [−390 −125 130 ] − [−168 −49 56 ] + [−18 −3 6 ]
390 130 −125 168 56 −69 18 6 −3
241 80 −80
𝐴4 = [−240 −79 80 ]
240 80 −79

Diagonalization of a matrix:
Theorem: If a square matrix A of order n has n linearly independent eigen vectors (X1,X2…Xn)
corresponding to the n eigen values λ1,λ2….λn respectively then a matrix P can be found such that
P-1AP is a diagonal matrix.
Proof: Given that (X1,X2…Xn) be eigen vectors of A corresponding to the eigen values λ1,λ2….λn
respectively and these eigen vectors are linearly independent Define P = (X1,X2…Xn)
Since the n columns of P are linearly independent |P|≠0
Hence P-1 exists
Consider AP = A[X1,X2…Xn]
= [AX1, AX2…..AXn]
= [λX1, λ2X2….λnXn]
Page | 54
1 0 ... 0
0  ... 0 
=[X1,X2…Xn]  2

 ... ... ... ... 


 
0 0 ... n 

Where D = diag (1 , 2 , 3 ,......... ..n )

AP=PD

P–1(AP) = P–1 (PD)  P −1 AP = P −1P D ( )


 P–1AP = (I)D

= diag (1 , 2 , 3 ,......... ..n )

Hence the theorem is proved.


Modal and Spectral matrices:
The matrix P in the above result which diagonalizable the square matrix A is called modal matrix of A
and the resulting diagonal matrix D is known as spectral matrix.
Note 1: If X1,X2…Xn are not linearly independent this result is not true.
2: Suppose A is a real symmetric matrix with n pair wise distinct Eigen values 1 , 2 n then

the corresponding Eigen vectors X1,X2…Xn are pair wise orthogonal.


Hence if P = (e1,e2…en)
Where e1 = (X1 / ||X1||), e2 = (X2 / ||X2||)….en = (Xn)/ ||Xn||
then P will be an orthogonal matrix.
i.e, PTP=PPT=I
Hence P–1 = PT
P–1 PTAP=D

Calculation of powers of a matrix:


We can obtain the power of a matrix by using diagonalization
Let A be the square matrix then a non-singular matrix P can be found such that D = P-1AP
D2= (P–1AP) (P–1AP)
= P–1A(PP–1)AP
= P–1A2P (since PP–1=I)
Similarly D3 = P–1A3P
In general Dn = P–1AnP……..(1)

Page | 55
To obtain An, Pre-multiply (1) by P and post multiply by P–1
Then PDnP–1 = P(P–1AnP)P–1
= (PP–1)An (PP–1) = An

 An = PD n P −1
 1n 0 0 0
 
0  n
0 0  −1
Hence A = P 
n 2
P
 
 
 0 0 0 nn 

PROBLEMS

1. Determine the modal matrix P of A= . Verify that is a diagonal matrix.

Sol: The characteristic equation of A is |A-λI| = 0

i.e,

0
Thus the Eigen values are λ=5, λ=-3 and λ=-3

when λ=5

By solving above we get X1 =

Similarly, for the given Eigen value λ=-3 we can have two linearly independent Eigen vectors

X2 =

Page | 56
=

is a diagonal matrix.
𝟏 𝟎 −𝟏
2. Find a matrix P which transforms the matrix 𝑨 = [𝟏 𝟐 𝟏 ] too diagonal form. Hence
𝟐 𝟐 𝟑
Calculate 𝑨𝟒 .
Sol: Characteristic equation of A is given by |A-λI| = 0

i.e,

If x1, x2, x3 be the components of an Eigen vector corresponding to the Eigen value λ, we
have

[A-λI]X =

i.e, 0.x1+0.x2+0.x3=0 and x1+x2+x3=0

x3=0 and x1+x2+x3=0


x3=0, x1=-x2
x1=1, x2=-1, x3=0
Eigen vector is [1,-1,0]T
Also every non-zero multiple of this vector is an Eigen vector corresponding to λ=1
For λ=2, λ=3 we can obtain Eigen vector [-2,1,2]T and [-1,1,2]T

Page | 57
P=

 0 2 − 1
1
The Matrix P is called modal matrix of A . P-1= −  2 2 0 
2
− 2 − 2 − 1

 1
 0 − 1 2  1 0 − 1  1 − 2 − 1
Now P −1 AP = − 1 − 1 0  1 2 1  − 1 1 1 
 1
1 1  2 2 3   0 2 2 
 2 

 − 1
 1 − 2 − 1 1 0 0   0 − 1
    2

= − 1 1 1  0 16 0  − 1 1 0
 
 0 2 2  0 0 81 − 2 − 2 − 1
 
−49 −50 −40
4
𝐴 = [ 65 66 40 ]
130 130 81

QUADRATIC FORMS
Quadratic form: A homogeneous expression of second degree in any number of variables is called a
quadratic form.
E.g.1. 3x + 5xy – 2y2 is a quadratic form in two variables x and y.
2

E.g.2. 2x2 + 3y2 – 4z2 + 2xy - 3yz + 5zx is a quadratic form in three variables x, y and z.

An expression of the form Q = XTAX = ∑𝑛𝑖=1 ∑𝑛𝑗=1 𝑎𝑖𝑗 𝑥𝑖 𝑥𝑗 , where 𝑎𝑖𝑗 ’s are constants, is called a
quadratic form in n variables x1,x2,…….xn.

Here X is variable matrix and A is a matrix of quadratic form.

To write the matrix of the quadratic form, we have to follow the following procedure.

❖ Write the coefficients of square terms along the diagonal and divide the coefficients of the
product terms xy, yz, zx by 2 and write them at the appropriate places.

𝑥 𝑦 𝑧
𝑥𝑦 𝑥𝑧
𝑥 𝑥2
2 2

Page | 58
𝑥𝑦 𝑦𝑧
𝑦 𝑦2
2 2
𝑥𝑧 𝑦𝑧
𝑧 𝑧2
2 2

E.g.1. Write the matrix of the quadratic form ax2 + 2hxy + by2.

Sol. The given quadratic form is ax2 + 2hxy + by2

𝑥 𝑦

𝑥𝑦
𝑥 𝑥2
2
𝑥𝑦
𝑦 𝑦2
2

𝑎 ℎ
The corresponding matrix is ( )
ℎ 𝑏
E.g.2. Find the matrix of the quadratic form x2 + 2y2 + 3z2 + 4xy + 5yz + 6zx.

Sol. The given quadratic form is x2 + 2y2 + 3z2 + 4xy + 5yz + 6zx

1 2 3 𝑥
5⁄
The matrix of the quadratic form is A= (2 2 2), X = [ ], XT = [𝑥
𝑦 𝑦 𝑧]
3 5⁄2 3 𝑧

1 2 3
E.g.3. Find the quadratic form relating to the symmetric matrix [2 1 3]
3 3 1
1 2 3
Sol. The given symmetric matrix is [2 1 3]
3 3 1
We know that the general form of quadratic form is Q = XTAX,

𝑥 1 2 3
Where X =[ 𝑦 ], XT = [𝑥 𝑦 𝑧] and A = [2 1 3]
𝑧 3 3 1
1 2 3 𝑥
Required quadratic form = XTAX = [𝑥 𝑦 𝑧 ] [2 1 3] [ 𝑦 ]
3 3 1 𝑧
𝑥 + 2𝑦 + 3𝑧
= [𝑥 𝑦 𝑧] [2𝑥 + 𝑦 + 3𝑧]
3𝑥 + 3𝑦 + 𝑧

= x2 + y2 + z2 + 4xy + 6yz + 6zx

Page | 59
CANONICAL FORM (OR) NORMAL FORM OF A QUADRATIC FORM

Let XTAX be a quadratic form in n variables. Then there exists a real non singular linear
transformation X = PY which transforms XTAX to another quadratic form of the type YTDY = 𝜆1 𝑦12 +
𝜆2 𝑦22 + … … … . +𝜆𝑛 𝑦𝑛2 , then YTDY is called the canonical form of XTAX. Where D = diag
[𝜆1 , 𝜆2 , … … … 𝜆𝑛 ].

Definition.1.The number of positive terms in canonical form of a quadratic form is known as the
index of the quadratic form, it is denoted by s.

Definition.2.The difference between the number of positive terms and the number of negative
terms in canonical form of a quadratic form is known as the signature of the quadratic form.

NATURE OF QUADRATIC FORMS

The quadratic form XTAX in n variables is said to be


(i) Positive definite, if r = n and s = n (or) if all the Eigen values of A are positive.
(ii) Negative definite, if r = n and s = 0 (or) if all the Eigen values of A are negative.
(iii) Positive semi definite, if r < n and s = r (or) if all the Eigen values of A≥ 0 and at least one Eigen
value is zero.
(iv) Negative semi definite, if r < n and s = 0 (or) if all the Eigen values of A≤ 0 and at least one
Eigen value is zero.
(v) Indefinite, in all other cases.
Note: Here r is rank of the matrix A, n is the number of variables in X and s is the index of quadratic
form.
E.g.1. find the nature of the quadratic form 2x2 + 2y2 + 2z2 +2yz.

Sol: Given quadratic form is 2x2 + 2y2 + 2z2 +2yz

2 0 0
The matrix of the quadratic form is A = [0 2 1]
0 1 2
The characteristic equation of matrix A is |𝐴 − 𝜆𝐼| = 0

2−𝜆 0 0
| 0 2−𝜆 1 |=0
0 1 2−𝜆
By solving the above we can get the characteristic roots are 1, 2, 3.

Since all the roots are positive, the nature of the quadratic form is positive definite.

Page | 60
Procedure to Reduce Quadratic Form into Canonical Form By Orthogonal transformation

Step1. Write the symmetric matrix A of given quadratic form.


Step2. Find the Eigen values of matrix A
Step3. Find the Eigen vectors of A corresponding to the Eigen values such that the Eigen vectors
must be pair wise orthogonal.
𝑋 𝑋 𝑋
Step4. Find the normalized Eigen vectors e1 = ‖𝑋1 ‖ , e2 = ‖𝑋2 ‖ ,……….. en = ‖𝑋𝑛‖ .
1 2 𝑛

Step5. Form the matrix P containing normalized Eigen vectors of A as column vectors.
i.e. P = [ e1,e2,……….en] then X = PY gives the required orthogonal transformation which
reduces quadratic form to canonical form.
Step6. Since P is an orthogonal matrix, we have P-1 = PT
Therefore D = P-1AP = PTAP, where D is diabolized matrix.
Step7. Write the canonical form using 𝜆1 𝑦12 + 𝜆2 𝑦22 + … … … . +𝜆𝑛 𝑦𝑛2 .

E.g1. Reduce the quadratic form 3x2 + 2y2 + 3z2 – 2xy - 2yz in to the normal form (canonical form).
3 −1 0
Sol: The matrix A of the given quadratic form is [−1 2 −1]
0 −1 3
The characteristic equation of A is |𝐴 − 𝜆𝐼| = 0

3−𝜆 −1 0
| −1 2−𝜆 −1 | = 0
0 −1 3−𝜆
(𝜆 − 3)(𝜆 − 1)(𝜆 − 4) = 0

The Eigen values are 1, 3, 4.

1
The Eigen vector corresponding to Eigen value 𝜆 = 3 is X1 = [ 0 ]
−1
1
The Eigen vector corresponding to Eigen value 𝜆 = 1 is X2 = [ 2 ]
1
1
The Eigen vector corresponding to Eigen value 𝜆 = 4 is X3 = [−1]
1
The three Eigen vectors are mutually orthogonal. Now we can normalize these vectors and
𝑋 𝑋 𝑋
obtain 𝑒1 = ‖𝑋1‖ , e2 = ‖𝑋2‖ , e3 = ‖𝑋3 ‖
1 2 3

Page | 61
1 1
1
√6 √3
√2 2 −1
𝑒1 = [ 0 ], 𝑒2 = √6
, 𝑒3 = √3
−1
1 1
√2 [ √6 ] [ √3 ]

1 1 1
√2 √6 √3
2 −1
Let P = The modal matrix in normalized form = [𝑒1 𝑒2 𝑒3 ] = 0 √6 √3
−1 1 1
[√2 √6 √3 ]

Since P is an orthogonal matrix we have D = PTAP, where D is diagonal matrix.


1 −1 1 1 1
0
√2 √2 3 −1 0 √2 √6 √3
1 2 1 2 −1
D= PTAP = √6 √6 √6
[−1 2 −1] 0 √6 √3
1 −1 1 0 −1 3 −1 1 1
[√3 √3 √3 ] [√2 √6 √3 ]

3 0 0
D = [0 1 0]
0 0 4
The required canonical form (or) normal form is

3 0 0 𝑦1
YTDY = [𝑦1 𝑦2 𝑦3 ] [0 1 0] [𝑦2 ] = 3𝑦12 + 𝑦22 + 4𝑦32
0 0 4 𝑦3
Here the linear transformation is X = PY
1 1 1
𝑥 √2 √6 √3 𝑦1
2 −1
[ ]= 0
𝑦 𝑦
[ 2]
√6 √3
𝑧 −1 1 1 𝑦3
[√2 √6 √3 ]

In the above canonical form


Rank of the matrix is r = 3
Index = s = the number of positive terms in canonical form = 3
n =The number of variables = 3
Since r = n and s = n (or) all Eigen values are positive, the nature of the quadratic form is positive
definite.
Exercise examples
1) Reduce the quadratic form 3x2 + 5y2 + 3z2 - 2yz + 2zx – 2xy to the canonical form and hence find
nature, rank, index and signature of the quadratic form.
2) Reduce the quadratic form 2x2 + 2y2 +2z2 + 2yz into the canonical form by orthogonal reduction
Page | 62
and hence find nature, rank, index and signature of the quadratic form.

Procedure To Reduce Quadratic Form Into Canonical Form By Diagonalization


Step1. Write the symmetric matrix A of the given quadratic form.
Step2. Write the matrix A in the relation form 𝐴𝑛×𝑛 = 𝐼𝑛×𝑛 𝐴 𝐼𝑛×𝑛
Step3. Reduce the matrix A on the hand side to a diagonal matrix
(i) by applying elementary row operations on the left identity matrix and on A on left hand side
(ii) by applying elementary column operations on the right identity matrix and on A on left hand side
Step 4. By applying these operations A = IAI will be reduced to the form D = PTAP
Where D is the diagonal matrix with elements d1,d2,………dn and P is the matrix used in the
linear transformation.
The canonical form is given by YTDY = 𝑑1 𝑦12 + 𝑑2 𝑦22 + ⋯ + 𝑑𝑛 𝑦𝑛2
Note: here some of the coefficients d1,d2,………dn may be zero.
E.g.1. Reduce the quadratic form 3x2 + 2y2 + 3z2 – 2xy – 2yz into canonical form by diagonalization.
3 −1 0
Sol. The matrix A of given quadratic form is A = [−1 2 −1]
0 −1 3
Now 𝐴3×3 = 𝐼3×3 𝐴𝐼3×3

3 −1 0 1 0 0 1 0 0
[−1 2 −1] = [ 0 1 0]A[0 1 0]
0 −1 3 0 0 1 0 0 1
Applying 𝑅2 → 3𝑅2 + 𝑅1 , 𝐶2 → 3𝐶2 + 𝐶1

3 0 0 1 0 0 1 1 0
[0 15 −3] = [ 1 3 0]A[0 3 0]
0 −3 3 0 0 1 0 0 1
Applying 𝑅3 → 5𝑅3 + 𝑅2, 𝐶3 → 5𝐶3 + 𝐶2

3 0 0 1 0 0 1 1 1
[0 15 0 ] = [ 1 3 0 ] A [ 0 3 3 ]
0 0 60 1 3 5 0 0 5
𝑅1 𝐶1 𝑅2 𝐶2 𝑅3 𝐶3
Applying , , , , ,
√3 √3 √15 √15 √60 √60

1 1 1 1
0 0
1 0 0 √3 √3 √15 √60
1 3 3 3
[0 1 0] = √15 √15
0 A 0 √15 √60
0 0 1 1 3 5 5
[ √60 √60 √60 ] [0 0
√60 ]

This is in the form D = PTAP

Page | 63
1 0 0
Where D is the diagonal matrix, D =[0 1 0]
0 0 1
1 1 1
√3 √15 √60
3 3
P= 0 √15 √60
5
[0 0
√60 ]

The required canonical form is YTDY = 𝑦12 + 𝑦22 + 𝑦32

Here the linear transformation is X = PY


1 1 1
𝑥 𝑦1 √3 √15 √60
3 3
𝑦
X =[𝑦] , Y =[ 2 ] , P = 0 √15 √60
𝑧 𝑦3 5
[0 0
√60 ]

Here the diagonal matrix D is in the form of I3


Rank (r) = 3
Index (s) = the number of positive terms in the canonical form = 3
Signature of quadratic form is 2s – r = 2(3) – 3 = 3
n = number of variables = 3
Since r = n and s = n, the nature of given quadratic form is positive definite.

Page | 64
UNIT-II
MEAN VALUE THEOREMS

Page | 65
I Rolle’s Theorem:
Let f(x) be a function such that
(i). It is continuous in closed interval [a, b]
(ii). It is differentiable in open interval (a, b) and
(iii). f(a) = f(b).
Then there exists at least one point ‘c’ in (a, b) such that f1(c) = 0.
Geometrical Interpretation of Rolle’s Theorem:
Let f : [ a, b] → R be a function satisfying the three conditions of Rolle ’s Theorem. Then the graph.

1. y=f(x) in a continuous curve in [a,b].


2. There exist a unique tangent line at every point x=c, where a<c<b
3. The ordinates f(a), f(b) at the end points A,B are equal so that the points A and B are equidistant from
the X-axis.
4. By Rolle’s Theorem, There is at least one point x=c between A and B on the curve at which the
tangent line is parallel to the x-axis and also it is parallel to chord of the curve.
1. Verify Rolle’s theorem for the function f(x) = sinx/ex or e-x sinx in [0,π]
Sol: i) Since sinx and ex are both continuous functions in [0, π].
Therefore, sinx/ex is also continuous in [0,π].
ii) Since sinx and ex be derivable in (0,π), then f is also derivable in (0,π).
iii) f(0) = sin0/e0 = 0 and f(π)= sin π/e π =0
 f(0) = f(π)
Thus all three conditions of Rolle ’s Theorem are satisfied.
There exists c є(0, π) such that f1(c)=0
Page | 66
e x cos x − sin x e x cos x − sin x
Now f 1 ( x) = =
(e x ) 2 ex
cos c − sin c
f1(c)= 0 => =0
ec
cos c = sin c => tan c = 1

c = π/4 є(0, π)
Hence Rolle’s theorem is verified.

 x 2 + ab 
2. Verify Rolle’s theorem for the functions log   in[a,b] , a>0, b>0,
 x( a + b ) 

 x 2 + ab 
Sol: Let f(x) = log  
 x( a + b ) 
= log(x2+ab) – log x –log(a+b)
(i). Since f(x) is a composite function of continuous functions in [a,b], it is continuous in [a,b].

1 1 x 2 − ab
(ii). f1(x) = .2 x − =
x 2 + ab x x( x 2 + ab)
f1(x) exists for all xє(a,b)

 a 2 + ab 
(iii). f(a) = log  = log1 = 0
 a + ab 
2

 b 2 + ab 
f(b) = log  = log1 = 0
 b + ab 
2

f(a) = f(b)
Thus f(x) satisfies all the three conditions of Rolle ’s Theorem.
So,  c  (a, b)  f1(c) = 0,
c 2 − ab
f1(c) = 0,  = 0  c2 = ab
c( c + ab )
2

 c = ab  (a, b)

Hence Rolle’s theorem verified.


3. Verify whether Rolle ’s Theorem can be applied to the following functions in the intervals.
i) f(x) = tan x in[0 , π] and ii) f(x) = 1/x2 in [-1,1]
(i) f(x) is discontinuous at x = π/2 as it is not defined there. Thus condition (i) of Rolle ’s Theorem is not
satisfied. Hence we cannot apply Rolle ’s Theorem here.
 Rolle’s theorem cannot be applicable to f(x) = tan x in [0,π].
(ii). f(x) = 1/x2 in [-1,1]
f(x) is discontinuous at x= 0.Hence Rolle ’s Theorem cannot be applied.

Page | 67
4. Verify Rolle’s theorem for the function f(x) = (x-a)m(x-b)n where m,n are positive integers in [a,b].
Sol: (i). Since every polynomial is continuous for all values, f(x) is also continuous in[a,b].
(ii) f(x) = (x-a)m(x-b)n
f1(x) = m(x-a)m-1(x-b)n+(x-a)m.n(x-b)n-1
= (x-a)m-1(x-b)n-1[m(x-b)+n(x-a)]
=(x-a)m-1(x-b)n-1[(m+n)x-(mb+na)]
Which exists
Thus f(x) is derivable in (a,b)
(iii) f(a) = 0 and f(b) = 0
 f(a) =f(b)
Thus three conditions of Rolle’s theorem are satisfied.
 There exists c є(a,b) such that f1(c)=0
(c-a)m-1(c-b)n-1[(m+n)c-(mb+na)]=0
 (m+n)c-(mb+na)=0
=> (m+n)c = mb+na
𝑚𝑏+𝑛𝑎
c = 𝑚+𝑛
є(a,b)
 Rolle ’s Theorem verified.

5. Using Rolle ’s Theorem, show that g(x) = 8x3-6x2-2x+1 has a zero between 0 and 1.
Sol: g(x) = 8x3-6x2-2x+1 being a polynomial, it is continuous on [0,1] and differentiable on (0,1)
Now g(0) = 1 and g(1)= 8-6-2+1 = 1
Also g(0)=g(1)
Hence, all the conditions of Rolle’s theorem are satisfied on [0,1].
Therefore, there exists a number cє(0,1) such that g1(c)=0.
Now g1(x) = 24x2-12x-2
 g1(c)= 0 => 24c2-12c-2 =0
3  21
 c= ie c= 0.63 or -0.132
12
only the value c = 0.63 lies in (0,1)
Thus there exists at least one root between 0 and 1.
6. Verify Rolle’s theorem for f(x) = x 2/3 -2x 1/3 in the interval (0, 8).
Sol: Given f(x) = x 2/3 -2x 1/3
f(x) is continuous in [0,8]
f1(x) = 2/3 . 1/x1/3 -2/3 . 1/x2/3 = 2/3(1/x1/3 – 1/x2/3)
Which exists for all x in the interval (0,8)
 f is derivable (0,8).
Now f(0) = 0 and f(8) = (8)2/3-2(8)1/3 = 4-4 =0

Page | 68
i.e., f(0) = f(8)
Thus all the three conditions of Rolle’s Theorem are satisfied.
 There exists at least one value of c in(0,8) such that f1(c)=0
1 1
ie. 1
− 2
= 0 => c = 1 є (0,8)
3 3
c c
Hence Rolle’s Theorem is verified.
7. Verify Rolle’s theorem for f(x) = x(x+3)e-x/2 in [-3,0].
Sol: - (i). Since x(x+3) being a polynomial is continuous for all values of x and e-x/2 is also continuous for all x,
their product x(x+3)e-x/2 = f(x) is also continuous for every value of x and in particular f(x) is continuous in the
[-3,0].
(ii). we have f1(x) = x(x+3)( -1/2 e-x/2)+(2x+3)e-x/2

-x/2
x 2 + 3x
=e [2x+3- ]
2
=e-x/2[6+x-x2/2]
Since f1(x) doesnot become infinite or indeterminate at any point of the interval(-3,0).
f(x) is derivable in (-3,0)
(iii) Also we have f(-3) = 0 and f(0) =0
 f (-3)=f(0)
Thus f(x) satisfies all the three conditions of Rolle’s theorem in the interval [-3,0].
Hence there exist at least one value c of x in the interval (-3, 0) such that f1(c)=0
i.e., ½ e-c/2(6+c-c2)=0 =>6+c-c2=0 (e-c/2≠0 for any c)
=> c2+c-6 = 0 => (c-3)(c+2)=0
c=3,-2
Clearly, the value c= -2 lies within the (-3,0) which verifies Rolle’s theorem.
II. Lagrange’s mean value Theorem
Let f(x) be a function such that (i) it is continuous in closed interval [a,b] & (ii) differentiable in (a,b). Then  at
least one point c in (a,b) such that
f (b) − f (a)
f1(c) =
b−a
Geometrical Interpretation of Lagrange’s Mean Value theorem:
Let f : [ a, b] → R be a function satisfying the two conditions of Lagrange’s theorem. Then the graph.

Page | 69
1. y=f(x) is continuous curve in [a,b]
2. At every point x=c, when a<c<b, on the curve y=f(x), there is unique tangent to the curve. By Lagrange’s
f (b) − f (a )
theorem there exists at least one point c  (a, b)  f (c) =
1

b−a
Geometrically there exist at least one point c on the curve between A and B such that the tangent line is

parallel to the chord AB

1. Verify Lagrange’s Mean value theorem for f(x) = x3-x2-5x+3 in [0, 4]


Sol: Let f(x) = x3-x2-5x+3 is a polynomial in x.
 It is continuous & derivable for every value of x.
In particular, f(x) is continuous [0,4] & derivable in (0,4)
Hence by Lagrange’s Mean value theorem  c (0,4) 
f (4) − f (0)
f1(c) =
4−0
f (4) − f (0)
i.e., 3c2-2c-5 = …………………….(1)
4
Now f(4) = 43-42-5.4+3 =64-16-20-3=67-36= 31 & f(0)=3
f (4) − f (0) (31 − 3)
= =7
4 4
From equation (1), we have
3c2-2c-5 =7 => 3c2-2c-12 =0

2  4 + 144 2  148 1  37
c= = =
6 6 3

1 + 37
We see that lies in open interval (0,4) & thus Lagrange’s Mean value theorem is verified.
3
2. Verify Lagrange’s Mean value theorem for f(x) = loge x in [1,e]

Sol: - f(x) = loge x

This function is continuous in closed interval [1,e] & derivable in (1,e).Hence L.M.V.T is applicable
here. By this theorem,  a point c in open interval (1,e) such that
f (e) − f (1) 1 − 0 1
f1(c) = = =
e −1 e −1 e −1
1 1 1
But f1(c) = == =
e −1 c e −1
c = e - 1
Note that (e-1) is in the interval (1, e).
Hence Lagrange’s mean value theorem is verified.
3. Give an example of a function that is continuous on [-1, 1] and for which mean value theorem does not
hold with explanations.

Page | 70
Sol:- The function f(x) = x is continuous on [-1,1]

But Lagrange Mean value theorem is not applicable for the function f(x) as its derivative does not
exist in (-1,1) at x=0.
b−a b−a
4. If a<b, P.T  Tan −1b − Tan −1 a  using Lagrange’s Mean value theorem. Deduce the
1+ b 2
1+ a2
following.
 3 4  1
i). +  Tan −1  +
4 25 3 4 6
5 + 4  +2
ii).  Tan −1 2 
20 4
Sol: consider f(x) = Tan-1 x in [a,b] for 0<a<b<1
Since f(x) is continuous in closed interval [a,b] & derivable in open interval (a,b).
We can apply Lagrange’s Mean value theorem here.
Hence there exists a point c in (a,b)
f (b) − f (a )
f1(c) =
b−a
1 1
Here f1(x) = & hence f 1 (c) =
1+ x 2
1+ c2
Thus  c, a < c < b
1 Tan −1b − Tan −1 a
= ------- (1)
1+ c2 b−a

a < c < b⟹ 𝑎2 < 𝑐 2 < 𝑏 2


⟹ 1 + 𝑎2 < 1 + 𝑐 2 < 1 + 𝑏 2
1 1 1
⟹ > >
1 + 𝑎2 1 + 𝑐 2 1 + 𝑏 2

We have 1+a2<1+c2<1+b2
1 1 1
  
1 + a 1 + c 1 + b2
2 2
……….. (2)
From (1) and (2), we have

1 Tan −1b − Tan −1 a 1


 
1+ a 2
b−a 1+ b2
or
𝑏−𝑎 𝑡𝑎𝑛−1 𝑏−𝑡𝑎𝑛−1 𝑎 𝑏−𝑎
⟹ > > ………………(3) 4>3>2 2<3<4
1+𝑎2 1 1+𝑏2

Page | 71
𝑏−𝑎 −1 −1
𝑏−𝑎
⟹ < 𝑡𝑎𝑛 𝑏 − 𝑡𝑎𝑛 𝑎 <
1 + 𝑏2 1 + 𝑎2
Hence the result

 3 4  1
+  Tan −1  +
Deductions: - 4 25 3 4 6
b−a b−a
(i)We have  Tan −1b − Tan −1 a 
1+ b 2
1+ a2
4
Take b = & a=1, we get
3

4 4 4−3
−1 −1
3 −1 4 3 4  4−3
−1
 Tan ( ) − Tan (1)  == 3  Tan −1 ( ) − 
1+
16 3 1 + 12
25 3 4 3
9 9 2
3  4  1
 +  Tan −1 ( )  +
25 4 3 4 6
5 + 4  +2
 Tan −1 2 
(ii) 20 4
b−a b−a
We have  Tan −1b − Tan −1 a 
1+ b 2
1+ a2
Taking b=2 and a=1, we get
2 −1 2 −1 1  1
 Tan −1 2 − Tan −11    Tan −1 2 − 
1+ 2 2
1+12
5 4 2
1  2+
 +  Tan −1 2 
5 4 4
4 + 5 2+
 +  Tan −1 2 
20 4
5. Show that for any x > 0, 1 + x < ex < 1 + xex.
Sol: - Let f(x) = ex defined on [0, x]. Then f(x) is continuous on [0, x] & derivable on (0, x).
By Lagrange’s Mean value theorem  a real number c є (0, x) such that
f ( x) − f (0)
= f 1 (c )
x−0
e x -e0 c e x -1 c
 =e  =e ………….(1)
x-0 x
Note that 0<c<x => e0<ec<ex ( ex is an increasing function)
ex −1
=> 1   e x From (1)
x
=> x<ex-1<xex
=> 1+x<ex<1+xex.

Page | 72
5
6. Calculate approximately 245 by using L.M.V.T.
5
Sol: - Let f(x) = x =x1/5& a=243, b=245 [a,b]=[243,245]
Then f1(x) = 1/5 x- 4/5& f1(c) = 1/5c- 4/5
By L.M.V.T, we have

f (b) − f (a)
= f 1 (c )
b−a
−4
f (245) − f (243) 1
=> = c 5
245 − 243 5
=> f (245) =f (243) +2/5c-4/5
=> c lies b/w 243 & 245 take c= 243
1 −4
2
=> 5
245 = (243) 1/5 -4/5
+2/5(243) = (3 ) + (35 ) 5
5 5

5
= 3+ (2/5) (1/81) = 3+2/405 = 3.0049

7. Find the region in which f(x) = 1-4x-x2 is increasing &the region in which it is decreasing using M.V.T.
Sol: - Given f(x) = 1-4x-x2
f(x) being a polynomial function is continuous on [a,b] & differentiable on (a,b)  a,b R
f satisfies the conditions of L.M.V.T on every interval on the real line.
f1(x) = - 4-2x= -2(2+x) xR
f1(x) = 0 if x = -2
for x<-2, f1(x) >0 & for x>-2 , f1(x)<0
Hence f(x) is strictly increasing on (-∞, -2) & strictly decreasing on (-2,∞)
8. Using Mean value theorem prove that Tan x > x in 0<x</2

 
Sol: - Consider f(x) = Tan x in  , x where 0<  <x</2

Apply L.M.V.T to f(x)

 a points c such that 0<  <c<x</2 such that

Tan x − Tan 
= sec2 c ==
x −
Tan x - Tan  = (x -  )sec2 c

Take → 0 + 0 thenTan x = x sec2 x


But sec2c>1.
Hence Tan x > x
9. If f1(x) = 0 Through out an interval [a, b], prove using M.V.T f(x) is a constant in that interval.
Sol: - Let f(x) be function defined in [a, b] & let f1(x) = 0  x in [a, b].

Page | 73
Then f1(t) is defined & continuous in [a, x] where axb.
& f(t) exist in open interval (a, x).
By L.M.V.T  a point c in open interval (a, x) 
f ( x) − f (a)
= f 1 (c )
x−a
But it is given that f1(c) = 0

 f(x) - f(a) = 0
 f(x) = f(a)  x
Hence f(x) is constant.
10 Using mean value theorem

i) x > log (1+x) > x>0

ii) π/6 + ( /15) < sin-1(0.6) < π/6 + (1/6)


iii) 1+x < ex< 1+xex x > 0

iv) < tan(-1)v - tan(-1)u < where 0 < u <v hence deduce

a) π/4+ (3/25) < tan(-1)(4/3) < π/4+ (1/6)


III. Cauchy’s Mean Value Theorem
If f: [a,b] →R, g:[a,b] →R  (i) f,g are continuous on [a,b] (ii) f,g are differentiable on (a,b)
(iii) g 1 ( x)  0x  (a, b), then

f 1 (c) f (b) − f (a)


a po int c  (a, b)  =
g 1 (c) g (b) − g (a)
1. Find c of Cauchy’s mean value theorem for
1
f ( x) = x & g ( x) =
x
in [a, b] where 0<a<b
Sol: - Clearly f, g are continuous on [a, b]  R+
1 −1
f 1 ( x) = and g 1 ( x) =
We have 2 x 2x x which exits on (a, b)
 f, g are differentiable on (a, b)  R +

Also g1 (x)0,  x (a, b)  R+


Conditions of Cauchy’s Mean value theorem are satisfied on (a, b) so c(a, b) 

f (b) − f (a ) f 1 (c)
=
g (b) − g (a) g 1 (c)
1
b− a b − a − 2c c
= 2 c == = = ab = c
1 1 −1 a− b 2 c

b a 2c c ab
Since a, b >0 , ab is their geometric mean and we have a<ab <b
Page | 74
c(a,b) which verifies Cauchy’s mean value theorem.
2. Verify Cauchy’s Mean value theorem for f(x) = ex& g(x) = e-x in [3, 7] &
Find the value of c.
Sol: We are given f(x) = ex& g(x) = e-x
f(x) & g(x) are continuous and derivable for all values of x.
=>f & g are continuous in [3, 7]

=> f & g are derivable on (3,7)


Also g1(x) = e-x0  x (3, 7)
Thus f & g satisfies the conditions of Cauchy’s mean value theorem.
Consequently,  a point c (3, 7) such that

f (7) − f (3) f 1 (c) e7 − e3 ec e7 − e3


= 1 == −7 = == = − e 2c
g (7) − g (3) g (c) e − e −3 − e − c 1 1

e7 e3
=> -e7+3 = -e2c
=> 2c = 10
=> c = 5(3, 7)
Hence C.M.T. is verified

TAYLOR’S SERIES

The series f ( x ) = f ( a ) + ( x − a ) f ( a ) + ( x − a ) f ( a ) + ...... + ( x − a ) f n ( a )


2 n

2! n!
is called Taylor series expansion of f (x ) of degree n about x = a assuming that f (x ) has successive
 
derivatives of all orders for x  a , b

MACLAURIN ‘S SERIES
2 n
The series f ( x) = f (0) + x f (0) + x f (0) + ......... + x f n (0) + .......
2! n!
is called maclaurin series expansion of f (x ) degree n about x = 0 assuming that f (x ) has successive
 
derivatives of all orders for x  0, b and the remainder after nth term in Lagrange’s form of remainder
xn n
f ( x) , (0    1)
n!
PROBLEMS

1).Obtain the Taylor’s series expansion of sin x in powers of x −


4
Sol: Given that f (x) = sin x and x − 
4
Page | 75
The Taylor`s series expansion of f (x) in powers of x – a is given by

( x − a) 2
f ( x) = f ( a ) + ( x − a ) f ( a ) + f ( a ) + .........................
2!
here f(x) = sin x and a = π/4
f ( x) = sin x f (a) = f (

) = cos

=
1
4 4 2

Now differentiate f(x) w.r.t. x successively , we get

f ( x) = cos x   1
f ( a ) = f ( ) = cos =
4 4 2

f ( x) = − sin x f ( a ) = f (



) = − sin

=−
1
4 4 2

f ( x) = − cos x f (a) = f (



) = − cos

=−
1
4 4 2

f v ( x) = sin x f v (a ) = f v (

) = sin

=
1
4 4 2

Hence the Taylor`s series expansion is

( x − a) 2 ( x − a) n n
f ( x ) = f (a ) + ( x − a ) f ( a ) + f ( a ) + ...... + f (a ) + − − −
2! n!

  
(x − )2 (x − )3 (x − )4
1  1 4 1 4 1 4 1
Sin x = + (x − ) + (− )+ (− )+ ( ) + ....
2 4 2 2! 2 3! 2 4! 2

  2  3  
 (x − ) (x − ) (x − )4 
1  4 4 4
Sin x = 1 + ( x − ) − − + + .....
2  4 2! 3! 4! 
 

2). Find the Taylors series expansion of sin2x about x = 


4

 
Sol: Write x − = t  x= + t
4 4
 
 Sin2x = Sin2( + t ) = sin ( + 2t ) = cos2t
4 2
22 t 2 24 t 4
=1- + - …………………., for all values of t
2! 4!
22  2 24 
=1 - (x − ) + ( x − ) 4 − .......... .......... ....... for all values of x
2! 4 4! 4
3). Expand 𝑒 𝑥𝑠𝑖𝑛𝑥 in powers of x
𝑥𝑠𝑖𝑛𝑥
Sol. f(x) = 𝑒

 f(0) = e0 = 1

f 1 ( x) = e x sin x ( x cos x + sin x) = f ( x)( x cos x + sin x)

f 1 (0) = f (0)(0 + 0) = 1(0) = 0

Page | 76
f 11 ( x) = f 1 ( x)( x cos x + sin x) + f ( x)(− x sin x + 2 cos x)

f 11 (0) = 0 + 1(0 + 2) = 2
f 111 ( x) = f 11 ( x)( x cos x + sin x) + f 1 ( x)(− x sin x + 2 cos x) + f 1 ( x)(− x sin x + 2 cos x) + f ( x)(− x cos x − 3 sin x)

f 111 (0) = 2(0 + 0) + 0 + 0 + 1(0 − 0) = 0

And so on.
Substituting the values of f(0) , f1(0) , etc ., in the Maclaurin’s series , we obtain
x 1 x 2 11 x 3 111
𝑒 𝑥𝑠𝑖𝑛𝑥 = f(x) = f(0) + f (0) + f (0) + f (0) + .......... .
1! 2! 3!
x2
= 1+ (2) + .......... ..........
2!
= 1 + x 2 + .......... .......

4) Show that log (1 + e x ) = log 2 +


x x2 x4
+ − + .... and hence deducethat
2 8 192
ex 1 x x3
= + − + ..........
e x + 1 2 4 48

Sol:

We know that the maclaurin`s series expansion of f (x ) is given by


x2 xn n
f ( x) = f (0) + x f (0) + f (0) + ......... + f (0) + .......
2! n!

Here f ( x) = log (1+ e x ) and f (a ) = f (0) = log 2


Now differentiate f ( x) w.r.t. x successively we get
ex e0 1
f ( x) = x f (0) = 0 =
e +1 e +1 2
(e x + 1)e x − e x .e x ex e0 1
f ( x) = = f (0) = =
(e x +1) 2 (e x +1) 2 (e 0 +1) 2 4

f ( x) = =
 
(e x + 1) 2 e x − 2e x (1 + e x )e x (1 + e x ) e x + e 2 x − 2e 2 x
=
e x − e2x
(e x +1) 4 (e x +1) 4 (e x +1) 3

e0 − e0
f (0) = 0 =0
(e +1) 3

Page | 77
(e x + 1) 3 (e x − 2e 2 x ) − (e x − e 2 x )3(1 + e x ) 2 e x
f ( x) =
iv

(e x +1) 6

=
 
(1 + e x ) e x − 2e 2 x − 3(e x − e 2 x )e x
(e x +1) 4

f (0) =
iv 
(1 + e 0 ) e 0 − 2e 0 − 3(e 0 − e 0 )e 0 2
=− =−
1
(e +1)
0 4
16 8

Substitute these values in the Maclaurin`s series

2 n
The series f ( x) = f (0) + x f (0) + x f (0) + ......... + x f n (0) + .......
2! n!

log (1 + e x ) = log 2 + x
1 x 2 1 x3 x4  1 
+ + (0) +  −  + ..........
2 2! 4 3! 4!  8 

log (1 + e x ) = log 2 +
x x2 x4 _______(1)
+ − + .......... .......
2 8 192

ex 1 x x3
Deduction: = + − + .......... :
e x + 1 2 4 48

Differentiate equation (1) w.r.t. x ,we get

ex 1 2x 4x3
= + − + .......... .......
e x + 1 2 8 192
ex 1 x x3
= + − + .......... .....
e x + 1 2 4 48

5. Obtain Taylors series expansion of ex about x = -1.


Sol: Let f(x) = ex
Put x+1 = t  x = t-1
1 t2 t3
f(x) = e = e x t -1 -1
= e .e = t
[1 + t + + + ....... ] for all values of t
e 2! 3!
1 ( x + 1) 2 ( x + 1) 3
f(x) = [1 + ( x + 1) + + + ....... ] for all values of x
e 2! 3!

note: Lagrange’s Form of remainder:


(b − a) n f ( n ) (c)
Rn =
n!
5
6).Verify Taylor`s theorem for f(x) = (1 − x) 2
with Lagrange`s form of remainder upto 2 terms in the
interval 0 ,1.
(1 − x) 2 in 0 ,1
5
Sol: Consider f(x) =
(i) f ( x) , f ( x) are continuous in 0 ,1

Page | 78
(ii) f (x) is differentiable in (0, 1)
Thus f(x) satisfies the conditions of Taylor’s theorem
Now we consider Taylor’s theorem with Lagrange’s form of remainder
x2
f ( x) = f (0) + x f (0) + f (x) with 0<  <1 → (1)
2!
Here a = 0 and x = 1 , n = p =2 ,b = 1
5
Now f (x) = (1 − x) 2
 f (0) =1
−5 −5 −5
f ( x) = (1 − x) 3 2  f (0) = (1 − 0) 3 2 =
2 2 2
1
15 15 15
f ( x) = (1 − x)1 2  f (x) = (1 − x)1 2  f 11 ( ) = (1 −  ) 2
4 4 4

And f(1) = 0
x2
From (1) , We have f ( x ) = f (0) + x f (0) + f (x)
2!
− 5 12 (15)
1
Substituting the values, we get 0 = 1 + 1( )+ (1 −  ) 2
2 2!(4)
9
 = = 0.36
25
  lies in between 0 and 1.
Thus Taylor’s theorem is verified.

7. Obtain the Maclaurin’s series expansion of coshx


Sol : Given f(x) = coshx
f1(x) =sinhx  f(0) = f11(0) = ….f2n(0) = cosh0 = 1
f11(x) =coshx

f111(x) =sinhx

 f(0) = f11(0) = ….=f2n(0) = cosh0 = 1

f1(0) = f111(0) …..=f2n+1(0) = sinh0 = 0

Hence by the Maclaurin’s series expansion of coshx is given by

x 2 11 x 3 111
coshx = f (0) + xf (0) + 1
f (0) + f (()) + ......
2! 3!

x2 x4 x6 x 2n
= 1+ + + + + ..... + + .......... ........
2! 4! 6! (2n)!

8. Find Maclaurin’s theorem with Lagrange’s form of remainder for f(x) = cosx.

Sol: Maclaurin’s theorem with Lagrange’s form of remainder is given by

x2 x n −1 xn n
f ( x) = f (0) + x f (0) + f (0) + ......... + f n −1
(0) + f (x)
2! (n − 1) ! n!

Consider f(x) = cosx

Page | 79
dn n
 f n ( x) = n
(cos x) = cos( + x)
dx 2

n
At x=0 , f n (0) = cos( )
2

Thus f(0) = cos0=1

2n
f 2n
(0) = cos = cos n = (−1) n
2
(2n + 1)
And f ( 2 n +1) (0) = cos[ ]=0
2

If n is even, coefficient of x will be (-1)n. If n is odd, coefficients of x are all zero.

Substituting these values, we have

x2 x4 x 2n x 2 n +1
cos x = f ( x) = 1 + 0 + (−1) + ......... + (1) + ...... + (−1) n + (−1) n (−1) cos(x)
2! 4! ( 2n) ! (2n + 1)!

x2 x4 x6 n x
2n
(−1) n+1 x 2 n+1
 cos x = 1 − + − + .......... . + (−1) + ...... + cos(x)
2! 4! 6! (2n)! (2n + 1)!

Worked out problems:

1. Obtain the Maclaurin’s series for the following functions

a) f(x) = (1+x) b) f(x) = loge(1+x) c) f(x) = sinhx d)f(x) = ex

2. Show that

sin −1 x x3
i) = x+4 + ...........
1 −x2 3!
1 1
ii) x =1 + ( x − 1) − ( x − 1) 2 + .......... for 0  x  2
2 8

Page | 80
TAYLOR`S SERIES FOR A FUNCTION OF TWO VARIABLES:
The series f ( x, y ) = f (a, b) + ( x − a ) f x (a, b) + ( y − b) f y (a, b)
1
+ [( x − a ) 2 f xx (a, b) + 2( x − a )( y − b) f xy (a, b) + ( y − b) 2 f yy (a, b)]
2!
1
+ [( x − a ) 3 f xxx (a, b) + 3( x − a ) 2 ( y − b) f xxy (a, b)
3!
+3( x − a )( y − b) 2 f xyy (a, b) + ( y − b) 3 f yyy (a, b)] + .................
is called the Taylor`s series expansion of f(x,y) at (a,b) in powers of (x-a) and (y-b).
NOTE: If we put a=0 and b=0 in taylor`s series expansion , we get
1
 f ( x, y ) = f (0,0) + xf x (0,0) + yf y (0,0) + [ x 2 f xx (0,0) + 2 xyf xy (0,0) + y 2 f yy (0,0)] + ...
2!
This is know as Maclaurin`s series for function of two variables.
Problem 1: Expand f(x,y) = x2y+3y-2 in powers of (x-1) and (y+2) upto the terms of third degree.
Solution: Given f(x,y) = x2y+3y-2 and (x-1) , (y+2)
Here x=a=1 and y=b=-2Now differentiate f(x,y) w.r.to x & y partially, we get
f ( x, y ) = x 2 y + 3 y − 2 ; f ( x, y ) = f (1,−2) = 12 ( −2) + 3( −2) − 2 = −10
 2
f x ( x, y ) = ( x y + 3 y − 2 ) = 2 xy ; f x ( x, y ) = f x (1,−2) = 2(1)(−2) = −4
x
 2
f y ( x, y ) = (x y + 3y − 2) = x2 + 3 ; f y ( x, y ) = f y (1,−2) = 12 + 3 = 4
y
2 
f xx ( x, y ) = 2 f ( x, y ) = ( 2 xy) = 2 y ; f xx ( x, y ) = f xx (1,−2) = 2 y = 2(−2) = −4
x x
2  2
f xy ( x, y ) = f ( x, y ) = ( x + 3) = 2 x ; f xy ( x, y ) = f xy (1,−2) = 2 x = 2(1) = 2
xy x
2  2
f yy ( x, y ) = 2 f ( x, y ) = ( x + 3) = 0 ; f yy ( x, y ) = f yy (1,−2) = 0
y y

Page | 81
3 
f xxx ( x, y ) = f ( x, y ) = (2 y ) = 0 ; f xxx ( x, y ) = f xxx (1,−2) = 0
x 3
x
3 
f xxy ( x, y ) = 2 f ( x, y ) = (2 y ) = 2 ; f xxy ( x, y ) = f xxy (1,−2) = 2
x y y
3 
f xyy ( x, y ) = f ( x, y ) = ( 0) = 0 ; f xyy ( x, y ) = f xyy (1,−2) = 0
xy 2
x
3 
f yyy ( x, y ) = f ( x, y ) = ( 0) = 0 ; f yyy ( x, y ) = f yyy (1,−2) = 0
y 3
y
1
 f ( x, y ) = −10 + ( x − 1)(−4) + ( y + 2)4 + [( x − 1) 2 (−4) + 2( x − 1)( y + 2)(2)]
2!
1
+ [3( x − 1) 2 ( y + 2)] + ..............
3!
x 2 y + 3 y − 2 = −1 − 4( x − 1) + 4( y + 2) − 2( x − 1) 2 + 2( x − 1)( y + 2) + ( x − 1) 2 ( y + 2) + ....

Worked out problems:

1. Expand (i).f(x,y)=x2+xy+y2

(ii).f(x,y)=x3+y3+xy2 in powers of (x-1)and (y-2) using taylar’s series

2. Expand exy in the neighbourhood of (1,1).

3. Expand excosy near(1,π/4) .

4. Expand f(x,y)=tan-1(y/x) in powers of (x-1)and (y-1) up to third degree terms

Page | 82
UNIT-III
Multivariable Calculus

Page | 83
We have already studied the notion of limit, continuity and differentiation in relation
with functions of a single variable. In this chapter we introduce the notion of a function of
several variables, (i.e) function of two or more variables.
In day -to-day life we deal with things which depend on two or more
quantities. For example, the area of a room which is a rectangle consists of two variables;
length(say a) and breadth (say b) is given by A = ab .Hence A has a definite value for a pair
of values a and b, Similarly , the volume of rectangular parallelepiped consists of three
variables: length a ,breadth b and height h is given by V = abh.

Partial Differentiation:
A Partial derivative of a function of several variables is its derivative with
respect to one of those variables, with the others held constant.
PROBLEMS:

 2 xy   2u  2u
1. If u = tan −1  2 2
, prove that + 2 =0
 x − y  x 2
y
 2 xy 
Sol: Given u = tan −1  2 2
x − y 

Page | 84
u 1   2 xy 
 = .  2
x 2 2
4x y x  x − y 2 
1+ 2
(x − y 2 )2
(x2 − y 2 )2 ( x 2 − y 2 )2 y − 2 xy .2 x
= 2 .
(x − y 2 )2 + 4x2 y 2 (x2 − y 2 )2
2 y( x 2 − y 2 − 2 x 2 ) − 2 y( x 2 + y 2 ) − 2y
= = = 2
(x + y )
2 2 2
(x + y )
2 2 2
(x + y2 )
 2u 2 y (2 x) 4 xy
and = 2 = 2
x 2
(x + y ) 2 2
(x + y 2 )2
similarly
u 2x  2 u − 2 y (2 x) − 4 xy
= 2 and = 2 = 2
y ( x + y 2 ) y 2
(x + y )
2 2
(x + y 2 )2
 2u  2u
 + =0
x 2 y 2

 2u  2u −1  x 
2. Verify = for the function u = tan  
xy yx  y

−1  x 
Sol: Let u = tan  
 y
u 1 1 y
=  = 2 (Here y is a constant)
x x y y + x2
1 + ( )2
y

 2 u ( y 2 + x 2 )1 − y (2 y ) x2 − y2
And = = 2 → (1)
yx ( y 2 + x2 )2 ( y + x2 )2

u 1 −x −x
Now = ( 2 ) = 2 (Here x is a constant)
y x y y + x2
1 + ( )2
y

 2u ( y 2 + x 2 )1 − x(2 x) y2 − x2 x2 − y2
= −[ ]=− 2 = → ( 2)
xy ( y 2 + x2 )2 ( y + x2 )2 ( y 2 + x2 )2

 2u  2u
From (1) & (2), we have =
xy yx

Page | 85
2 z 2 2z
3. If z = f ( x + ay) +  ( x − ay) , Prove that =a 2
y 2 x

Sol: Given z = f ( x + ay) +  ( x − ay)


z
 = f 1 ( x + ay) +  1 ( x − ay),
x
 z
2
= f 11 ( x + ay) +  11 ( x − ay) → (1)
x 2

z
= f 1 ( x + ay).a +  1 ( x − ay)(− a )
y
= a ( f 1 ( x + ay) −  1 ( x − ay)
2z
= a ( f 11 ( x + ay).a +  11 ( x − ay)a )
y 2

= a 2 ( f 11 ( x + ay) +  11 ( x − ay))
2z 2  z
2
= a (by(1))
y 2 x 2

 2u  2u  2u
4 .If r = x + y + z and u = + 2 + 2 = m(m + 1)r m − 2
2 2 2 2 m
r then Prove that
x 2
y z

Sol: We have r = x + y + z → (1)


2 2 2 2

Differentiating partially w.r.t, x, we get

r r x
2r = 2 x or = → ( 2)
x x r

r y r z
Similarly = and =
y r z r

Also we have u = rm

u u r x
 = . = m.r m −1 . = mr m − 2 .x (Using (2))
x r x r

 2u r
And = m[ x.( m − 2)r m−3 . + r m−2 .1]
x 2
x

Page | 86
m−2 x2
= mr [( m − 2). 2 + 1] (using (2))
r

mr m−2
= 2
[( m − 2).x 2 + r 2 ]
r

 2 u  2 u  2 u mr m − 2
Hence + 2 + 2 = [( m − 2) x 2 +  r 2 ]
x 2
y z r 2

mr m−2
= 2
[( m − 2)r 2 + 3r 2 ]
r

= mrm-2[(m-2) +3]

= m (m+1) rm-2

w w w
5. If w = (y-z) (z-x) (x-y) find the value of + +
x y z

Sol: Given w = (y-z) (z-x) (x-y)

w 
 =(y -z) (z-x) (x-y)] [Since y, z are constants]
x x
= (y-z) [(z-x). 1 + (x-y). (-1)]
= (y-z) [z-x - x + y]
= (y-z) (y+z-2x)
= (y-z) (y+z)-2x(y-z)
= y2-z2-2x(y-z) → (1)

By symmetry, we have
w
= z2-x2-2y(z-x) → ( 2)
y
w
And = x2-y2-2z(x-y) → (3)
z
(1) +(2) + (3) gives
Page | 87
w w w
+ + = (y2-z2)-2x(y-z) + (z2-x2)-2y(z-x) + (x2-y2)-2z(x-y)
x y  z

w w w
+ + =0
x y  z

Hence the result.


1  2U  2U  2U
6. If U = , x 2 + y 2 + z 2  0 then Prove that + + 2 =0
x2 + y2 + z2 x 2 y 2 z

−1
Sol: Given U = (x + y + z )
2 2 2 2

U − 1 2 −3

= (x + y 2 + z 2 ) 2  (x2 + y 2 + z 2 )
x 2 x
−3
−1 2
= ( x + y 2 + z 2 ) 2  (2 x) Since y, z are constants
2
−3

= − (x + y + z )
2 2 2 2
 ( x)

−3 −5
 2U −3
= − [( x 2 + y 2 + z 2 ) 2 + x( )( x 2 + y 2 + z 2 ) 2 .2 x]
x 2
2

−5
= − ( x + y + z ) [ x 2 + y 2 + z 2 − 3x 2 ]
2 2 2 2

−5
= ( x + y + z ) [2 x 2 − y 2 − z 2 ] → (1)
2 2 2 2

Similarly, we get

−5
 2U
= ( x + y + z ) [ − x 2 + 2 y 2 − z 2 ] → ( 2)
2 2 2 2

y 2

−5
 2U
= ( x 2 + y 2 + z 2 ) 2 [− x 2 − y 2 + 2 z 2 ] → (3)
z 2

(1)+ (2) + (3) gives

 2U  2U  2U
+ + 2 =0
x 2 y 2 z

Page | 88
𝜕 𝜕 𝜕 2 −9
7. If 𝑈 = log(𝑥 3 + 𝑦 3 + 𝑧 3 − 3𝑥𝑦𝑧) then show that ( + + ) 𝑈 =
𝜕𝑥 𝜕𝑦 𝜕𝑧 (𝑥+𝑦+𝑧)2

Sol. Given that 𝑈 = log(𝑥 3 + 𝑦 3 + 𝑧 3 − 3𝑥𝑦𝑧)


Differentiate U w.r.t. ‘x’ partially, we get
𝜕𝑈 3x2 −3yz
=
𝜕𝑥 𝑥 3 +𝑦 3 +𝑧 3 −3𝑥𝑦𝑧

Similarly
𝜕𝑈 3y2 −3xz
=
𝜕𝑦 𝑥 3 +𝑦 3 +𝑧 3 −3𝑥𝑦𝑧

𝜕𝑈 3z2 −3xy
And =
𝜕𝑧 𝑥 3 +𝑦 3 +𝑧 3 −3𝑥𝑦𝑧

𝜕𝑈 𝜕𝑈 𝜕𝑈 3(𝑥 2 +𝑦 2 +𝑧 2 −𝑥𝑦−𝑦𝑧−𝑧𝑥)
∴ + + =
𝜕𝑥 𝜕𝑦 𝜕𝑧 𝑥 3 +𝑦 3 +𝑧 3 −3𝑥𝑦𝑧

𝜕𝑈 𝜕𝑈 𝜕𝑈 3(𝑥 2 +𝑦 2 +𝑧 2 −𝑥𝑦−𝑦𝑧−𝑧𝑥)
+ + = (𝑥+𝑦+𝑧)(𝑥 2
𝜕𝑥 𝜕𝑦 𝜕𝑧 +𝑦 2 +𝑧 2 −𝑥𝑦−𝑦𝑧−𝑧𝑥)
𝜕𝑈 𝜕𝑈 𝜕𝑈 3
+ + = (𝑥+𝑦+𝑧)
𝜕𝑥 𝜕𝑦 𝜕𝑧
𝜕 𝜕 𝜕 2 𝜕 𝜕 𝜕 𝜕 𝜕 𝜕
Now ( + + ) 𝑈 =( + + )( + + )𝑈
𝜕𝑥 𝜕𝑦 𝜕𝑧 𝜕𝑥 𝜕𝑦 𝜕𝑧 𝜕𝑥 𝜕𝑦 𝜕𝑧

𝜕 𝜕 𝜕 𝜕𝑈 𝜕𝑈 𝜕𝑈
=( + + )( + + )
𝜕𝑥 𝜕𝑦 𝜕𝑧 𝜕𝑥 𝜕𝑦 𝜕𝑧
𝜕 𝜕 𝜕 3
=( + + )
𝜕𝑥 𝜕𝑦 𝜕𝑧 (𝑥+𝑦+𝑧)
𝜕 1 𝜕 1 𝜕 1
= 3( (𝑥+𝑦+𝑧)
+ (𝑥+𝑦+𝑧)
+ )
𝜕𝑥 𝜕𝑦 𝜕𝑧 (𝑥+𝑦+𝑧)
−1 −1 −1
= 3((𝑥+𝑦+𝑧)2 + (𝑥+𝑦+𝑧)2 + (𝑥+𝑦+𝑧)2)
−9
= (𝑥+𝑦+𝑧)2
𝜕 𝜕 𝜕 2 −9
∴( + + ) 𝑈 =
𝜕𝑥 𝜕𝑦 𝜕𝑧 (𝑥+𝑦+𝑧)2

Page | 89
HOMOGENEOUS FUNCTION:

Definitions:

1. A function f(x,y) is said to be homogeneous function of degree n if the degree of each


term in f(x,y) is n , where n is a real number.

For example: If f(x,y) = a0 x n + a1 x n−1 y + a2 x n−2 y 2 + ...... + an−1 xy n−1 + an y n → (1)

Then f(x,y) is a homogeneous function of degree n ,since the degree of each term is n.

The above definition of homogeneity applies to polynomial functions only.

2. (i) A function f(x,y) is said to be homogeneous function of degree or order n in variables


x,y if f(kx,ky) = kn f(x,y), n is a real number.

(ii) A function f(x,y) is said to be a homogeneous function of degree or order n in variables

x,y,z if f(kx,ky,kz) = kn f(x,y,z) ,n isa real number .

Further a homogeneous function of order n in x&y can be expressed as xn f(y/x) or yn f(x/y).

Examples :
1. Let f(x, y) = x2+y2.
Now f (kx, ky) = k2x2+k2y2 = k2 (x2+y2) = k2 f(x, y)
 f(x, y) is a homogeneous function of order 2 .
2. Let f(x, y) = logy +2logx.
Now f(kx, ky) = log(ky) + 2log(kx) = 3logk + f(x,y)  f(x,y)
 f(x,y) is not a homogeneous function.
3x y
3. Let f(x, y) = + log( )
y x
3kx ky 3x y
Now f (kx, ky) = + log( ) = + log( ) = k 0 f ( x, y )
ky kx y x

 f(x, y) is a homogeneous function of degree 0 .


4. Let f(x,y) = sin (xy) is not a homogeneous function since f(kx,ky) = sin (k2 xy)  Kn f(x,y).

Page | 90
EULER’S THEOREM OR HOMOGENEOUS FUNCTIONS:

z z
Statement: If z = f(x, y) is a homogeneous function of degree n, then x + y = nz,x, y
x y
in the domain of the function.
u u u
Corollary: If u is a homogeneous function of x, y & z of degree n ,then x +y +z = nu .
x y z
Examples:

x− y kx − ky
1. If f(x, y) = ,then f (kx, ky) = = k 0 f ( x, y )
x+ y kx + ky

 f (x, y) is a homogeneous function of degree 0 .


f f
By Euler’s theorem x +y = 0. f = 0
x y

 xf x + yf y = 0
2 2 2 2
 x  y x  kx   ky  kx
2. If u =   +   + , then u (kx, ky, kz) =   +   +
z z y  kz   kz  ky

 x  2  y  2 x
= k   +   +
0

 z   z  y 

= k0 u
 u is a homogeneous function of degree 0 in x , y , z.
u u u
By Euler’s theorem, x +y +z = nu = 0. (u) = 0
x y z

Deductions from Euler’s Theorem:


If z is a homogeneous function of x, y of degree n and z= f(u), then
u u f (u )
i) x +y =n 1
x y f (u )

 2u  2u 2  u
2
f (u )
ii) x 2
+ 2 xy +y = g (u )[ g 1 (u ) − 1] where g (u) = n
x 2
xy y 2
f 1 (u )

Page | 91
Problems:

u u x3 y3
1. Find x +y if u = 3
x y x + y3

x3 y3
Sol: Given u (x,y) =
x3 + y3

(k 3 x 3 )(k 3 y 3 ) k 6 x3 y3
Now u (kx , ky) = = = k 3 u ( x, y )
k x +k y
3 3 3 3
k (x + y )
3 3 3

 u is a homogeneous function of degree 3.


u u
By Euler’s theorem, x +y = 3u .
x y
−1
u
2. If u = f (x,y,z) = ( x 2 + y 2 + z 2 ) 2 Show that  x x = −u
−1 −1
−1 −1
Sol: f(kx, ky, kz) = ( k k x + k y + k z )
2 2 2 2 2 2 2
= k ( x + y + z ) = k f ( x, y , z )
2 2 2 2

 u = f(x, y, z) is a homogeneous function in x, y, z of degree -1.


u u u
By Euler’s theorem, x +y +z = (−1)u .
x y z
u
x = −u
x
3. Verify Euler’s theorem for z = ax2 + 2hxy + by2.
Sol: Given z = ax2 + 2hxy + by2 → (1)

Z is a homogeneous function in x & y of degree 2, since f (kx, ky ) = k 2 f ( x, y )


z z
 To Verify Euler’s Theorem we have to show that x + y = 2z
x y
z
Now = 2ax + 2hy → (2)
x
z
= 2hx + 2by → (3)
y

Multiplying ( 2 ) by x and ( 3 ) by y and adding we get


z z
x + y = 2ax 2 + 2hxy + 2hxy + 2by 2 = 2 (ax2 + 2hxy + by2)
x y

= 2z using (1)
Hence Euler’s theorem is verified.

Page | 92
x2 + y2 u u
4. If u = log Prove that x + y = 1
x+ y x y

x2 + y2
Sol: Given u = log → (1)
x+ y

Here we cannot check homogeneous condition, so we write (1) as

x2 + y 2
e =
u
 f (u ) = f ( x, y )
x+ y
(kx) 2 + (ky )2 x2 + y 2
f (kx, ky ) = =k
kx + ky x+ y
 f is a homogeneous function of x and y of degree 1.
u u f (u )
By Deduction from Euler’s theorem, we have x +y =n 1
x y f (u )

u u eu
x +y =1
x y d (e u )
eu
=
eu
=1

x3 + y3 u u
5. If u = tan −1 ( ) Prove that x +y = sin 2u
x+ y x y

x3 + y3
Sol: Given u = tan −1 ( ) -----→(1)
x+ y

Here we cannot check homogeneous condition, so we write (1) as


x3 + y 3
tan u =  f (u ) = f ( x, y)
x+ y

(kx)3 + (ky)3 2 x + y
3 3
f (kx, ky) = =k
kx + ky x+ y
 f(x,y) is a homogeneous function of x and y of degree 2.
u u f (u )
By Deduction from Euler’s theorem, we have x +y =n 1
x y f (u )

Page | 93
u u tan u
x +y =2
x y d (tan u )
tan u
=2 2
sec u
 sin u 
 
=2 
cos u 
 1 
 
 cos u 
2

= 2sin u cos u
= sin 2u

x  y u u
6. If u = sin −1   + tan −1   Prove that x +y =0
y   x   x y

x  y
Sol: Given u = sin −1   + tan −1  
 y x

u 1 1 1  −y 
Now =  + 2  2 
x  x   y  1+  y   x 
2

1−    
 y x

u 1 y
= − → (1)
x y2 − x2 x + y2
2

u 1  −x  1 1
Similarly =   +  
y  x   y  1+  y   x 
2 2 2

1−    
 
y x

u −x x
= + 2 → ( 2)
y y y −x
2 2 x + y 2

Multiplying ( 1 ) by x and ( 2 ) by y and adding , we get


u u x xy x xy
x +y = − − + = 0.
x y y2 − x2 x + y2
2
y2 − x2 x + y2
2

 x3 − y 3  u u
7. If u = sec−1   Prove that x +y = 2 cot u then evaluate
 x+ y  x y

 2u  2u 2  u
2
x2 + 2 xy + y
x 2 xy y 2

 x3 − y 3 
Sol: We have u = sec −1   → (1)
 x+ y 

Here we cannot check homogeneous condition, so we write (1) as


Page | 94
x3 − y 3
sec u =  f (u ) = f ( x, y )
x+ y

(kx)3 − (ky)3 2 x − y
3 3
f (kx, ky ) = =k
kx + ky x+ y
 f(x,y) is a homogeneous function of x & y of degree 2.
u u f (u )
By Deduction from Euler’s theorem, we have x +y =n 1
x y f (u )
u u sec u
x +y =2
x y d (sec u )
sec u
=2
sec u tan u
1
=2
tan u
= 2 cot u

Again By Deduction from Euler’s theorem, we have


 2u  2u 2  u
2
f (u )
x2
+ 2 xy +y = g (u )[ g 1 (u ) − 1] where g (u ) = n 1
x 2
xy y 2
f (u )

= 2cotu [-2cosec2u - 1]
=-2 cotu [2 cosec2 u + 1]

Page | 95
Exact differential:
In thermo dynamics, we come across more number of variables such as temperature (T),
Volume (V), Pressure (P), Work (W), etc. In this case any one f these can be expressed as a function
of remaining two variables. For example: T = f (P, V), T = f (P,W),Then we have the following results.
T T T T
dT = dp + dv → (1) dT = dp + dw → (2)
p v p w

T
In (1) ,the value of where T is a function of P& V is obtained treating V as a constant and
p
T T
we can write ( ) v .In the same way we can have ( ) w in (2) . This dT is Exact Differential of T.
p p
 
In general, if  =  ( x, y ) , d = dx + dy is called Exact Differential of  .
x y

The Chain Rule of Partial Differentiation:

1. If z = f(x, y) where x =  (t), y =  (t ) then z is called composite function of a variable t.

2. If z = f(x,y) where x =  (u,v) ,y =  (u , v) then z is called a composite function of two variables u


&v

Theorem: Let z = f (u, v) where u =  (x, y) and v = g(x, y) .Then

z z u z v z z u z v
= + and = +
x u x v x y u y v y

These are referred to as the chain rule of partial differentiation .The above rule can be
extended to functions of more than two independent variables.

Total Differential Coefficient:

Let z = f(x, y) where x =  (t) and y = g(t)

Substituting x and y in z = f(x, y), z becomes a function of a single variable t.

dz
Then is called the total differential coefficient or total derivative of z.
dt

dz z dx z dy
 = +
dt x dt y dt

Page | 96
𝝏𝒖 𝝏𝒖 𝝏𝒖
1. If 𝒖 = 𝒖(𝒚 − 𝒛, 𝒛 − 𝒙, 𝒙 − 𝒚) then prove that + + =𝟎
𝝏𝒙 𝝏𝒚 𝝏𝒛
Sol: Given 𝑢 = 𝑢(𝑦 − 𝑧, 𝑧 − 𝑥, 𝑥 − 𝑦)
Let 𝑋 = 𝑦 − 𝑧, 𝑌 = 𝑧 − 𝑥, 𝑍 = 𝑥 − 𝑦 then 𝑢 = 𝑢(𝑋, 𝑌, 𝑍) where X, Y, Z are
functions of x, y, z.
Then
𝜕𝑢 𝜕𝑢 𝜕𝑋 𝜕𝑢 𝜕𝑌 𝜕𝑢 𝜕𝑍
= + +
𝜕𝑥 𝜕𝑋 𝜕𝑥 𝜕𝑌 𝜕𝑥 𝜕𝑍 𝜕𝑥
𝜕𝑢 𝜕𝑢 𝜕𝑢
= .0 + . (−1) + .1
𝜕𝑋 𝜕𝑌 𝜕𝑍

𝜕𝑢 𝜕𝑢
=− + --------------- (i)
𝜕𝑌 𝜕𝑍

𝜕𝑢 𝜕𝑢 𝜕𝑋 𝜕𝑢 𝜕𝑌 𝜕𝑢 𝜕𝑍
= + +
𝜕𝑦 𝜕𝑋 𝜕𝑦 𝜕𝑌 𝜕𝑦 𝜕𝑍 𝜕𝑦
𝜕𝑢 𝜕𝑢 𝜕𝑢
= .1 + .0 + . (−1)
𝜕𝑋 𝜕𝑌 𝜕𝑍
𝜕𝑢 𝜕𝑢
= − --------------- (ii)
𝜕𝑋 𝜕𝑍

𝜕𝑢 𝜕𝑢 𝜕𝑋 𝜕𝑢 𝜕𝑌 𝜕𝑢 𝜕𝑍
And = + +
𝜕𝑧 𝜕𝑋 𝜕𝑧 𝜕𝑌 𝜕𝑧 𝜕𝑍 𝜕𝑧
𝜕𝑢 𝜕𝑢 𝜕𝑢
= . (−1) + .1 + .0
𝜕𝑋 𝜕𝑌 𝜕𝑍
𝜕𝑢 𝜕𝑢
=− + --------------- (iii)
𝜕𝑋 𝜕𝑌

𝜕𝑢 𝜕𝑢 𝜕𝑢
By adding (i), (ii) and (iii) we get + + =0
𝜕𝑥 𝜕𝑦 𝜕𝑧

du
2. If u = x logxy where x3+y3+3xy = 1 .Find
dx

Sol: We have u = x logxy = x(logx+logy) = xlogx+xlogy

u 1
 = x. + log x(1) + log y = 1 + log x + log y
x x

= 1+log xy → (1)

u 1 x
And = x. = → (2)
y y y
Also given x 3 + y 3 + 3xy = 1
dy dy
Differentiating , 3x 2 + 3 y 2 + 3( x + y) = 0
dx dx

Page | 97
dy − ( x 2 + y )
 = → (3)
dx ( y 2 + x)
du u u dy
Now = +
dx x y dx
x  − ( x 2 + y) 
= 1 + log xy +   from (1), (2), (3)
y  y2 + x 
x( x 2 + y
= 1 + log xy −
y ( y 2 + x)

JACOBIAN:

Let u=u(x,y), v=v(x,y) be any two functions of two variables. Then

u u
x y ux uy
The determinant or is called the Jacobian of u and v w.r.to x and y and
v v vx vy
x y
 (u , v)  u, v 
the determinant value is denoted by or J   or J ( u , v )
 ( x, y )  x, y 

If u=u(x,y,z), v=v(x,y,z), w=w(x,y,z) be any three functions of three variables, then the
u u u
x y z ux uy uz
 (u , v, w) v v v
Jacobian of u, v, w, w.r.to x, y, z is given by = or v x vy vz
 ( x, y, z ) x y z
w w w wx wy wz
x y z

PROPERTIES OF JACOBIAN:

 (u, v)  ( x, y )
1. If J = and JI = then JJI = 1
 ( x, y )  (u, v)

2. If u, v are functions of r, s and r, s are functions of x, y then


(u, v) (u, v)  (r , s)
= .
 ( x, y )  ( r , s )  ( x, y )
U,v ---→ r,s

X,y

Page | 98
FUNCTIONAL DEPENDENCE:

Let u=u(x,y) & v=v(x,y) are functions of x & y.

1. u, v are said to FUNCTIONALLY DEPENDENT (F.D) if, the JACOBIAN is ZERO and
there exist a relation between them.

 u, v 
i.e., J   = 0 then u & v are said to be functionally dependent
 x, y 

2. u, v are said to FUNCTIONALLY INDEPENDENT (F.I) if, the JACOBIAN is NOT


ZERO.

 u, v 
i.e., J    0 then u & v are said to be functionally Independent
 x, y 

PROBLEMS:
𝑦𝑧 𝑧𝑥 𝑥𝑦 𝜕(𝑢,𝑣,𝑤)
1. If 𝑢 = ,𝑣 = ,𝑤 = 𝑡ℎ𝑒𝑛 𝑃𝑟𝑜𝑣𝑒 𝑡ℎ𝑎𝑡 =4
𝑥 𝑦 𝑧 𝜕(𝑥,𝑦,𝑧)
𝑦𝑧 −𝑦𝑧 𝑧 𝑦
Sol: 𝑢= ⟹ 𝑢𝑥 = ; 𝑢𝑦 = ; 𝑢𝑧 =
𝑥 𝑥2 𝑥 𝑥
𝑧𝑥 𝑧 −𝑧𝑥 𝑥
𝑣= ⟹ 𝑣𝑥 = ; 𝑣𝑦 = ; 𝑣𝑧 =
𝑦 𝑦 𝑦2 𝑦
𝑥𝑦 𝑦 𝑥 −𝑥𝑦
𝑤= ⟹ 𝑤𝑥 = ; 𝑤𝑦 = ; 𝑣𝑧 =
𝑧 𝑧 𝑧 𝑧2
𝑢𝑥 𝑢𝑦 𝑢𝑧
𝜕(𝑢,𝑣,𝑤)
Now = | 𝑣𝑥 𝑣𝑦 𝑣𝑧 |
𝜕(𝑥,𝑦,𝑧)
𝑤𝑥 𝑤𝑦 𝑤𝑧
−𝑦𝑧 𝑧 𝑦
𝑥2 𝑥 𝑥
𝑧 −𝑧𝑥 𝑥
= || 𝑦 𝑦2 𝑦 |
|
𝑦 𝑥 −𝑥𝑦
𝑧 𝑧 𝑧2
−𝑦𝑧 𝑧𝑥 2 𝑦 𝑥2 𝑧 −𝑥𝑦𝑧 𝑥𝑦 𝑦 𝑥𝑧 𝑥𝑦𝑧
= [ − ]− [ − ]+ [ + ]
𝑥2 𝑦2𝑧 2 𝑦𝑧 𝑥 𝑧 2𝑦 𝑦𝑧 𝑥 𝑦𝑧 𝑦2𝑧
𝑧 𝑥 𝑥 𝑦 𝑥 𝑥
= [ + ]+ [ + ]
𝑥 𝑧 𝑧 𝑥 𝑦 𝑦
=2+2=4

Page | 99
 ( x, y )  ( r , )  ( x , y )  ( r , )
2. If x = r cos , y = r sin , find and . Also showthat . =1
 (r , )  ( x, y )  ( r , )  ( x , y )
Sol: Given that
 ( x, y )
I J= :
 (r , )
x = r cos .........(1)
y = r sin  .........(2)
Now differentiate eqn (1) & (2) w.r.to r and  partially we get
x x y y
= cos ; = −r sin  ; = sin  ; = r cos
r  r 
x x
 ( x, y ) r  cos  −r sin 
J = = = = r (cos 2  + sin 2  ) = r
 (r ,  ) y y sin  r cos 
r 
r r
 (r , ) x y
Now we have to find =
 ( x, y )  
x y
From eqn (1) & (2)
x 2 = r 2 cos 2 
y 2 = r 2 sin 2 
x2 + y2 = r 2 ......(3)
From these y sin  y
= = tan    = tan −1   ......(4)
x cos x
Now differentiate eqn (3) and (4) w.r.to x & y partially we get

r x r y  −y  x
= ; = ; = 2 2 ; = 2 2
x r y r x x + y y x + y
r r x y
 (r , ) x y x2 y2 1
J = = = −ry r = + =
 ( x, y )   x r(x + y ) r(x + y ) r
2 2 2 2

x y x2+ y2 x +y
2 2

 ( x , y )  ( r , ) 1
 JJ  = . = r. = 1
 ( r , )  ( x , y ) r

Page | 100
3. If x = r sin  cos  , y = r sin  sin  , z = r cos  , show that
 ( x, y , z ) (r ,  ,  )
= r 2 sin  and find
 (r ,  ,  ) ( x, y, z )

Sol : Given
x = r sin  cos ........(1)
y = r sin  sin  ........( 2)
z = r cos ........( 3)

Now differentiate eqn (1), (2) and (3) w.r.to r,  , partially we get

x x x
= sin cos ; = r cos cos ; = −r sin sin
r  
y y y
= sin sin ; = r cos sin ; = r sin cos
r  

z z z
= cos ; = −r sin ; =0
r  

x x x
r  
sin  cos  r cos  cos  − r sin  sin 
 ( x, y, z ) y y y
J= = = sin  sin  r cos  sin  r sin  cos 
 (r ,  ,  ) r  
cos  − r sin  0
z z z
r  
= sin  cos  (0 + r 2 sin 2  cos  ) − r cos  cos  (0 − r sin  cos  cos  )
− r sin  sin  ( − r sin 2  sin  − r cos 2  sin  )
= r 2 sin 3  cos 2  + r 2 sin  cos 2  cos 2  + r 2 sin 3  sin 2  + r 2 sin  cos 2  sin 2 
= r 2 sin 3   cos 2  + sin 2   + r 2 sin  cos 2  cos 2  + sin 2  
= r 2 sin 3  + r 2 sin  cos 2 
= r 2 sin   sin 2  + cos 2  
= r 2 sin 

 ( x, y , z )
 J= = r 2 sin 
 (r , ,  )

 ( x, y , z )  ( r ,  ,  ) (r , ,  ) 1
we know that JJ  = 1  . =1  = 2
 ( r ,  ,  )  ( x, y , z )  ( x, y , z ) r sin 

 ( x, y , z )
3. If x +y + z = u , y + z = uv, z = uvw , then evaluate
 (u , v , w)

Sol: Given u = x +y + z → (1)


Page | 101
uv =y + z → (2)

uvw = z → (3)
From eq (2), uv = y + z  y = uv - z
 y =uv-uvw (from (3) )
From eq (1), u = x +y + z
 x = u - (y+z)

 x = u – uv (using (2))
x  x x
 = (u − uv) = 1 − v , = −u , =0
u u v w

y  ( uv − uvw ) y y
= = v − vw , = u − uw , = −uv
u u v w
z z z
= vw , = uw , = uv
u v w
x x x
u v w 1− v −u 0
 ( x, y , z ) y y y
 = = v − vw u − uw − uv
 (u , v , w) u v w
z z z vw uw uv
u v w

= (1-v)[(u-uw) (uv)+uv(uw) ]+u [ (v-vw(uv)+(uv)(vw) ]


 ( x, y , z )
= u2v.
 (u , v , w)

 (u , v)
4. If u = x 2 − y 2 , v = 2 xy where x = r cos , y = r sin  , show that = 4r 3
 (r ,  )

Sol: Given u = x 2 − y 2 , v = 2 xy , x = r cos , y = r sin 

 (u , v)  (u , v)  ( x, y )
= .
 ( r ,  )  ( x, y )  ( r ,  )
u u x x
We know that
x y r 
= .
v v y y
x y r 

Page | 102
2x −2 y cos  − r sin 
= .
2 y 2 x sin  r cos 
( )
= 4 x 2 + y 2 .r (1)
= 4(r 2 )r
= 4r 3

5. Show that the functions u = xy + yz + zx , v = x2 +y2 +z2 and w = x + y+ z are

Functionally related. Find the relation between them.

Sol: Given u = xy + yz + zx, v = x2 +y2 +z2 and w = x + y+ z ……………….(1)

u = xy + yz + zx , v = x2 +y2 +z2 and w = x + y+ z

Now differentiate eqn (1) w.r.to x, y & z partially we get

u u u
u = xy + yz + zx  = y+z ; = z+x ; = x+ y
x y z
v v v
v = x2 + y2 + z 2  = 2x ; = 2 y ; = 2z
x y z
w w w
w= x+ y+z  =1 ; =1 ; =1
x y z

u u u
x y z y+z x+z y+x
 (u , v, w) v v v
J = = = 2x 2y 2z
 ( x, y , z ) x y z
w w w 1 1 1
x y z
= ( y + z )(2 y − 2 z ) − ( x + z )(2 x − 2 z ) + ( y + x)(2 x − 2 y )
. = 2( y + z )( y − z ) − 2( x + z )( x − z ) + 2( x + y )( x − y )
= 2 y 2 − 2z 2 − 2x 2 + 2z 2 + 2x 2 − 2 y 2
 (u , v, w)
J= =0
 ( x, y , z )

Hence u, v and w are functionally dependent. Hence there exist a relation between u,
v and w.

Relation:

Now w2 = (x+ y+ z)2 = x2 + y2 + z2 + 2( xy + yz + zx)

W2= v + 2u is the functional relation between u, v & w

x+ y  (u, v)
6. If u = , v = tan −1 x + tan −1 y .Find . Hence Prove that u & v are
1 − xy  ( x, y )

Page | 103
Functionally dependent. Find the functional relation between them.

x+ y
Sol: Given u = , v = tan −1 x + tan −1 y
1 − xy
u 1 + y 2 u 1+ x2
 = , =
x (1 − xy) 2 y (1 − xy) 2

v 1 v 1
= , =
x 1 + x 2 y 1 + y 2

u u 1+ y2 1+ x2
 (u, v) x y (1 − xy) 2 (1 − xy) 2
Now = =
 ( x, y ) v v 1 1
x y 1+ x2 1+ y2
1 1
= − = 0.
(1 − xy) (1 − xy) 2
2

Hence u & v are functionally dependent.

Relation:
Now v = tan −1 x + tan −1 y
−1 x+ y
= tan ( )
1 − xy
−1
v = tan u
−1
v = tan u is the functional relation between them.

** Maximum & Minimum for function of a single Variable:


To find the Maxima & Minima of f(x) we use the following procedure.
(i) Find f1(x) and equate it to zero
(ii) Solve the above equation we get x0,x1as roots.
(iii) Then find f11(x).
If f11(x)(x = x0)> 0, then f(x) is minimum at x0
If f11(x)(x = x0) , < 0, f(x) is maximum at x0 . Similarly we do this for other stationary points.
1. Find the max & min of the function f(x) = x5 -3x4 + 5 (’08 S-1)
Sol: Given f(x) = x5 -3x4 + 5
f1(x) = 5x4 – 12x3
for maxima or minima f1(x) =0
5x4 – 12x3 = 0
x =0, x= 12/5
f11(x) = 20 x3 – 36 x2
At x = 0 => f11(x) = 0. So f is neither maximum nor minimum at x = 0
Page | 104
At x = (12/5) => f11(x) =20 (12/5)3 – 36(12/5)
=144(48-36) /25 =1728/25 > 0
So f(x) is minimum at x = 12/5
The minimum value is f (12/5) = (12/5)5 -3(12/5)4 + 5

**Maxima & Minima for functions of two Variables:


Working procedure:
1. Find and Equate each to zero. Solve these equations for x & y we get the pair of

values (a1, b1) (a2,b2) (a3 ,b3) ………………

Find l =  2f ,m =  f , n =  f2
2 2 2
2.
x  x y y

3. i. If l n –m2> 0 and l < 0 at (a1,b1) then f(x ,y) is maximum at (a1,b1) and
maximum value is f(a1,b1)

ii. If l n –m2> 0 and l > 0 at (a1,b1) then f(x ,y) is minimum at (a1,b1) and minimum
value is f(a1,b1) .
iii. If l n –m2< 0 and at (a1, b1) then f(x, y) is neither maximum nor minimum at (a1, b1).
In this case (a1, b1) is saddle point.
iv. If l n –m2 = 0 and at (a1, b1), no conclusion can be drawn about maximum or
minimum and needs further investigation. Similarly we do this for other stationary
points.
*Extremum: A function which has a maximum or minimum or both is called ‘Extremum’
*Extreme value- The maximum value or minimum value or both of a function is Extreme
value.

*Stationary points: - To get stationary points we solve the equations = 0 and

= 0 i.e the pairs (a1, b1), (a2, b2) ………….. are called Stationary.

Page | 105
PROBLEMS:
1. Locate the stationary points & examine their nature of the following function.
u =x4 + y4 -2x2 +4xy -2y2, (x > 0, y > 0)
Sol: Given u(x, y) = x4 + y4 -2x2 +4xy -2y2
u u
For maxima & minima = 0, =0
x y

P=0: = 4x3 -4x + 4y = 0  x3 – x + y = 0 -------------------> (1)

q=0: = 4y3 +4x - 4y = 0  y3 + x – y = 0 -------------------> (2)

 2u
r= = 12x2 – 4
x 2

 2u
s= = ( ) =4
xy

 2u
t= = 12y2 – 4
y 2

Adding (1) & (2),


x3 + y3 = 0
 (𝑥 + 𝑦)(𝑥 2 − 𝑥𝑦 + 𝑦 2 ) = 0

 (𝑥 + 𝑦) = 0 or (𝑥 2 − 𝑥𝑦 + 𝑦 2 ) = 0

 x = – y -------------------> (3)

Substitute x= -y in eqn(1) or in eqn(2)


(1) x3 – x + y = 0  x3 – 2x =0  x(x2 -2)0  x= 0, 2, − 2

Hence (3)  y = 0, - 2, 2

The stationary points are (0,0), ( , ),(√2, −√2)

rt – s2 = (12x2 – 4 )( 12y2 – 4 ) -16

At ( , ), rt – s2 = (24 – 4) (24 -4) -16 = (20) (20) – 16 > 0 and r=20>0

Here rt – s2>0, r>0 ➔( , ) is minimum point


Similarly (√2, −√2) is also a minimum point
At (0, 0), rt – s2 = (0– 4)(0 -4) -16 = 0
Hence f(x,y) fails to have maximum or minimum at (0,0)

Page | 106
2. Investigate the maxima & minima, if any, of the function f(x,y) = x3y2 (1-x-y).
Sol: Given f(x) = x3y2 (1-x-y) = x3y2- x4y2 – x3y3

Step –I : p= = 3x2y2 – 4x3y2 -3x2y3

q= = 2x3y – 2x4y -3x3y2

2 f
r= =6xy2 – 12x2y2 -6xy3
x 2

2 f   f 
s= =   =6x2y -8 x3y – 9x2y2
xy x  y 

2 f
t= = 2x3 -2x4 -6x3y
y 2

Step –II: p= =0

 3x2y2 – 4x3y2 -3x2y3 = 0 => x2y2(3 – 4x -3y) = 0


 x=0 ------> (1); y=0 ------> (2); 4x+3y=3 ------> (3)
q= =0

 2x3y – 2x4y -3x3y2 = 0 => x3y(2 – 2x -3y) = 0


 x=0------> (4) ; y=0 ------> (5); 2x+3y=2 ------> (6)
From (1) & (4): x = 0 only
From (1) & (5): (x,y) = (0,0)
From (1) & (6): x = 0, y =2/3 (x,y)=( 0,2/3 )
From (2) & (4): (x,y) = (0,0)
From (2) & (5): y = 0 only
From (2) & (6): (x,y) = (1,0)
From (3) & (4): (x,y) = (0,1)
From (3) & (5): (x,y) = ( 3/4,0 )
From (3) & (6): (x,y)
4x + 3y = 3
2x + 3y = 1
Solving this we get 2x = 1 => x = ½
4 (½) + 3y – 3 = 0 => 3y = 3 -2, y = (1/3)
(x,y)= (1/2 ,1/3)
Here the stationary points are (0,0) ; ( 0,2/3 ) ; (1,0) ; (0,1) ; (3/4,0) ; (1/2, 1/3)

Page | 107
At (0,0) ; ( 0,2/3 ) ; (1,0) ; (0,1) ; (3/4,0), rt- s2 =0
At (1/2,1/3) : rt-s2 = (6xy2 – 12x2y2 -6xy3)( 2x3 -2x4 -6x3y) –(6x2y -8 x3y – 9x2y2)2
= (-1/9) (-1/8) – (-1/12)2 >0
Here rt-s2 > 0 and r < 0 ==> (1/2, 1/3) is maximum point
The function has a maximum value at (1/2, 1/3)
 1 1   1 1  1 1 1 1 1 1
Maximum value is f  ,  =   1 − −  =  − =
 2 3   8 9  2 3 72  2 3  432

3. Find three positive numbers whose sum is 100 and whose product is maximum.
Sol: Let x, y, z be three +ve numbers.
Then x + y + z = 100
 z = 100 – x – y
Let f(x,y) = xyz =xy(100 – x – y) =100xy –x2y-xy2

For maxima or minima = 0 and =0

=0: =100y –2xy-y2 = 0 => y(100- 2x –y) = 0 ----------------> (1)

= 100x –x2 -2xy = 0 => x(100 –x -2y) = 0 ------------------> (2)

100 -2x –y = 0
200 -2x -4y =0
-----------------------------
-100 + 3y = 0 => 3y =100 => y =100/3

100 – x –(200/3) = 0 => x = 100/3


 f
2
r= = - 2y
x 2
 2 f 
 2  (100/3, 100/3) = - 200/3
 x 

2 f   f 
s= =   = 100 -2x -2y
xy x  y 
 2 f 
  (100/3, 100/3) = 100 – (200/3) –(200/3) = -(100/3)
 xy 

2 f
t= 2 = -2x
y
 2 f 
 2  (100/3, 100/3) = - 200/3
 y 
rt -s2 = (-200/3) (-200/3) - (-100/3)2 = (100)2 /3
The function has a maximum value at (100/3, 100/3)
Page | 108
100 100 100
i.e. at x = 100/3, y = 100/3 z = 100 − − =
3 3 3
The required numbers are x = 100/3, y = 100/3, z = 100/3
4. Find the maxima & minima of the function f(x) = 2(x2 –y2) –x4 +y4
Sol: Given f(x) = 2(x2 –y2) –x4 +y4 = 2x2 –2y2 –x4 +y4

For maxima & minima = 0 and =0

P=0: = 4x - 4x3 = 0 => 4x(1-x2) = 0 => x = 0 , x = ± 1

q=0: = -4y + 4y3 = 0 => -4y (1-y2) = 0 =>y = 0, y = ± 1

 2 f 
r =  2  = 4-12x2
 x 

 2 f 
s =   =   f 
 
=0
 xy  x  y 

 2 f 
t =   = -4 +12y2
 y
2

we have rt – s2 = (4-12x2)( -4 +12y2 ) – 0


= -16 +48x2 +48y2 -144x2y2
= 48x2 +48y2 -144x2y2 -16
i) At ( 0 , ± 1 )
rt – s2 = 0 + 48 - 0 -16 =32 > 0
r= 4-0 = 4 > 0
f has minimum value at ( 0 , ± 1 )
f (x ,y ) = 2(x2 –y2) –x4 +y4
f ( 0 , ± 1 ) = 0 – 2 – 0 + 1 = -1
The minimum value is ‘-1 ‘.
ii) At ( ± 1 ,0 )
rt – s2 = 48 + 0 - 0 -16 =32 > 0
r = 4-12 = - 8 < 0
f has maximum value at ( ± 1 ,0 )
f (x ,y ) = 2(x2 –y2) –x4 +y4
f ( ± 1 , 0 ) =2 -0 -1 + 0 = 1
The maximum value is ‘1 ‘.

Page | 109
iii) At (0,0) , (± 1 , ± 1)
rt – s2 < 0
(0 , 0) & (± 1 , ± 1) are saddle points.
f has no max & min values at (0 , 0) , (± 1 , ± 1).

5. A rectangular box open at the top is to have volume of 32 cubic feets. Find the
Dimensions of the box requiring least material for its construction
Sol: let ‘x’ feet ’y’ feet, ’z’ feet are dimensions of the box(i.e, length, breadth, height)
The surface area of the box is
s=2xy+2yz+zx ……………………. (1)
we can take surface area as s= xy+2yz+2zx or s= 2xy+yz+2xz
The volume of the box is xyz=32 …………………….. (2)

32
From (2), z=
xy
‘z’ value is substitute in eq(1)

32 32
S= 2 xy + 2 y ( ) + ( )x
xy xy

64 32
Let f(x,y) = 2 xy + + …………………………………… (3)
x y
Now differentiate equation (3) w.r.to x and y we get
f 64
= p = 2y − 2
x x
f 32
= q = 2x − 2
y y

for max imum and min imum , we are taking


64
p=0  2y − = 0.................................(4)
x2
32
q = 0  2 x − 2 = 0..................................(5)
y
64 32
from(4)  2 y = 2  y = 2
x x
32 32 32.32
from (5)  2 x = 2  2 xy 2 = 32  2 x( 2 ) 2 = 32  2 x. 4 = 32
y x x
32
2. 3 = 1  x 3 = 64  x = 4
x

Page | 110
substitute x in eqn (5)
32 16 16 16
(5)  2 x = 2
 x = 2  4 = 2  y2 =  y = 2
y y y 4
substitute x & y in z we get xyz = 32
32 32 32
z= z =  z= z=4
xy 4.2 8

x=4 feet, y=2 feet, z=4feet are the dimensions of the box required for its construction.

6. Find the extreme value of f(x,y)=sinx+siny+sin(x+y)

Sol: Given that f(x,y)=sinx+siny+sin(x+y)


f
p= = cos x + cos( x + y )
x
f
q= = cos y + cos( x + y )
y

2 f  f 
r= = ( ) = [cos x + cos( x + y )] = − sin x − sin( x + y )
x 2
x x x
 f
2
 f 
s= = ( ) = [cos y + cos( x + y )] = − sin( x + y )
xf x y x
2 f  f 
t = 2 = ( ) = (cos y + cos( x + y )) = − sin y − sin( x + y )
y y y y

f f
=0 =0
x x
cos x + cos( x + y ) = 0 ....................(1)

cosy+cos(x+y)=0 ................................(2)
(1)-(2): cosx=cosy  x=y
Substitute x=y in eqn (1), we get
𝐶+𝐷 𝐶−𝐷
Cosx + Cos2x=0 cos 𝐶 + cos 𝐷 = 2 cos ( ) cos ( )
2 2

3x x 3x x
2 cos cos = 0  cos =0 (or ) cos = 0
2 2 2 2
3x x
 = cos−1 (0) = cos−1 (0)
2 2

Page | 111
3x  x 
 = =
2 2 2 2
 3 x =  x = 

x= x = − ,+
3
 the stationary po int s are
   
(− , − )( ,  )(− , − ), ( , )
3 3 3 3

Case1: At (-π,-π)
r=0 ; s=0 ; t=0
rt-s2=0
No conclusion
case2: At (π,π)
r=0, s=0,t=0
rt-s2=0
No conclusion
 
case3: At ( − ,− )
3 3
3 3 3
r = − sin x − sin( x + y ) = ++ = 3 ,s = ,t = 3
2 2 2
3 3
rt − s 2 = 3. 3 − ( ) 2 = 3 −
2 4
9
rt − s 2 =  0 and r = 3  0
4

Page | 112
7.

 
(− , − ) is a po int min imum
3 3
  3 3
The min imum value is = f (− ,− ) = −
3 3 2
 
case4 : At ( , )
3 3
3
r = − 3, s = − ,t = − 3
2
9
rt − s 2 =  0 and r = − 3  0
4
 
( , ) is a po int of max imum
3 3
     
 The max imum value is = f ( , ) = sin + sin + sin( + )
3 3 3 3 3 3
  3 3 3 3 3
f ( , )= + + =
3 3 2 2 2 2
7. Find the shortest distance from origin to the surface 𝑥𝑦𝑧 2 = 2.
Sol: Given surface 𝑥𝑦𝑧 2 = 2
Let P(x,y,z) be any point on the surface

Now 𝑂𝑃 = √𝑥 2 + 𝑦 2 + 𝑧 2
𝑂𝑃2 = 𝑥 2 + 𝑦 2 + 𝑧 2
2
= 𝑥2 + 𝑦2 +
𝑥𝑦

2
Let 𝑓(𝑥, 𝑦) = 𝑥 2 + 𝑦 2 +
𝑥𝑦

2 −1
𝑝 = 2𝑥 + ( 2 )
𝑦 𝑥
2 −1
𝑞 = 2𝑦 + ( 2 )
𝑥 𝑦

Page | 113
2 −2
𝑟 = 2 − ( 3)
𝑦 𝑥
2
𝑠=
𝑥 2𝑦2
2 −2
𝑡 = 2 − ( 3)
𝑥 𝑦
2 −1
𝑝 = 0 ⟹ 2𝑥 + ( 2 ) = 0 ⟹ 𝑥 3 𝑦 − 1 = 0 -----→(1)
𝑦 𝑥

2 −1
𝑞 = 0 ⟹ 2𝑦 + ( 2 ) = 0 ⟹ 𝑥𝑦 3 − 1 = 0 -----→(2)
𝑥 𝑦

Solving eqn(1) and (2) 𝑥 3 𝑦 − 𝑥𝑦 3 = 0 we have 𝑥 2 = 𝑦 2 ⟹ ±𝑥 = ±𝑦


Put x=y in eqn(2): 𝑥 4 = 1 ⟹ 𝑥 = 1 ⟹ 𝑦 = 1 ⟹ (𝑥, 𝑦) = (1,1)
Put -x=-y in eqn(2): 𝑥 4 = 1 ⟹ 𝑥 = 1 ⟹ 𝑦 = 1 ⟹ (𝑥, 𝑦) = (1,1)
∴ (𝑥, 𝑦) = (1,1)𝑖𝑠 𝑡ℎ𝑒 𝑠𝑡𝑎𝑡𝑖𝑜𝑛𝑎𝑟𝑦 𝑝𝑜𝑖𝑛𝑡

2 −2 2 −2 2 2
Now 𝑟𝑡 − 𝑠 2 = (2 − ( 3 )) (2 − ( 3 )) − ( 2 2)
𝑦 𝑥 𝑥 𝑦 𝑥 𝑦

At (1,1): 𝑟𝑡 − 𝑠 2 = (6)(6) − 4 = 32 > 0


Here 𝑟𝑡 − 𝑠 2 > 0 𝑎𝑛𝑑 𝑟 > 0 ⟹ (1,1)𝑖𝑠 𝑚𝑖𝑛𝑖𝑚𝑢𝑚 𝑝𝑜𝑖𝑛𝑡
2
Now 𝑧 2 = ⟹ 𝑧 = ±√2
𝑥𝑦

∴ (𝑥, 𝑦, 𝑧) = (1,1, ±√2) 𝑎𝑟𝑒 𝑡ℎ𝑒 𝑝𝑜𝑖𝑛𝑡𝑠 𝑜𝑛 𝑡ℎ𝑒 𝑠𝑢𝑟𝑓𝑎𝑐𝑒 𝑡ℎ𝑎𝑡 𝑎𝑟𝑒 𝑛𝑒𝑎𝑟𝑒𝑠𝑡 𝑡𝑜 𝑜𝑟𝑖𝑔𝑖𝑛

Shortest distance : 𝑂𝑃 = √𝑥 2 + 𝑦 2 + 𝑧 2 = 2

Page | 114
LEGRANGE’S METHOD OF UNDETERMINED MULTIPLIERS
Working rule:
Suppose it is required to find the extrema for the function f(x,y,z) subject to the
condition
 ( x, y, z ) = 0. .............(1)
STEP1: From Lagrangean function F ( x, y , z ) = f ( x, y , z ) +  ( x, y , z ) where  is
called the lagrangean multiplier , which is det er min ed by the following conditions.
STEP 2 : obtain the equations
F f 
= 0 i.e. + =0 ...............(2)
x x x
F f 
= 0 i.e. + =0 ................(3)
y y y
F f 
= 0 i.e. +  =0 ..................(4)
z z z
STEP3 : solve the equations (2), (3), (4)and (1)to get x, y, z values
the values of x, y, z so obtained will give the stationary po int of f ( x, y, z ).

Problem:
1. Find the minimum value of x2 +y2 +z2, given x + y + z =3a
Sol: f(x,y,z) = x2 +y2 +z2
= x + y + z - 3a = 0
Using Lagrange’s function
F(x , y , z) = f(x , y , z) + λ ( x , y , z)

=( x2 +y2 +z2)+ λ(x + y + z - 3a)

For maxima or minima


F f 
=0 + = 2x + λ = 0 ------------ (1)
x x x
F f 
=0 +  = 2y + λ = 0 ------------ (2)
y y y

F f 
= 0: +  = 2z + λ = 0 ------------ (3)
z z z
(1) , (2) & (3)
λ= -2x = -2y = -2z

Page | 115
Substitute x=y=z in = x + y + z - 3a = 0
 = x + x + x - 3a = 0
 x=a
x= y =z = a
hence the stationary point is (x,y,z)=(a,a,a) which is the minimum point
Minimum value of f(x,y,z) = x2 +y2 +z2 = a2 + a2 + a2 =3 a2

2. Find the maximum value of x m y n z p given that x + y + z = a where m  n  p .

Sol: Given that f ( x, y, z ) = x m y n z p

Page | 116
given condition x + y + z = a  x + y + z − a = 0
  ( x, y , z ) = x + y + z − a ........................... (1)
By lagrange ' s method
F ( x, y, z ) = f ( x, y, z ) +  ( x, y, z )
F = xm y n z p +  ( x + y + z − a)
Differentiate equation (1) w.r .to x, y , z partially , we get
F x
= 0  y n z p mx m −1 +  = 0   = − my n z p x m −1 *
x x
−mf −mf
 = x= ..........(2)
x 
F y
= 0  x m z p ny n −1 +  = 0   = −nx m z p y n −1 *
y y
−nf −nf
 = y= ..........(3)
y 
F z
= 0  x m y n pz p −1 +  = 0   = − px m y n z p −1 *
z z
− pf − pf
 = z= ..........(4)
z 
−mf −nf − pf
= = = but m  n  p we cant say x = y = z
x y z
Substitute (2),(3) & (4) in eqn(1)
−mf − nf − pf
(1) : x + y + z = a  + + =a
  
−( m + n + p ) f
=
a
−mf ma
From(2) : x = =
 m+n+ p
−nf na
From(3) : y = =
 m+n+ p
− pf pa
From(4) : z = =
 m+n+ p

Page | 117
 ma na pa 
 ( x, y , z ) =  , , 
 m+n+ p m+n+ p m+n+ p
is the stationary po int which is max imum po int
The Maximum value is f ( x, y , z ) = x m y n z p
m n p
 ma   na   pa 
=     
m+n+ p m+n+ p m+n+ p
a m + n + p .m m .n n . p p
=
(m + n + p)m + n + p

HOME WORK: FIND THE EXTREMA OF x 2 y 3 z 4 given that 2 x + 3 y + 4 z = a .

3.Find a point on the plane 3x+2y+z-12=0 which is nearest to the origin.

Sol: Let p(x,y,z) be a point on the plane which is nearest to the origin

op = ( x − 0) 2 + ( y − 0) 2 + ( z − 0) 2
op 2 = x 2 + y 2 + z 2
let op 2 = f ( x, y , z )
 f ( x, y , z ) = x 2 + y 2 + z 2 ..................(1)
given that 3 x + 2 y + z + 2 = 0
 ( x, y, z ) = 3 x + 2 y + z − 12 ...................(2)
By Lagrange ' s method
F ( x, y , z ) = f ( x, y , z ) +  (3 x + 2 y + z − 12) ...............(3)
differentiate equation (3) w.r .to x, y , z partially and equating to zero
F F F
= 0, = 0, =0
x y z

2 x + 3 = 0 .......... ........( 4)
. 2 y + 2 = 0 .......... .........( 5)
2z +  = 0 .......... .......... (6)

−3 −
from (4) x = ; from (5) y = − ; from (6) z =
2 2
substitute x, y , z values in (2)
−3  −12
3( ) + 2( − ) − − 12 = 0  =
2 2 7

18 12 6
x= ; y= ;z=
7 7 7

18 12 6
 The stationary po int is ( , , )
7 7 7
18 12 6
 p ( , , ) is the po int on the the plane which is nearest to origin
7 7 7

Page | 118
4. Find the volume of the largest rectangular parallelepiped that can be inscribed in the
x2 y2 z 2
ellipsoid 2 + 2 + 2 = 1.
a b c
Sol: Let x, y, z be the length, breadth and height of the rectangular parallelepiped that
can be inscribed in the ellipsoid
Its volume is V = xyz
Let f(x,y,z) =xyz
x2 y 2 z 2
Given  ( x, y, z ) = 0  2 + 2 + 2 − 1 = 0
a b c
Form Lagrangean function F(x,y,z) as
𝐹 (𝑥, 𝑦, 𝑧) = 𝑓(𝑥, 𝑦, 𝑧) + 𝜑(𝑥, 𝑦, 𝑧)

 x2 y 2 z 2 
F ( x, y, z ) = xyz +   2 + 2 + 2 − 1 ...........(2)
a b c 
Where λ is the multiplier to be determined such that
F F F
= 0, = 0, =0
x y z
2x
yz + 2  = 0 ...........(3)
a
2y
zx + 2  = ...........(4)
b
2z
xy + 2  = 0 ...........(5)
c
Now (3),(4),(5) are combined as:
− a 2 yz −b 2 zx −c 2 xy
= = =  .............(6)
2x 2y 2z
from first two fractions , we have
a 2 y b2 x x2 y2
=  2 = 2 ............(7)
x y a b
y2 z2
similarly =
b2 c2
substituting (7)and (8) in (1), we get
x2 y 2 z 2
 ( x, y , z ) = 0 :
+ + −1 = 0
a 2 b2 c2
x2 x2 x2 3x 2 a2
+ + − 1 = 0  = 1  x 2
=
a2 a2 a2 a2 3
a b c
x= ,y= ,z =
3 3 3
 a b c 
hence the possible extreme po int is p  , , 
 3 3 3

a b c abc
Thus v is maximum at p( , , )and its maximum value is v=xyz =
3 3 3 3 3
Page | 119
Page | 120
UNIT -IV
Multiple Integrals

Page | 121
Let y=f(x) be a function of one variable defined in the interval (a,b) then
𝑏
∫𝑎 𝑓(𝑥) 𝑑𝑥 is defined as the definite integral of f(x) between x=a and x=b.
This integral gives the area bounded by the curve y=f(x) between the
lines x=a and x=b.
Extending this definition to a function of two variables is called double
integral and to a function of three variables is called triple integral.
Double integral:
If f(x,y) is a function of two variables over a region R then
∬𝑅 𝑓(𝑥, 𝑦)𝑑𝑥𝑑𝑦 is called Double integral.
If the region R is bounded by the lines 𝑥 = 𝑥1 , 𝑥 = 𝑥2 , 𝑦 = 𝑦1 , 𝑦 = 𝑦2
2 2 𝑥 𝑦
then ∬𝑅 𝑓(𝑥, 𝑦)𝑑𝑥𝑑𝑦 = ∫𝑥=𝑥 ∫𝑦=𝑦 𝑓(𝑥, 𝑦)𝑑𝑥𝑑𝑦
1 1

Evaluation of Double Integrals:


1. If all the limits are constants ie 𝑥 = 𝑥1 , 𝑥 = 𝑥2 , 𝑦 = 𝑦1 , 𝑦 = 𝑦2 then
𝑥2 𝑦2
∬ 𝑓(𝑥, 𝑦)𝑑𝑥𝑑𝑦 = ∫ ∫ 𝑓(𝑥, 𝑦)𝑑𝑥𝑑𝑦
𝑅 𝑥=𝑥1 𝑦=𝑦1

2 𝑥 2 𝑦 2 2 𝑦 𝑥
= ∫𝑥=𝑥 [∫𝑦=𝑦 𝑓 (𝑥, 𝑦)𝑑𝑦] 𝑑𝑥 (or) = ∫𝑦=𝑦 [∫𝑥=𝑥 𝑓 (𝑥, 𝑦)𝑑𝑥 ] 𝑑𝑦
1 1 1 1

2. If x limits are constants and y has variable limits (i.e., the limits of y are
functions of x) i.e., 𝑥 = 𝑥1 , 𝑥 = 𝑥2 , 𝑦1 = 𝜑1 (𝑥 ), 𝑦2 = 𝜑2 (𝑥) then
𝑥2 𝜑2 (𝑥)
∬ 𝑓(𝑥, 𝑦)𝑑𝑥𝑑𝑦 = ∫ [∫ 𝑓 (𝑥, 𝑦)𝑑𝑦] 𝑑𝑥
𝑅 𝑥=𝑥1 𝑦=𝜑1 (𝑥)

3. If y limits are constants and x has variable limits (i.e., the limits of x are
functions of y ) i.e., 𝑥1 = 𝜑1 (𝑦), 𝑥2 = 𝜑2 (𝑦), 𝑦 = 𝑦1 , 𝑦 = 𝑦2 , then
𝑦2 𝜑2 (𝑦)
∬ 𝑓(𝑥, 𝑦)𝑑𝑥𝑑𝑦 = ∫ [∫ 𝑓 (𝑥, 𝑦)𝑑𝑥 ] 𝑑𝑦
𝑅 𝑦=𝑦1 𝑥=𝜑1 (𝑦)

Double integral in Polar form:


Page | 122
If the region is bounded by the lines 𝜃 = 𝜃1 , 𝜃 = 𝜃2 ,
𝑟1 = 𝜑1 (𝜃), 𝑟2 = 𝜑2 (𝜃) then
𝜃2 𝜑2 (𝜃)
∬ 𝑓(𝑟, 𝜃)𝑑𝑟𝑑𝜃 = ∫ [∫ 𝑓 (𝑟, 𝜃)𝑑𝑟] 𝑑𝜃
𝑅 𝜃=𝜃1 𝑟1 =𝜑1 (𝜃)

Problems
2 3

  xy dx dy
2
1. Evaluate
1 1

2 3 2
3 2 
  xy dx dy = y =1  x=1 xy dx dy
2
Sol :
1 1
3
2
 x2 
=    y 2 dy
y =1 
2  x =1
2
y2
=  9 − 1 dy
1
2
2
8
=  y 2 dy
21
2
= 4  y 2 dy
1
2
 3 
= 4y 
 3 1
4
= 8 − 1
3
4*7 28
= =
3 3

Page | 123
2 x 2 x
2. evaluate   y dx dy =
0 0
 
x =0 y =0
y dx dy


2 x

=    y dy dx
0 0 
x
 y2 
2
=    dx
0 
2 0
2
 x2 
=   − 0 dx
0  
2
2
1  x3 
=  
2  3 0
1 8  4
= −0 =
2  3  3

2 x  x  2

  y dy dx =  y=0  dx

x =0 
y dy
x =0 y =0  

5 x2

  x(x + y 2 )dx dy
2
3. Evaluate
0 0

x2 x2
5
 xy 3 
5
Sol.   x ( x + y ) dy dx =   x 3 y +
2 2
 dx
x =0 y =0 x =0  3  y =0
5
 3 2 x ( x 2 )3 
=   x .x +
3 
dx
x =0 
5
5
 x7   x 6 1 x8 
=   x5 +  dx =  + . 
x =0 
3  6 3 8 0
6
5 58
= +
6 24

1 1+ x 2
dydx
4. Evaluate  
0 0
1 + x2 + y 2

1 1+ x 2
dydx  1 1+ x 2
1 
Sol:   1 + x 2 + y 2 x=0 
=   dy
(1 + x2 ) + y 2 
 dx
0 0  y =0

Page | 124
1+ x2
 
1+ x
 −1 y 
2
1
 1  1
1
=   dy  dx =  Tan  dx
( )
2
x =0 1 + x  1 + x2 
2
x =0  y =0 1 + x2 + y 2 
  y =0

1 1
[ x 2
+a 2
dx = tan −1 ( x )]
a a
1
1
= 
x =0 1+ x 2
Tan −11 − Tan −1 0  dx

 1
1
=
4 
x =0 1 + x2
dx

 1
= sinh −1 x 
4 x =0

  
=  − 0
4 2 
2
=
4

4 x2
5. Evaluate e
y/x
dydx
0 0

Answer: 3e4-7
1 x

6. Evaluate   ( x 2 + y 2 )dxdy
0 x

Answer: 3/35
2 x

e
( x+ y )
7. Evaluate dydx
0 0

1
Answer: 2 [𝑒 4 − 2𝑒 2 + 1]


2 1

x
2
8. Evaluate y 2 dxdy
0 −1

3
Answer:
36

Page | 125


e
− ( x2 + y 2 )
9. Evaluate dxdy
0 0

 
  − x2 
e dxdy =  e   e dx  dy
− ( x2 + y 2 ) − y2
Sol:
0 0 0 0 

 
 
=  e− y  e dx =
−x
2 2
dy
0
2 0
2

   
e
− y2
= dy = . =
2 0
2 2 4

Alter:

 2 

e   e − r rdrd ( x 2 + y 2 = r 2 )
−( x + y )
dxdy =
2 2 2

0 0  =0 r =0

(Changing to polar coordinates taking x = r cos  , y = r sin  )


  
 e− r   0 − 1
2
2 2
= 
0

 −2
 d =
 0
  −2  d
0

( )

( )0 2 =  2 − 0
1 1
=
2 2

=
4
  xy( x + y)dxdy over the region R bounded by y=x
2
10. Evaluate and y=x
2
Sol: y= x is a parabola through (0,0) symmetric about y-axis y=x is a straight line through
(0,0) with slope1.
2 2
Let us find their points of intersection solving y= x , y=x we get x =x  x=0,1Hence
y=0, 1

The point of intersection of the curves are (0,0), (1,1)


Consider  xy ( x + y )dxdy
R

For the evaluation of the integral, we first integrate w.r.t ‘y’ from y=x 2 to y=x
and then w.r.t. ‘x’
from x=0 to x=1

Page | 126
1
 x xy ( x + y ) dy  dx = 1  x ( x 2 y + xy 2 ) dy dx
x=0  y= x2  x=0 y= x2 

x
1  y 2 xy 3 
=   x2 +  dx
x =0
 2 3  y = x2

1  x4 x4 x6 x7 
=   + − − dx
x =0
 2 3 2 3

1  5x x6 x7 
4
=  − − dx
x =0
 6 2 3
1
 5 x 5 x 7 x8 
= . − − 
 6 5 14 24 0
1 1 1 28 − 12 − 7 28 − 19 9 3
= − − = = = =
6 14 24 168 168 168 56

11. Evaluate   xydxdy


R
where R is the region bounded by x-axis and x=2a and the curve

x2=4ay.
Sol. The line x=2a and the parabola x2=4ay intersect at B(2a,a)
The given integral =   xy dx dy
R

Let us fix ‘y’

For a fixed ‘y’, x varies from 2 ay to 2a. Then y varies from 0 to a.

Hence the given integral can also be written as


a x =2a a
 x =2 a xdx  ydy
 
y = 0 x = 2 ay
xy dx dy = 
y =0  x =2 ay 

Page | 127
2a
a  x2 
= 2 ydy
y =0
  x =2 ay

a
=  2a 2 − 2ay  y dy
y =0

a
 2a 2 y 2 2ay 3  2a 4 3a 4 − 2a 4 a 4
= −  = a 4
− = =
 2 3 0 3 3 3


2
1
12. Evaluate   r sin  d dr
0
0

 2  
r =0   =0 sin  d dr = r =0 r ( − cos  ) =20 dr
1 1
Sol. r

(
−r cos  )
1
= − cos 0 dr
r =0 2
1
 r2  1 1
=  −r ( 0 − 1) dr =  rdr =   = − 0 =
1 1

r =0 0
 2 0 2 2
13. Evaluate  ( x + y 2 )dx dy in the positive
2

quadrant

For Which
x + y 1

 ( x + y 2 )dx dy =  (x + y 2 ) dy
1 y =1− x
Sol. 2
dx  2
x =0 y =0
R
1− x
 2
1 y3 
=   x y +  dx
x =0
 3 0
1
 1 3  x3 x 4 1 4
=   x 2 − x3 + (1 − x )  dx =  − − (1 − x ) 
1

x =0
 3   3 4 12 0

1 1 1 1
= − −0+ =
3 4 12 6

Page | 128
 ( x + y 2 )dx dy over the area bounded by the ellipse
x2 y2
14. Evaluate 2
+ =1
a 2 b2

x2 y2
Sol. Given ellipse is + =1
a 2 b2
y2 x2 b2 2
2 ( ) 2 (
a − x2 )
1 2
i.e., 2
= 1 − 2
= a − x 2
( or ) y 2
=
b a a a
b 2
y = a − x2
a
Hence the region of integration R can be expressed as
−b 2 b
− a  x  a, a − x2  y  a2 − x2
a a

  ( x 2 + y 2 ) dx dy =  (x + y 2 ) dx dy
a b a2 − x2

a 2
x =− a y = − b a 2 − x 2
R a

a  b3 3 
= 2  x 2 . b a 2 − x 2 + 3 ( a 2 − x 2 ) 2  dx
−a a
 3a 
a b3 3 
= 4  b x 2 a 2 − x 2 + 3 ( a 2 − x 2 ) 2  dx
a
0
 3a 
Changing to polar coordinates
putting x = a sin 
dx = a cos  d
x x
= sin    = sin −1
a a
x → 0,  → 0

x → a,  →
2

Page | 129
  ab3   1 1  ab3 3 1  
= 4 2  a3b sin 2  cos 2  + cos 4   d = 4  a 3b. . + . . . 
0
 3   4 2 2 3 4 2 2
  1 
 .
n −1 n−3 2 2
2

0    =
m n
 sin cos d . ......... 
 m+n m+n−2 m 
 
4 3  ab 2 2
=
16
( a b + ab3 ) =
4
(a + b )

1 1 𝑑𝑥 𝑑𝑦
15. Evaluate ∫0 ∫0
√1−𝑥 2 √1−𝑦 2

1 1 𝑑𝑥 𝑑𝑦 1 1 1 𝑑𝑥
Sol: Given integral ∫0 ∫0 =∫0 [∫0 𝑑𝑦] √1−𝑥2
√1−𝑥 2 √1−𝑦 2 √1−𝑦 2

1 1 𝑑𝑥
= ∫0 [∫0 𝑑(sin−1 𝑦)] √1−𝑥2

1 𝑑𝑥
=∫0 (sin−1 𝑦)10 √1−𝑥2

𝜋 1
= ( ) ∫0 𝑑 (sin−1 𝑥 ) 𝑑𝑥
2

𝜋
= [sin−1 𝑥 ]10
2

𝜋 𝜋
= ( )
2 2

1 1 𝑑𝑥 𝑑𝑦 𝜋2
∴ ∫0 ∫0 =
√1−𝑥 2 √1−𝑦 2 4

Double integrals in polar co-ordinates:


Page | 130
 a sin  rdrd
 
4
1. Evaluate 0 0
a2 − r 2
 a sin  rdrd   a sin  r    a sin  −2r 
Sol.  
4
=  4  dr d = −1  4   dr  d
a2 − r 2 a2 − r 2  2 0 a2 − r 2 
 
0 0 0 0 0

−1  4
( )
a sin  
4 
=  2 a 2
− r 2
d = ( −1 )  2 a 2 − a 2 sin 2  − a 2 − 0  d
2 0 0 0  


= ( −a )  ( cos  − 1) d = ( −a )( sin  −  )0
4
4
0

= ( −a )  sin  −   − ( 0 − 0 ) 
 4 4 
= ( −a )  1 −   = 2  − 1 
 2 4  4 2

 a sin  a 2
1. Evaluate 0  r dr d Ans:
0 4
 

Evaluate 0  e− r r d dr
2
2
2. Ans:
0 4
 a (1+ cos ) 3 a 2
3. Evaluate 0  r dr d Ans:
0 4

Change of order of Integration:


4a 2 ax
1. Change the order of Integration and evaluate  x =0 y=x
2
4a
dy dx

x2
Sol. In the given integral for a fixed x, y varies from to 2 ax and then x varies
4a

x2
from 0 to4a. Let us draw the curves y = and y = 2 ax
4a

The region of integration is the shaded region in diagram.


4a 2 ax
The given integral is = x =0 y = x 2 dy dx
4a

Page | 131
y2
Changing the order of integration, we must fix y first, for a fixed y, x varies from to
4a

4ay and then y varies from 0 to 4a.

Hence the integral is equal to


4a 2 ay 4a  2 ay 
 
y =0 x = y
2
4a
dx dy = 
y = 0  x = y

2
4a
dx dy

 y 2 d y
 x x = y
4a 4a
= dy =  −
2 ay
2 ay
y =0  4a 
2
y =0
4a 
4a
 y 2 1 y3 
3

= 2 a . − . 
 3 4a 3 
 2 0
4 1
= . a .4a 4a − .64a 3
3 12a
32 2 16 2 16 2
= a − a = a
3 3 3

(x + y 2 ) dx dy
a x
2. Change the order of integration and evaluate =  x a 2
0
a

x x
Sol. In the given integral for a fixed x, y varies from to and then x varies from 0 to a
a a

x x
Hence we shall draw the curves y = and y =
a a

i.e ay=x and ay2=x


We get ay = ay 2

 ay − ay 2 = 0

 ay (1 − y ) = 0

 y = 0, y = 1
If y=0, x=0 if y=1, x=a

Page | 132
The shaded region is the region of integration. The given integral is

(x + y 2 )dx dy
a y= x
 
a 2
x =0 y= x
a

Changing the order of integration, we must fix y first. For a fixed y, x varies from ay 2 to
ay and then y varies from 0 to 1.
Hence the given integral, after change of the order of integration becomes

(x + y 2 )dx dy =    (x + y 2 ) dx  dy
1 ay 1 ay
 
2 2
y =0 x = ay 2
y =0 x = ay 2 
ay
1  x3 
=   + xy 2  dy
y =0
3  x =ay 2

1  a3 y 3 a3 y 6 
=  + ay 3
− − ay 4  dy
y =0
 3 3 
1
 a3 y 4 ay 4 a3 y 7 ay 5 
= + − − 
 12 4 21 5  y =0

a3 a a3 a a3 a
= + − − = +
12 4 21 5 28 20

1 2− x
3. Change the order of integration in 
0 x2
xydxdy and hence evaluate the double integral.

Sol. In the given integral for a fixed x, y varies from x2 to 2-x and then x varies from 0 to 1.
Hence we shall Draw the curves y=x2 and y=2-x
The line y=2-x passes through (0, 2), (2, 0)
Solving y=x2, y=2-x

Then we get x = 2 − x
2

 x2 + x − 2 = 0

Page | 133
 x2 + 2 x − x − 2 = 0
 x ( x + 2 ) − 1( x + 2 ) = 0

 ( x − 1)( x + 2 ) = 0

 x = 1, −2
If x = 1, y = 1
If x = −2, y = 4
Hence the points of intersection of the curves are (-2, 4), (1, 1)
The Shaded region in the diagram is the region of intersection.
Changing the order of integration, we must fix y, for the region within OACO for a fixed

y, x varies From 0 to y

Then y varies from 0 to 1


For the region within CABC, for a fixed y, x varies from 0 to 2-y , then y varies from 1 to 2
1 2− x
Hence 0 x2
xy dy dx =
OACO
 xy dx dy + 
CABC
xy dx dy

=
1  y xdx  y dy + 2  2− y x dx  y dy
 x =0
y =0   y =1 x=0 
2− y
y
1  x2  2  x 
2
=    y dy +    y dy
y =0 y =1 2
 2  x =0   x =0

2 (2 − y)
2
1 y
= . y dy +  y dy
y =0 2 y =1 2

. ( 4 y − 4 y 2 + y 3 ) dy
1 1 2 1 2
=
2  y = 0
y dy +
2 y =1
1 2
1  y3  1  4 y 2 4 y3 y 4 
= .  + .  − + 
2  3 0 2  2 3 4 1

1 1 1
= . +  2.4 − 2.1 − 4 ( 8 − 1) + 1 (16 − 1) 
2 3 2 3 4 

1 1  28 15  1 1  72 − 112 + 45  1 1  5  4 + 5 9 3
= + 6− +  = +   = 6 + 2 12  = 24 = 24 = 8
6 2  3 4  6 2 12

a 2a− x
4. Changing the order of integration  
x2
xy 2 dy dx
0
a

Page | 134
1 1− x2 
5. Change of the order of integration   y 2 dx dy Ans :
0 0 16

Hint: Now limits are y = 0 to 1and x = 0 to 1 − y 2


put y = sin 
1 − y 2 = cos 
dy = cos  d
1
=  y 2 1 − y 2 dy
0

  
= 2
sin 2  cos 2  d =  2
sin 2  d −  2
sin 4  d
0 0 0

=
1 
( )
− .  =
3 1
2 2 4 2 2 16 ( )

Change of variables:
The variables x, y in  f ( x, y )dx dy are changed to u, v with the help of the relations
R

x = f1 ( u, v ) , y = f 2 ( u, v ) then the double integral is transferred into

 ( x, y )
 f  f1 ( u, v ) , f 2 ( u, v )  du dv
R1
 ( u, v )

Where R1is the region in the uv plane, corresponding to the region R in the xy-plane.

Changing from Cartesian to polar co-ordinates


x = r cos  , y = r sin 
x x
 ( x, y )  r  cos  −r sin 
   = =
 ( r ,  )  y y sin  r cos 
r 

= r ( cos2  + sin 2  ) = r   f ( x, y )dx dy =  f ( r cos  , r sin  )r dr d


R R1

Page | 135
Note: In polar form dxdy is replaced by r dr d
Problems:
  (
− x2 + y 2 )
1. Evaluate the integral by changing to polar co-ordinates  
0 0
e dx dy

Sol. The limits of x and y are both from 0 to  .


The region is in the first quadrant where r varies from 0 to  and  varies from 0 to

2
Substituting x = r cos  , y = r sin  and dx dy = r dr d
  (
− x2 + y 2 )  
  dx dy =   e− r r dr d
2
Hence e 2
0 0  =0 r =0

Put r 2 = t
 2rdr = dt
 r dr = dt
2
Where r = 0  t = 0 and r =   t = 
  (
− x2 + y 2 )   1 −t
  e dx dy =  2
 e dt d
0 0  =0 t =0 2
 −1 − t 
=
0
2

2
( e )0 d
−1  2 
0 − 1) d  ( )0 2 =  = 
1 1
=  (
2 0 2 2 2 4

(x + y 2 )dx dy
a a2 − y2

2
3. Evaluate the integral by changing to polar co-ordinates 0 0

Sol. The limits for x are x=0 to

x = a2 − y2
 x2 + y 2 = a2

The given region is the first quadrant of the circle.


By changing to polar co-ordinates
x = r cos  , y = r sin  , dx dy = r dr d

Here ‘r’ varies from 0 to a and ' ' varies from 0 to  2


a
   r4  a4
(x + y ) dx dy = 
a2 − y2 
r rdrd =  ( )0 2
a a
 
2 2 2

2 2
  d =
 =0 r =0
 4 0 4
0 0 0

=  a4
8

Page | 136
4a y x2 − y 2  5
4. Show that  
0 y 2
4a x + y
2 2
dx dy = 8a 2  − 
 2 3
2
Sol. The region of integration is given by x = y 4a , x = y and y=0, y=4a

i.e., the region is bounded by the parabola y2=4ax and the straight line x=y.
Let x = r cos  , y = r sin  .Then dx dy = rdrd
The limits for r are r=0 at O and for P on the parabola
4a cos 
r 2 sin 2  = 4a ( r cos  )  r =
sin 2 

For the line y=x, slope m=1 i.e., Tan = 1,  =  4

The limits for  :  4 →  2

Also x2 − y 2 = r 2 ( cos2  − sin 2  ) and x2 + y 2 = r 2

x2 − y 2  4 a cos
( cos 2  − sin 2  )rdrd
4a y
  2 dx dy = 2  sin 2 

4a x + y
0 y 2 2  = r = 0
4

4 a cos
  r2  sin 2 
=   ( cos  − sin  )  
2 2 2
d
= 4
 2 0
 cos 2 
= 8a 2 
4
2
( cos2  − sin 2  ) sin 4 
d


 3 − 8    5
= 8a 2  2

4
( cos 4
 − cot 2  ) d = 8a 2 
 12
+ − 1 = 8a 2  −  .
4   2 3

Page | 137
Triple integrals:
If x1, x2 are constants. y1, y2 are functions of x and z1, z2 are functions of x and y,
then f(x, y, z) is first integrated w.r.t. ‘z’ between the limits z1 and z2 keeping x and y fixed.
The resulting expression is integrated w.r.t ‘y’ between the limits y1 and y2 keeping x
constant. The resulting expression is integrated w.r.t. ‘x’ from x1 to x2

y = g2 ( x ) z = f2 ( x, y )
i.e.  f ( x, y, z )dx dy dz = f ( x, y, z ) dz dy dx
b

v
x=a  y = g1 ( x )  z = f1 ( x , y )

Problems
1 1− x 2 1− x 2 − y 2
1. Evaluate 
0 0 0
xyz dx dy dz

1 1− x 2 1− x 2 − y 2 1 1− x 2 1− x 2 − y 2
Sol.  
x =0 y =0 
z =0
xyz dx dy dz = 
x =0
dx 
y =0
dy 
z =0
xyz dz

1− x 2 − y 2
1 1− x 2  z2 
= dx  xy   dy
x =0 y =0
 2  z =0

xy (1 − x 2 − y 2 ) dy
1 1 1− x2

2 x =0 y =0
= dx

x (1 − x 2 ) y − y 3  dy
1 1 1− x2
=
2 x = 0
dx y = 0

1− x 2
1 1  y2 y4 
=  x (1 − x 2 ) −  dx
2 x =0  2 4 0

1− x 2
1 1  y 2 x2 y 2 y 4 
= . x  − −  dx
2 x =0  2 2 4 0

= . x  2 (1 − x 2 ) − 2 x 2 (1 − x 2 ) − (1 − x 2 )  dx
1 1 2

8 x = 0  
1
1  x2 2 x4 x6 
=  ( x − 2 x3 + x5 ) dx =  −
1 1
+ 
8 x =0 8 2 4 6 0

11 1 1 1
=  − + =
8  2 2 6  48

Page | 138
x+ z
   ( x + y + z )dx dy dz
1 z
2. Evaluate −1 0 x− z

1 z x+ z
 y 2 + zy  dx dz
x+ z

( x + y + y ) dx dydz = 
1 z

−1 0
Sol :     xy +
2  
−1 0 x − z   x − z 

x+ z x−z
2 2

x( x + z) − x( x − z) + 
1 z
=  − + z ( x + z ) − z ( x − z )dx dz
−1 0
 2   2 
1 z  1 
=   2 z ( x + z ) + 2 4 xz dx dz
−1 0

z
 x21 x2 
= 2  z. + z 2 x + z.  dz
−1
 2 2 0
1
 z31 z3   z4 
= 2.  + z 3 + dz = 4.   = 0
−1 2
 2  4 −1

1 2 3
3. Evaluate ∫0 𝑑𝑥 ∫1 𝑑𝑦 ∫1 𝑥𝑦𝑧𝑑𝑧

3
1 2 3 1 2 𝑧2
Sol: ∫0 𝑑𝑥 ∫1 𝑑𝑦 ∫1 𝑥𝑦𝑧𝑑𝑧 = ∫0 𝑑𝑥 ∫1 𝑑𝑦 [𝑥𝑦 ( ) ]
2 1

1 2 1
= ∫0 𝑑𝑥 ∫1 𝑑𝑦 [2 𝑥𝑦(9 − 1)]

1 2 8
= ∫0 𝑑𝑥 ∫1 𝑑𝑦 [2 𝑥𝑦]

1 2
= ∫0 𝑑𝑥 ∫1 4𝑥𝑦𝑑𝑦

2
1 𝑦2
= ∫0 𝑑𝑥(4𝑥) ( 2 )
1

=4

𝑒 log 𝑦 𝑒𝑥
4. Evaluate ∫1 ∫1 ∫1 log 𝑧 𝑑𝑧𝑑𝑥𝑑𝑦

𝑒 log 𝑦 𝑒𝑥 𝑒 log 𝑦 𝑥
Sol: ∫1 ∫1 ∫1 log 𝑧 𝑑𝑧𝑑𝑥𝑑𝑦 = ∫1 ∫1 [𝑧 log 𝑧 − 𝑧]1𝑒 𝑑𝑥𝑑𝑦

𝑒 log 𝑦
= ∫1 ∫1 [(𝑒 𝑥 log 𝑒 𝑒 𝑥 − 𝑒 𝑥 ) − (0 − 1)]𝑑𝑥𝑑𝑦

Page | 139
𝑒 log 𝑦
= ∫1 [∫1 (𝑥𝑒 𝑥 − 𝑒 𝑥 + 1)𝑑𝑥] 𝑑𝑦

𝑒 log 𝑦
= ∫1 (𝑥𝑒 𝑥 − 𝑒 𝑥 (1) − 𝑒 𝑥 + 𝑥)1 𝑑𝑦

𝑒 log 𝑦
= ∫1 (𝑥𝑒 𝑥 − 2𝑒 𝑥 + 𝑥)1 𝑑𝑦

𝑒
=∫1 [(log 𝑦 𝑒 log 𝑦 − 2𝑒 log 𝑦 + log 𝑦) − (1𝑒 − 2𝑒 + 1)]𝑑𝑦

𝑒
= ∫1 [(log 𝑦 (𝑦) − 2𝑦 + log 𝑦) − (1 − 𝑒)]𝑑𝑦

𝑒
𝑦2 𝑒 𝑦2 1 𝑦2
=[log 𝑦 ( 2 ) − ∫1 (𝑦) 𝑑𝑦 − 2 + (𝑦 log 𝑦 − 𝑦) − (1 − 𝑒)𝑦]
2 2 1

𝑒2 13
= − 2𝑒 +
4 4

5. Evaluate ∭𝑉(𝑥𝑦 + 𝑦𝑧 + 𝑧𝑥)𝑑𝑥𝑑𝑦𝑑𝑧 where V is the region of space bounded by the

planes x=0, x=1, y=0, y=2 and z=0, z=3.

3 2 1
Sol: ∭𝑉(𝑥𝑦 + 𝑦𝑧 + 𝑧𝑥)𝑑𝑥𝑑𝑦𝑑𝑧 = ∫𝑧=0 ∫𝑦=0 ∫𝑥=0 ∭𝑉 (𝑥𝑦 + 𝑦𝑧 + 𝑧𝑥 )𝑑𝑥𝑑𝑦𝑑𝑧

3 2 𝑥2 𝑥2
= ∫𝑧=0 ∫𝑦=0[ 𝑦 + 𝑥𝑦𝑧 + 𝑧]10 𝑑𝑦𝑑𝑧
2 2

3 2 𝑦 𝑧
= ∫𝑧=0 ∫𝑦=0[ + 𝑦𝑧 + ]𝑑𝑦𝑑𝑧
2 2

3 𝑦2 𝑦2 𝑦𝑧 2
= ∫𝑧=0[ + 𝑧+ ] 𝑑𝑧
4 2 2 0

3
= ∫𝑧=0[ 1 + 3𝑧]𝑑𝑧

3𝑧 2 3 33
= [𝑧 + ] =
2 0 2

Volume as a triple integral:

Suppose a three dimensional solid is cut in to elemental rectangular


parallelepipeds by drawing planes parallel to the coordinate planes. The volume of an
elemental parallelepiped v is x, y, z .Hence the total volume of the solid
is  dv =  dxdydz z where the integration is carried over the entire volume.
v v

Page | 140
1. Evaluate   ( x + y + z ) dzdydxx where R is the region bounded by the planes x =0,x =1,
R

y=0, y=1,z =0,z =1.


1 1 1

Sol:   ( x + y + z ) dzdydxx =    ( x + y + z) dzdydxx


R x =0 y =0 z =0

1 1
z2 1
=   ( xz + yz + ) 0 dydx x
0 0
2

1 1
1
=   ( x + y + 2 )dydx
0 0

1
y2 y 1
=  ( xy + + ) 0 dx
0
2 2

1
1 1
=  ( x + 2 + 2 )dx
0

1
=  ( x + 1)dx
0

x2
= ( + x)10
2

1 3
= +1 = .
2 2

Change of Variables in a Triple Integral:

Let the functions x = 1 (u, v, w), y =  2 (u, v, w) & z = 3 (u, v, w) be the transformations from
Cartesian coordinates to the curvilinear coordinate’s u, v, and w.

The Jacobin for this transformation is given by

 ( x, y , z )
J=
 (u , v, w)

Then  f ( x, y, z)dxdydz =  f ( 


v v
1, 2, 3  ). J dudvdw

Where v is the corresponding domain in the curvilinear coordinates u, v, w.

Page | 141
a) Change of variables from Cartesian to spherical polar coordinate system :

In problems having symmetry w.r.t a point O, it would be convenient to use spherical


coordinates with this point chosen as origin.
The relation between Cartesian coordinates x,y,z and spherical polar
coordinates  , ,  (i.eu =  , v =  , w =  ) are given by
x =  sin cos ((or)r sin cos )
y =  sin sin  ((or)r sin sin  )
z =  sin ((or)r cos )
And dxdydz = J ddd (or) J drdd

x x x
  
 ( x, y , z ) y y y
Where J = =
 (  ,  ,  )   
z z z
  

sin  cos  cos cos −  sin  sin 


=| sin  sin   cos sin   sin cos |
cos −  sin  0

=  2 sin (or)r 2 sin

 f ( x, y, z)dxdydz =  f (  sin cos ,  sin sin ,  cos )  sinddd


2

v v

The region V in (x, y, z) is to be covered by the limits of  ,  ,  and is denoted as V1


b) Change of variables from Cartesian to cylindrical coordinate system :

The relation between the Cartesian coordinates x, y, z and cylindrical coordinates


 , , z (i.eu =  , v =  , w = z ) are given by
x =  cos ((or )r cos )
y =  sin  ((or ) r sin  )
z=z
x x x
  z
 ( x, y , z ) y y y
And J = =
 (  ,  , z )   z
z z z
  z

Page | 142
cos −  sin  0
= sin   cos 0 =
0 0 1

 f ( x, y, z)dxdydz =  f (  cos ,  sin , z) dddz


v v

1 (1− x 2 ) (1− x 2 − y 2
dxdydz
1. Evaluate  
0 0

0 (1 − x 2 − y 2 − z 2 )
by changing in to spherical polar coordinates.

Sol: Changing to spherical polar coordinates by putting

x = r sin  cos
y = r sin  sin 
z = r cos

We have J = r2sin  , x 2 + y 2 + z 2 = r 2 & dxdydz = r 2 sindrdd.

Also given region of integration is the volume of the sphere x 2 + y 2 + z 2 = 1


In the positive octant for which r varies from 0 to 1,  varies from 0 to and
2


 Varies from 0 to .
2
 
(1− x 2 ) (1− x 2 − y 2
r 2 sin drdd
1 1 2 2
dxdydz
   = 
0 0 0 (1 − x 2 − y 2 − z 2 ) 0 0 0 (1 − r 2 )

 
2 2  1 1 − (1 − r 2 )dr 
= 0 0 0 (1 − r 2 )  sindd
 
 
 1
2 2
1 
=    − (1 − r 2 ) dr sin dd
0 0  0 (1 − r ) 
2

 
1

2 2
r 1 
=   sin −1 r −  1 − r 2 + sin −1 r  sindd
0 0 2 2  0

Page | 143
 

 1
2 2
=    2 − 2 2  sindd
0 0


 2 
  2

=    sind  d
40 0
 
 



 2
=
4  (− cos ) 02 d
0

 2
  2
=
4  d =
0
4 2
( − 0) =
8
.

2. Evaluate  ( x + y 2 + z 2 )dxdydz taken over the volume enclose by the sphere
2

x 2 + y 2 + z 2 = 1 by transforming into spherical polar coordinates.

Sol: Changing to spherical polar coordinates by putting

x = r sin  cos
y = r sin  sin 
z = r cos

We have J = r2sin  , x 2 + y 2 + z 2 = r 2 & dxdydz = r 2 sindrdd.


2  1

 ( x + y + z )dxdydz =    r (r 2 sin drdd )


2
Now 2 2 2

=0 =0 r =0

2 
r5
=  =0 =0 5 sindd
2
1
= 
5 0
(− cos ) 0 d

2
2 2
=  d = ( ) 02
5 0 5
4
=
5

Page | 144
3. Using cylindrical coordinates, find the volume of cylinder with base radius a and height h.

Sol: The region of integration is bounded by x 2 + y 2  a 2 ,0  z  h .

The same region in cylindrical coordinates will be as follows:

r : 0 → a,  : 0 → 2 , z : 0 → h

h 2 a
 Required volume =  dxdydz =    rdrddz
z =0 =0 r =0

h 2
r2 a
=  
z =0  =0
(
2
) 0 ddz

h
a2
 ( )
2
= 0 dz
2 z =0

h
a2
= (2 )  dz
2 z =0

= a 2 ( z ) 0h

= a 2 h .

4. Find the volume of the sphere 𝑥 2 + 𝑦 2 + 𝑧 2 = 𝑎2

Sol: The sphere 𝑥 2 + 𝑦 2 + 𝑧 2 = 𝑎2 is cut into 8 equal parts by three co-ordinate planes.
Hence the volume of the sphere is equal to 8 times the volume of the solid bounded by
x=0, y=0, z=0 and 𝑥 2 + 𝑦 2 + 𝑧 2 = 𝑎2 .
Z-varies from 0 to √𝑎2 − 𝑥 2 − 𝑦 2

Y-varies from 0 to √𝑎2 − 𝑥 2


X-varies from 0 to a
𝑎 √𝑎2 −𝑥 2 √𝑎2 −𝑥 2 −𝑦 2
Required volume V= 8∫0 ∫0 ∫0 𝑑𝑧𝑑𝑦𝑑𝑥
𝑎 √𝑎2 −𝑥 2 √𝑎 2 −𝑥 2 −𝑦 2
= 8∫0 ∫0 [𝑧]0 𝑑𝑦𝑑𝑥
𝑎 √𝑎2 −𝑥 2
= 8∫0 ∫0 √𝑎2 − 𝑥 2 − 𝑦 2 𝑑𝑦𝑑𝑥
𝑎 𝑦 (𝑎2 −𝑥 2 ) 𝑦 2 −𝑥 2
= 8∫0 [ √𝑎2 − 𝑥 2 − 𝑦 2 + sin−1 (√𝑎2 )]√𝑎
0 𝑑𝑥
2 2 −𝑥 2

Page | 145
𝑎 (𝑎2 −𝑥 2 ) 𝜋
= 8∫0 𝑑𝑥
2 2
𝑎
= 2𝜋 ∫0 (𝑎2 − 𝑥 2 )𝑑𝑥
𝑥3 𝑎
= 2𝜋[𝑎2 𝑥 − ]
3 0

4𝜋𝑎3
=
3

Page | 146
UNIT -V
Special Functions

Page | 147
Beta Function:
1
The definite integral ∫0 𝑥 𝑚−1 (1 − 𝑥)𝑛−1 𝑑𝑥 is called the Beta function
and is denoted by 𝛽(𝑚, 𝑛) and read as “Beta of m,n”.The above integral converges
for 𝑚 > 0, 𝑛 > 0.
1
Thus 𝛽(𝑚, 𝑛) = ∫0 𝑥 𝑚−1 (1 − 𝑥)𝑛−1 𝑑𝑥 where 𝑚 > 0, 𝑛 > 0
❖ Beta function is also called Eulerian integral of the first kind.
Properties of Beta Function:
1. Symmetry of Beta function: i.e., 𝛽(𝑚, 𝑛) = 𝛽(𝑛, 𝑚)

Proof: By the def , we have


1
𝛽(𝑚, 𝑛) = ∫0 𝑥 𝑚−1 (1 − 𝑥)𝑛−1 𝑑𝑥

Put (1 − 𝑥) = 𝑦 ⟹ 𝑥 = (1 − 𝑦) so 𝑑𝑥 = −𝑑𝑦

limits: when 𝑥 = 0 ⟹ 𝑦 = 1

𝑥=1⟹𝑦=0
1
∴ 𝛽(𝑚, 𝑛) = ∫0 𝑥 𝑚−1 (1 − 𝑥)𝑛−1 𝑑𝑥

0
= ∫1 (1 − 𝑦)𝑚−1 𝑦 𝑛−1 (−𝑑𝑦)

0
= − ∫1 𝑦 𝑛−1 (1 − 𝑦)𝑚−1 𝑑𝑦

1 𝑏 𝑎
= ∫0 𝑦 𝑛−1 (1 − 𝑦)𝑚−1 𝑑𝑦 ∴ ∫𝑎 𝑓(𝑥)𝑑𝑥 = − ∫𝑏 𝑓(𝑥)𝑑𝑥

1 𝑏 𝑏
= ∫0 𝑥 𝑛−1 (1 − 𝑥)𝑚−1 𝑑𝑥 ∴ ∫𝑎 𝑓(𝑥)𝑑𝑥 = ∫𝑎 𝑓(𝑦)𝑑𝑦

= 𝛽(𝑛, 𝑚)

∴ 𝛽(𝑚, 𝑛) = 𝛽(𝑛, 𝑚)

Alternate Proof:
1
We know that 𝛽(𝑚, 𝑛) = ∫0 𝑥 𝑚−1 (1 − 𝑥)𝑛−1 𝑑𝑥

Page | 148
From the properties of definite integrals, we have
𝑎 𝑎
∫0 𝑓(𝑥)𝑑𝑥 = ∫0 𝑓(𝑎 − 𝑥)𝑑𝑥
1
∴ 𝛽(𝑚, 𝑛) = ∫0 (1 − 𝑥)𝑚−1 (1 − (1 − 𝑥))𝑛−1 𝑑𝑥
1
= ∫0 𝑥 𝑛−1 (1 − 𝑥)𝑚−1 𝑑𝑥
= 𝛽(𝑛, 𝑚)
∴ 𝛽(𝑚, 𝑛) = 𝛽(𝑛, 𝑚)

𝜋
2. Prove that 𝛽(𝑚, 𝑛) = 2 ∫0 𝑠𝑖𝑛2𝑚−1 𝜃𝑐𝑜𝑠 2𝑛−1 𝜃 𝑑𝜃
2

Proof: By the def , we have


1
𝛽(𝑚, 𝑛) = ∫0 𝑥 𝑚−1 (1 − 𝑥)𝑛−1 𝑑𝑥

Put 𝑥 = 𝑠𝑖𝑛2 𝜃 ⟹ 𝑑𝑥 = 2𝑠𝑖𝑛𝜃𝑐𝑜𝑠𝜃𝑑𝜃

when 𝑥 = 0 ⟹ 𝜃 = 0
𝜋
𝑥=1⟹𝜃=
2
𝜋
∴ 𝛽 (𝑚, 𝑛) = ∫02 (𝑠𝑖𝑛2 𝜃)𝑚−1 (1 − 𝑠𝑖𝑛2 𝜃)𝑛−1 2𝑠𝑖𝑛𝜃𝑐𝑜𝑠𝜃𝑑𝜃
𝜋
= 2 ∫0 (𝑠𝑖𝑛2 𝜃)𝑚−1 (𝑐𝑜𝑠 2 𝜃)𝑛−1 𝑠𝑖𝑛𝜃𝑐𝑜𝑠𝜃𝑑𝜃
2

𝜋
= 2 ∫02 𝑠𝑖𝑛2𝑚−2 𝜃 𝑐𝑜𝑠 2𝑛−2 𝜃 𝑠𝑖𝑛𝜃𝑐𝑜𝑠𝜃𝑑𝜃
𝜋
= 2 ∫02 𝑠𝑖𝑛2𝑚−1 𝜃𝑐𝑜𝑠 2𝑛−1 𝜃 𝑑𝜃
𝜋
∴ 𝛽 (𝑚, 𝑛) = 2 ∫02 𝑠𝑖𝑛2𝑚−1 𝜃𝑐𝑜𝑠2𝑛−1 𝜃 𝑑𝜃
𝜋
1
Note: ∫0 𝑠𝑖𝑛2𝑚−1 𝜃𝑐𝑜𝑠 2𝑛−1 𝜃 𝑑𝜃 = 𝛽(𝑚, 𝑛)
2
2

Page | 149
3. Prove that 𝛽(𝑚, 𝑛) = 𝛽(𝑚 + 1, 𝑛) + 𝛽(𝑚, 𝑛 + 1)

Proof: By the def , we have


1
𝛽(𝑚, 𝑛) = ∫0 𝑥 𝑚−1 (1 − 𝑥)𝑛−1 𝑑𝑥

1
𝛽(𝑚 + 1, 𝑛) = ∫0 𝑥 𝑚 (1 − 𝑥)𝑛−1 𝑑𝑥

1
𝛽(𝑚, 𝑛 + 1) = ∫0 𝑥 𝑚−1 (1 − 𝑥)𝑛 𝑑𝑥

1 1
Now 𝛽(𝑚 + 1, 𝑛) + 𝛽(𝑚, 𝑛 + 1) = ∫0 𝑥 𝑚 (1 − 𝑥)𝑛−1 𝑑𝑥 + ∫0 𝑥 𝑚−1 (1 − 𝑥)𝑛 𝑑𝑥

1
= ∫0 [𝑥 𝑚 (1 − 𝑥)𝑛−1 + 𝑥 𝑚−1 (1 − 𝑥)𝑛 ]𝑑𝑥

1
= ∫0 𝑥 𝑚−1 (1 − 𝑥)𝑛−1 [ 𝑥 + (1 − 𝑥)]𝑑𝑥

1
= ∫0 𝑥 𝑚−1 (1 − 𝑥)𝑛−1 𝑑𝑥

= 𝛽(𝑚, 𝑛)

Hence 𝛽(𝑚, 𝑛) = 𝛽(𝑚 + 1, 𝑛) + 𝛽(𝑚, 𝑛 + 1)


(𝑚−1)!(𝑛−1)!
4. If m and n are positive integers then 𝛽(𝑚, 𝑛) =
(𝑚+𝑛−1)!

Proof: By the def , we have


1
𝛽(𝑚, 𝑛) = ∫0 𝑥 𝑚−1 (1 − 𝑥)𝑛−1 𝑑𝑥 -----------------(1)

(1−𝑥)𝑛 1 1 (1−𝑥)𝑛
= [𝑥 𝑚−1 ] − ∫0 (𝑚 − 1)𝑥 𝑚−2 𝑑𝑥 (By integration by parts)
𝑛(−1) 0 𝑛(−1)

(𝑚−1) 1
=
𝑛
∫0 𝑥 𝑚−2 (1 − 𝑥)𝑛 𝑑𝑥

(𝑚−1)
𝛽(𝑚, 𝑛) = 𝛽(𝑚 − 1, 𝑛 + 1) ------------------(2)
𝑛

Now we have to find 𝛽(𝑚 − 1, 𝑛 + 1)

To obtain this put 𝑚 = 𝑚 − 1 and 𝑛 = 𝑛 + 1 in equation (2), we have

Page | 150
(𝑚−2)
𝛽(𝑚 − 1, 𝑛 + 1) = 𝛽(𝑚 − 2, 𝑛 + 2)
(𝑛+1)

From equation (2)


(𝑚−1) (𝑚−2)
𝛽 (𝑚, 𝑛) = . 𝛽 (𝑚 − 2, 𝑛 + 2) -------------------(3)
𝑛 (𝑛+1)

Changing 𝑚 = 𝑚 − 2 and 𝑛 = 𝑛 + 2 in equation (2), we have


(𝑚−3)
𝛽 (𝑚 − 2, 𝑛 + 2) = 𝛽(𝑚 − 2, 𝑛 + 2)
(𝑛+1)

From equation (3)


(𝑚−1) (𝑚−2) (𝑚−3)
𝛽 (𝑚, 𝑛) = . . 𝛽 (𝑚 − 2, 𝑛 + 2) ----------------(4)
𝑛 (𝑛+1) (𝑛+1)

Proceeding like this, we get


(𝑚−1) (𝑚−2) (𝑚−3) (𝑚−(𝑚−1))
𝛽(𝑚, 𝑛) = . . ………. 𝛽(𝑚 − (𝑚 − 1), 𝑛 + (𝑚 − 1))
𝑛 (𝑛+1) (𝑛+1) (𝑛+𝑚−2)

(𝑚−1)(𝑚−2)(𝑚−3)…………1
𝛽(𝑚, 𝑛) = 𝛽(1, 𝑛 + 𝑚 − 1)-------------------(5)
𝑛(𝑛+1)(𝑛+2)………….(𝑛+𝑚−2)

1 1
𝛽(1, 𝑛 + 𝑚 − 1) = ∫0 𝑥 0 (1 − 𝑥)𝑛+𝑚−2 𝑑𝑥 = ∫0 (1 − 𝑥)𝑛+𝑚−2 𝑑𝑥

(1−𝑥)𝑛+𝑚−1 1 1
=[ ] =
(𝑛+𝑚−1)(−1) 0 𝑛+𝑚−1

From equation (5), we have


(𝑚−1)(𝑚−2)(𝑚−3)…………1
𝛽(𝑚, 𝑛) =
𝑛(𝑛+1)(𝑛+2)………….(𝑛+𝑚−2)(𝑛+𝑚−1)

(𝑚−1)!
𝛽(𝑚, 𝑛) = (𝑛+𝑚−1)(𝑛+𝑚−2)…..(𝑛+2)(𝑛+1)𝑛

Multiplying the numerator and denominator by (𝑛 − 1)!, we have

(𝑚−1)!(𝑛−1)! (𝑚−1)!(𝑛−1)!
𝛽(𝑚, 𝑛) = (𝑛+𝑚−1)(𝑛+𝑚−2)…..(𝑛+2)(𝑛+1)𝑛 =
(𝑛−1)! (𝑚+𝑛−1)!

(𝑚−1)!(𝑛−1)!
∴ 𝛽(𝑚, 𝑛) =
(𝑚+𝑛−1)!

Page | 151
Other forms of Beta Function:
∞ 𝑥 𝑚−1 ∞ 𝑥 𝑛−1 ∞ 𝑦 𝑛−1
1. Show that 𝛽(𝑚, 𝑛) = ∫0 𝑚+𝑛
𝑑𝑥 = ∫0 𝑚+𝑛
𝑑𝑥 (or) ∫0 𝑑𝑦
(1+𝑥) (1+𝑥) (1+𝑦)𝑚+𝑛

Proof: By the def , we have


1
𝛽(𝑚, 𝑛) = ∫0 𝑥 𝑚−1 (1 − 𝑥)𝑛−1 𝑑𝑥 -------------------(1)

1 −1
Put 𝑥 = so that 𝑑𝑥 = 𝑑𝑦
1+𝑦 (1+𝑦)2

when 𝑥 = 0 ⟹ 𝑦 → ∞

𝑥=1⟹𝑦=0

From equation (1), we have


0 1 1 (−1)
𝛽(𝑚, 𝑛) = ∫∞( )𝑚−1 (1 − )𝑛−1 𝑑𝑦
1+𝑦 1+𝑦 (1+𝑦)2

0 1 𝑦 𝑛−1 (−1)
𝛽(𝑚, 𝑛) = ∫∞ 𝑚−1
. . 𝑑𝑦
(1+𝑦) (1+𝑦)𝑛−1 (1+𝑦)2

∞ 𝑦 𝑛−1
𝛽(𝑚, 𝑛) = ∫0 𝑑𝑦
(1+𝑦)𝑚+𝑛

∞ 𝑥 𝑛−1
∴ 𝛽(𝑚, 𝑛) = ∫0 𝑑𝑥
(1+𝑥)𝑚+𝑛

Since Beta function is symmetrical in m and n, we also have

∞ 𝑥 𝑚−1
𝛽(𝑚, 𝑛) = ∫0 𝑑𝑥
(1+𝑥)𝑚+𝑛

∞ 𝑥 𝑚−1 ∞ 𝑥 𝑛−1
Hence 𝛽(𝑚, 𝑛) = ∫0 𝑚+𝑛
𝑑𝑥 = ∫0 𝑑𝑥
(1+𝑥) (1+𝑥)𝑚+𝑛

1 𝑥 𝑚−1 +𝑥 𝑛−1
2.Show that 𝛽(𝑚, 𝑛) = ∫0 𝑑𝑥
(1+𝑥)𝑚+𝑛

∞ 𝑥 𝑚−1
Proof: We have 𝛽(𝑚, 𝑛) = ∫0 𝑑𝑥
(1+𝑥)𝑚+𝑛

1 𝑥 𝑚−1 ∞ 𝑥 𝑚−1
= ∫0 𝑚+𝑛
𝑑𝑥 + ∫1 𝑑𝑥 --------------(1)
(1+𝑥) (1+𝑥)𝑚+𝑛

∞ 𝑥 𝑚−1
Now consider ∫1 𝑑𝑥
(1+𝑥)𝑚+𝑛

Page | 152
1 −1
Put 𝑥 = so that 𝑑𝑥 = 𝑑𝑦
𝑦 𝑦2

when 𝑥 = 1 ⟹ 𝑦 = 1

𝑥→∞⟹𝑦=0
𝑚−11
∞ 𝑥 𝑚−1 0 (𝑦) −1
∴ ∫1 (1+𝑥)𝑚+𝑛 𝑑𝑥 = ∫1 1 𝑚+𝑛 . 2 𝑑𝑦
(1+ ) 𝑦
𝑦

1
1 𝑦𝑚−1 1
= ∫0 (1+𝑦)𝑚+𝑛 . 𝑦 2 𝑑𝑦
𝑦𝑚+𝑛

1 𝑦 𝑛−1 1 𝑥 𝑛−1
= ∫0 𝑑𝑦 = ∫0 𝑑𝑥
(1+𝑦)𝑚+𝑛 (1+𝑥)𝑚+𝑛

Hence equation (1) becomes

1 𝑥 𝑚−1 1 𝑥 𝑛−1
𝛽(𝑚, 𝑛) = ∫0 𝑚+𝑛
𝑑𝑥 + ∫0 𝑑𝑥
(1+𝑥) (1+𝑥)𝑚+𝑛

1 𝑥 𝑚−1 +𝑥 𝑛−1
∴ 𝛽(𝑚, 𝑛) = ∫0 𝑑𝑥
(1+𝑥)𝑚+𝑛

∞ 𝑥 𝑚−1
3.Show that 𝛽(𝑚, 𝑛) = 𝑎𝑚 𝑏 𝑛 ∫0 𝑑𝑥
(𝑎𝑥+𝑏)𝑚+𝑛

∞ 𝑥 𝑚−1 ∞ 𝑥 𝑚−1
Proof: We have, 𝑎𝑚 𝑏 𝑛 ∫0 𝑚+𝑛
𝑑𝑥 = 𝑎𝑚 𝑏 𝑛 ∫0 𝑎 𝑑𝑥
(𝑎𝑥+𝑏) 𝑏𝑚+𝑛 ( 𝑥+1)𝑚+𝑛
𝑏

𝑎𝑥 𝑏𝑦 𝑏
Put 𝑦 = ⟹𝑥= so that 𝑑𝑥 = 𝑑𝑦
𝑏 𝑎 𝑎

when 𝑥 = 0 ⟹ 𝑦 = 0

𝑥=∞⟹𝑦=∞
𝑚−1𝑏𝑦
𝑚 𝑛 ∞ 𝑥 𝑚−1 𝑎𝑚 𝑏𝑛 ∞ (𝑎) 𝑏
∴𝑎 𝑏 ∫0 (𝑎𝑥+𝑏)𝑚+𝑛 𝑑𝑥 = 𝑚+𝑛 ∫0 𝑎 𝑏𝑦 . 𝑑𝑦
𝑏 ( . +1) 𝑚+𝑛 𝑎
𝑏 𝑎

𝑏𝑚 ∞ 𝑦 𝑚−1
= 𝑎𝑚 𝑏 −𝑚 ∫ 𝑑𝑦
𝑎𝑚 0 (1+𝑦)𝑚+𝑛

∞ 𝑦 𝑚−1
= ∫0 𝑑𝑦
(1+𝑦)𝑚+𝑛

∞ 𝑥 𝑚−1 ∞ 𝑦 𝑛−1
We know that 𝛽(𝑚, 𝑛) = ∫0 𝑚+𝑛
𝑑𝑥 = ∫0 𝑑𝑦
(1+𝑥) (1+𝑦)𝑚+𝑛

∞ 𝑥 𝑚−1 ∞ 𝑦 𝑚−1
Hence 𝑎𝑚 𝑏 𝑛 ∫0 𝑚+𝑛
𝑑𝑥 = ∫0 𝑑𝑦 = 𝛽(𝑚, 𝑛)
(𝑎𝑥+𝑏) (1+𝑦)𝑚+𝑛
Page | 153
∞ 𝑥 𝑚−1
∴ 𝛽(𝑚, 𝑛) = 𝑎𝑚 𝑏 𝑛 ∫0 𝑑𝑥
(𝑎𝑥+𝑏)𝑚+𝑛

1 𝑥 𝑚−1 (1−𝑥)𝑛−1 𝛽(𝑚,𝑛)


4.Show that ∫0 𝑑𝑥 =
(𝑥+𝑎)𝑚+𝑛 𝑎𝑛 (1+𝑎)𝑚

Proof: By the definition , we have


1
𝛽(𝑚, 𝑛) = ∫0 𝑥 𝑚−1 (1 − 𝑥)𝑛−1 𝑑𝑥 -------------------(1)

(1+𝑎)𝑦 𝑎(1+𝑎)
Put 𝑥 = so that 𝑑𝑥 = 𝑑𝑦
(𝑦+𝑎) (𝑦+𝑎)2

when 𝑥 = 0 ⟹ 𝑦 = 0

𝑥=1⟹𝑦=1

Now equation (1) becomes

1 (1+𝑎)𝑚−1 𝑦 𝑚−1 (1+𝑎)𝑦 𝑛−1 𝑎(1+𝑎)


𝛽(𝑚, 𝑛) = ∫0 . (1 − ) . 𝑑𝑦
(𝑦+𝑎)𝑚−1 (𝑦+𝑎) (𝑦+𝑎)2

𝑛−1
1 𝑥𝑚−1 (1−𝑥) 𝛽(𝑚,𝑛)
∴ ∫0 𝑚+𝑛 𝑑𝑥 = 𝑚
(𝑥+𝑎) 𝑎𝑛 (1+𝑎)

𝑏
5. Show that ∫𝑎 (𝑥 − 𝑏)𝑚−1 (𝑎 − 𝑥)𝑛−1 𝑑𝑥 = (𝑎 − 𝑏)𝑚+𝑛−1 𝛽(𝑚, 𝑛), 𝑚 > 0, 𝑛 > 0.

Proof: By the definition


1
𝛽(𝑚, 𝑛) = ∫0 𝑥 𝑚−1 (1 − 𝑥)𝑛−1 𝑑𝑥

𝑦−𝑏 1
Put 𝑥 = so that 𝑑𝑥 = 𝑑𝑦
𝑎−𝑏 𝑎−𝑏

Page | 154
when 𝑥 = 0 ⟹ 𝑦 = 𝑏

𝑥=1⟹𝑦=𝑎
𝑏 𝑦−𝑏 𝑚−1 𝑦−𝑏 1
∴ 𝛽(𝑚, 𝑛) = ∫𝑎 ( ) (1 −( ))𝑛−1 . 𝑑𝑦
𝑎−𝑏 𝑎−𝑏 𝑎−𝑏

𝑏 (𝑦−𝑏)𝑚−1 (𝑎−𝑦)𝑛−1 1
= ∫𝑎 . . 𝑑𝑦
(𝑎−𝑏)𝑚−1 (𝑎−𝑏)𝑛−1 𝑎−𝑏

𝑏 (𝑦−𝑏)𝑚−1 (𝑎−𝑦)𝑛−1
= ∫𝑎 𝑑𝑦
(𝑎−𝑏)𝑚+𝑛−1

𝑏 (𝑥−𝑏)𝑚−1 (𝑎−𝑥)𝑛−1
= ∫𝑎 𝑑𝑥
(𝑎−𝑏)𝑚+𝑛−1

𝑏 (𝑥−𝑏)𝑚−1 (𝑎−𝑥)𝑛−1
∫𝑎 𝑑𝑥 = 𝛽(𝑚, 𝑛)
(𝑎−𝑏)𝑚+𝑛−1

𝑏
Hence ∫𝑎 (𝑥 − 𝑏)𝑚−1 (𝑎 − 𝑥)𝑛−1 𝑑𝑥 = (𝑎 − 𝑏)𝑚+𝑛−1 𝛽(𝑚, 𝑛)


2
1 m +1 n +1
 sin  cos d =  (
m n
6. Show that , )
0
2 2 2

 
2 2

 sin  cos d =  sin m−1 cosn−1  (sin cos )d


m n
Sol:
0 0


2 m −1 n −1

=  (sin 2 ) 2
(cos2  ) 2
(sin cos )d
0

dx
Put sin 2  = x so that sin  cosd =
2

2 1 m −1 n −1
1
 sin  cos d = 2 0
m n
x 2
(1 − x) 2
dx
0

1 m +1 n +1
1 ( ) −1 ( ) −1

2 0
= x 2
(1 − x) 2
dx

1 m +1 n +1
= ( , )
2 2 2

Aliter:

Page | 155

2
1
 sin  cos2 n −1 d =  (m, n) → (1)
2 m −1
We have
0
2

Put p= 2m-1, q = 2n-1


( p + 1) (q + 1)
m= ,n =
2 2

From Eq (1)

2
1 p +1 q +1
0    = (
p q
sin cos d , )
2 2 2

Or

2
1 m +1 n +1
 sin  cos d = 2  (
m n
, )
0
2 2

Hence the Result.

Problems:
1
x
1. Express the following integral in terms of Beta function 
0 1− x2
dx

1
x
Sol: Given  0 1− x2
dx

put x 2 = y

−1
dy 1 2
 dx = = y dy
2x 2

When x= 0, y = 0;

When x = 1, y = 1.
1
1 1 2 −1
x y1 2

0 1− x2
dx = 
0 1− y 2
y dy

1 −1 1 1
1 1 −1 1 1
=  y (1 − y ) 2 dy =  y 1−1 (1 − y ) 2 dy =  (1, ) .
20 20 2 2

Page | 156
3
dx
2. Express the following integral in terms of Beta function 
0 9 − x2

3
dx
Sol: Given 
0 9 − x2

Put x 2 = 9 y

−1
3 2
 dx = y dy
2

When x= 0, y = 0;

When x = 3, y = 1.
3 3 −1 1 1 −1
dx − 3 2
 =  (9 − x ) dx =  (9 − 9 y )
2 2 2
y dy
0 9 − x2 0 0
2

1 1 −1
3 − 1
=
20 (1 − y ) 2 y 2 dy
3

1 1 1
1 −1 −1
=
20 (1 − y ) 2 y 2 dy

1 1 1
= ( , ).
2 2 2

1 m +1
1
3. Show that x (1 − x n ) p dx = ( , p + 1)
m

0
n n

1
Sol: Given that  x m (1 − x n ) p dx
0

Put x = y
n

1
1 n −1
 dx = y dy
n

When x= 0, y = 0;

When x = 1, y = 1.
1 1 m 1− n 1 m +1− n
1 1
0 x (1 − x ) dx = 0 y (1 − y) n y dy =  y (1 − y ) p dy
m n p n p n n
n0

Page | 157
1 m +1
1 −1
= y n
(1 − y ) ( p +1) −1 dy
n0

1 m +1
= ( , p + 1)
n n

 (1 + x) (1 − x) n−1 dx = 2 m+ n−1  (m, n)


m −1
4. Show that
−1

Sol: we have  (m, n) =  x (1 − x) dx


m −1 n −1

1+ y 1
Put x =  dx = dy
2 2

When x= 0, y = -1;

When x= 1, y = 1

(1 + y) m−1 1 + y n−1 1
1
 (m, n) =  m −1
(1 − ) dy
−1 2 2 2

(1 + y) m−1 (1 − y) n−1
1

= dy
−1 2 m+n−1

1
1
 (1 + x)
m −1
= m + n −1
(1 − x) n−1 dx
2 −1

1
  (1 + x) m−1 (1 − x) n−1 dx = 2 m + n −1  (m, n) .
−1

1
xdx 1 2 1
5. Prove that  = ( , )
0 1 − x5 5 5 2

1
xdx
Sol: Given that 
0 1 − x5

Put x = yx= y
5 5

1 −4
1 −1 1
 dx = y 5 dy = y 5 dy
5 5

Also x= 0, y =0; when x= 1,y =1

Page | 158
1
1 1 5 −4
xdx y1
 =  y 5
dy
0 1 − x5 0 1− y 5

1 −3 −1
1
=  y 5 (1 − y ) 2 dy
50

1 2 1
1 −1 −1
=  y 5 (1 − y ) 2 dy
50

1 2 1
= ( , ) .
5 5 2

1
x 2 dx
6. Evaluate 
0 1 − x5
in terms of Beta function.

1 1 −1
x 2 dx
Sol: Given  = x (1 − x ) dx
2 5 2

0 1 − x5 0

1 −1
x2
=  4 (1 − x 5 ) 2 x 4 dx
0 x

1 −1

=  x −2 (1 − x 5 ) 2 x 4 dx
0

Put x 5 = y  5x 4 dx = dy

dy
 x 4 dx =
5

Also x= 0, y =0; when x= 1,y =1


1 1 −2 −1
x 2 dx dy
 =  y (1 − y) 5 2

0 1 − x5 0
5

1 3 1
1 −1 −1
=  y 5 (1 − y ) 2 dy
50

1 3 1
= ( , ) .
5 5 2
b
7. Show that  ( x − a) (b − x) n dx = (b − a) m + n +1  (m + 1, n + 1)
m

b
Sol: Given that  ( x − a) (b − x) n dx
m

Page | 159
t−a
Put x = a+ (b-a) y ( or x = ) so that dx = (b-a)dy
b−a

Also when x = a, y = 0 and x = b, y = 1


b 1

 ( x − a) (b − x) dx =  [(b − a) y ] m [b − a − (b − a) y ]n (b − a)dy
m n

a 0

1
=  (b − a) m y m (b − a) n (1 − y ) n (b − a)dy
0

1
= (b − a) m+ n+1  y m (1 − y ) n dy
0

1
= (b − a) m+ n+1  y ( m+1)−1 (1 − y ) ( n+1)−1 dy
0

= (b − a)
m + n +1
 (m + 1, n + 1) .

x m −1 dx
0 ( x + a) m+ n = a  (m, n)
−n
8. Show that


x m−1 dx
Sol: we have  (m, n) =  m+n
0 (1 + x )

t dt
Put x =  dx =
a a

Also when x = 0, t= 0 and x =∞, t = ∞


  
t m−1 dt 1 t m−1a m+ n t m−1
 ( m, n ) = 
a a m 0 (a + t ) m+ n
= dt = a 
n
m+ n
dt
0 (t + a )
t
0 a m −1 (1 + ) m + n
a

1 x m−1
 (m, n) =  m+ n
dx
an 0 ( x + a )


x m −1dx
Hence  m+n
= a −n  (m, n)
0 ( x + a)

Gamma Function:

The definite integral ∫0 𝑒 −𝑥 𝑥 𝑛−1 𝑑𝑥 is called the Gamma function and it’s
denoted by Γ(𝑛) and read as “Gamma of n”.The integral converges only for 𝑛 > 0.
Page | 160

Thus, Γ(𝑛) = ∫0 𝑒 −𝑥 𝑥 𝑛−1 𝑑𝑥 where 𝑛 > 0

Gamma function is also called Eulerian integral of the second kind.



Note: The integral ∫0 𝑒 −𝑥 𝑥 𝑛−1 𝑑𝑥 does not converge if 𝑛 ≤ 0

Properties of Gamma function:


1. Show that Γ(1) = 1

Sol: By the def of Gamma function, we have



Γ(𝑛) = ∫0 𝑒 −𝑥 𝑥 𝑛−1 𝑑𝑥
∞ ∞ ∞
∴ Γ(1) = ∫0 𝑒 −𝑥 𝑥 1−1 𝑑𝑥 = ∫0 𝑒 −𝑥 𝑥 0 𝑑𝑥 = ∫0 𝑒 −𝑥 𝑑𝑥

𝑒 −𝑥 ∞
=[ ] = −[𝑒 −∞ − 𝑒 −0 ] = −[0 − 1] = 1
−1 0

∴ Γ(1) = 1

2. Show that Γ(𝑛) = (𝑛 − 1)Γ(𝑛 − 1) where 𝑛 > 1

Sol: By the def of Gamma function, we have



Γ(𝑛) = ∫0 𝑒 −𝑥 𝑥 𝑛−1 𝑑𝑥

𝑒 −𝑥 ∞ ∞ 𝑒 −𝑥
= [𝑥 𝑛−1 ] − ∫0 (𝑛 − 1)𝑥 𝑛−2 ( )𝑑𝑥 (Integrate by parts)
−1 0 −1

= (𝑛 − 1) ∫0 𝑒 −𝑥 𝑥 𝑛−2 𝑑𝑥

= (𝑛 − 1)Γ(𝑛 − 1)

∴ Γ(𝑛) = (𝑛 − 1)Γ(𝑛 − 1)

Note: 1. Γ(𝑛 + 1) = 𝑛Γ(𝑛)

2. If n is a positive fraction then we can write

Γ(𝑛) = (𝑛 − 1)(𝑛 − 2) … … … (𝑛 − 𝑟)Γ(𝑛 − 𝑟) Where (𝑛 − 𝑟) > 0

3. If n is non negative integer then Γ(𝑛 + 1) = 𝑛!

Sol: From property 2, we have

Γ(𝑛 + 1) = 𝑛Γ(𝑛) = 𝑛(𝑛 − 1)Γ(𝑛 − 1) (Again by property 2)


Page | 161
= 𝑛(𝑛 − 1)(𝑛 − 2)Γ(𝑛 − 2) (Again by property 2)

= 𝑛(𝑛 − 1)(𝑛 − 2)(𝑛 − 3)Γ(𝑛 − 3)

= 𝑛(𝑛 − 1)(𝑛 − 2)(𝑛 − 3) … … … 3.2.1 Γ(1)

= 𝑛(𝑛 − 1)(𝑛 − 2)(𝑛 − 3) … … … 3.2.1

= 𝑛!

∴ Γ(𝑛 + 1) = 𝑛!

Problems:
9
1. Find Γ ( )
2

9 9 9 7 7 7 7 7
Sol: Γ ( ) = ( − 1) Γ ( − 1) = Γ ( ) = ( − 1) Γ ( − 1)
2 2 2 2 2 2 2 2

7 5 5 7 5 5 5
= . Γ ( ) = . . ( − 1) Γ ( − 1)
2 2 2 2 2 2 2

7 5 3 3 7 5 3 3 3
= . . Γ ( ) = . . ( − 1) Γ ( − 1)
2 2 2 2 2 2 2 2 2

7 5 3 1 1
= . . . Γ( )
2 2 2 2 2

7 5 3 1 1
= . . . √π [ Γ ( ) = √π ]
2 2 2 2 2

9 105
Γ( ) = √π
2 16

13
2. Solve Find Γ ( )
3

13 10 7 4 1 1
Sol: Γ ( ) =. . . . Γ( )
3 3 3 3 3 3

Note: When n is a negative fraction

We have Γ(𝑛 + 1) = 𝑛Γ(𝑛)

Γ(𝑛+1)
Γ(𝑛) =
n

1
3. Compute Γ (− )
2

Γ(𝑛+1)
Sol: We know that Γ(𝑛) =
n

1
Put 𝑛 = −
2

Page | 162
1
1 Γ(−2+1)
Γ (− ) = 1
2 −2

1
1 Γ(2)
Γ (− ) = 1
2 −2

1 √π
Γ (− ) = 1
2 −2

1
∴ Γ (− ) = −2√π
2

5
4. Compute Γ (− )
2

Γ(𝑛+1)
Sol: We know that Γ(𝑛) =
n

5
5 Γ(−2+1) 2 3
Γ (− ) =
2 5 = − 5 Γ (− 2)
−2

3
2 Γ(−2+1) 4 1
=− 3 = Γ (− )
5 − 15 2
2

1
4 Γ(−2+1) 8 1 8
= 1 =− Γ( ) = − √π
15 −2 15 2 15

5 −8√π
∴ Γ (− ) =
2 15

Relation between  & functions:


 ( m)  ( n )
Prove that  (m, n) =
( m + n)

Proof: By the definition of  - function



(m) =  e − x x m −1 dx
0

Put x = t 2  dx = 2tdt

When x = 0, t = 0 & x =  , t = 
 
(m) =  e (t )−t 2 2 m−1
2tdt =  e −t t 2 m−2+1 2dt
2

0 0
Page | 163

= 2 e −t t 2 m−1dt
2


 (m) = 2 e − x x 2 m −1 dx
2


Similarly (n) = 2 e − y y 2 n−1dy
2


 (m)(n) = 4 e −( x + y2 )
x 2 m−1 y 2 n−1dxdy
2

0 0

Transform it into polar coordinates

x= rcos  ,y = rsin  , dxdy = rdrd 



r varies from 0 to , varies from 0 to
2


2
 (m)(n) = 4  e −r r 2( m+ n)−1 cos2 m−1  sin 2 n−1 drd
2

0 0


 2
= 2 e −r r 2( m+n )−1dr. 2  cos2 m−1  sin 2 n−1  d
2

0 0


= (m + n). (m, n)  (m) = 2 e − x x 2 m −1 dx
2

 ( m) ( n )
  (m, n) =
( m + n)

Problems:
1
1. Show that ( ) = 
2

 ( m)  ( n )
Proof: We know that  (m, n) =
 ( m + n)

Taking m = n = ½, we have

Page | 164
1 1
 ( ) ( )
1 1 2 2 = (( 1 )) 2
( , ) = since (1) = 1
2 2 1 1 2 ………………….. (1)
( + )
2 2

1 1 1 1 1 1
1 1 −1 −1 − −
But  ( , )
2 2 0
=  x 2
(1 − x ) 2
dx = 
0
x 2
(1 − x ) 2
dx

Put x = sin 2   dx = 2 sin  cosd


Also when x =0,  = 0; when x =1,  =
2

2
1 1 1 1
( , ) =  2 sin cosd
2 2 0
sin cos



2

= 2  d = 2( ) 02 = 2( − 0) =  …………………….(2)
0
2

From (1) & (2) we have


1
[( )]2 = 
2

1
 ( ) = 
2


e
−x2
2. To show that dx =
0 2


Proof: We have (n) =  e = x x n−1dx
0

 −1
1
Taking n = ½, we have  ( ) =  e = x x 2 dx
2 0

Put x =t  dx = 2tdt
2

Also when x=0, t = 0; when x → , t → 


 −1 
1
 ( ) =  e −t (t 2 ) 2 2tdt = 2 e −t dt
2 2

2 0 0


 2 e − x dx = 
2

Page | 165


 e − x dx =
2

0
2

Note:
0

e
− x2
1. dx =
−
2

e dx = 
− x2
2.
−

3. (n) is defined when n > 0.


4. (n) is defined when n is a negative fraction but (n) is not defined when n
= 0 and n is a negative integer.


4. Show that (n)(1 − n) =
sin n

x n −1
Proof: We know that  (m, n) =  m+ n
dx (by one of the forms of Beta function)
0 (1 + x )

 ( m)  ( n )
Also we have  (m, n) =
 ( m + n)


x n −1  ( m ) ( n )
 dx =
0
(1 + x) m + n  ( m + n)

Taking m+n = 1 so that m = 1-n, we get



x n −1 (1 − n)(n)
0 (1 + x) dx = (1)


x n −1
(1 − n)(n) = 0 (1 + x) dx ………………………..(1)

x 2m  2m + 1
We have 0 1 + x 2n dx = 2n cos ec( 2n ) where m > 0, n > 0 and n > m

2m + 1
Put x 2 n = t & = s we have
2n

2m 1
 ( )
t t 2n 2n

0 2n(1 + t )t 2n cosecs
dt =

2m 1
 ( ) −1
1 t t 2n 2n


2n 0 (1 + t )
dt =
2n
cos ecs

Page | 166
 [( 2 m +1) / 2 n ]−1
t

0
(1 + t )
dt =  cos ecs


t s −1 
0 (1 + t )dt = sin s
 
x s −1  x n−1 
0 (1 + x) sin s 0 (1 + x) sin n
dx = or dx =
…………….(2)

From (1) & (2) we have


(n)(1 − n) =
sin n
1
1
5. show that (n) =  (log ) n−1 dx , n > 0
0
x


Sol: We have (n) =  e − x x n −1 dx → (1)
0

1 −x
Put x = log = − log y  y = e
y

1
 dy = −e − x dxordx = − dy
y

Also when x=0, y= 1; when x=∞, y=0

Now (1) becomes


0 1
1 1 1
(n) = −  (log ) n −1 y. dy =  (log ) n −1 dy
1
y y 0
y
1
1
(n) =  (log ) n −1 dx
0
x

x (1 − x) 2 dx
4
6. Evaluate
0

1 1

Sol: Given  x (1 − x) dx =  x 5−1 (1 − x) 3−1 dx


4 2

0 0

Page | 167
(5)(3)
=  (5,3) =
(8)
4!2!
=
7!
2
=
765
1
=
105
2 1

 x(8 − x ) 3 dx
3
7. Evaluate
0

2 1

Sol: Given that  x(8 − x ) dx 3 3

1 −2
2
Put x 3 = 8 y  x = 2 y 3  dx = y 3 dy
3

When x = 0, y = 0; When x = 2, y = 1
2 1 1 1 1 −2
2 3
 x(8 − x ) 3 dx =  2 y 3 (8 − 8 y) 3
3
y dy
0 0
3

1 1 1
8 −3
= 
30
y (1 − y ) 3 dy

1 2 4
8 3 −1 −1
= 
30
y (1 − y ) 3 dy

2 4 2 4 4
 ( ) ( ) ( )( − 1)( − 1)
8 2 4 8 3 3 8 3 3 3
= ( , ) = =  (n) = (n − 1)(n − 1)
3 3 3 3 2 4 3 (2)
( + )
3 3
81 2 1 8 1 1
= ( )( ) = ( )(1 − )
33 3 3 9 3 3
8  16 
= = [(n)(1 − n) = ]
9  9 3 sin n
sin( )
3

2 7

8. Evaluate  sin  cos d


5 2


2
1
 sin  cos2 n −1 d =  (m, n) → (1)
2 m −1
Solution: We have
0
2

Put 2m – 1 = 5 & 2n - 1 = 7/2 so that m = 3 , n = 9/4

Page | 168
Eq (1) becomes
 9
2 7 (3)( )
1 9 1
0 sin  cos d = 2  (3, 4 ) = 2
5 2 4
9
(3 + )
4

 ( m)  ( n )
Since  (m, n) =
 ( m + n)

9 9
(3)( ) ( )
1 4 = 4
=  (3) = 2!= 2
2 21 21
( ) ( )
4 4

 9
2 7 ( )
64
 sin  cos
5 2
d = 4 =
17 13 9 9 1989
0 . . ( )
4 4 4 4

3
−4 x2
9. Evaluate dx
0

3
−4 x2
Solution: Given dx
0

 3 −4 x = e −4 x
2 2
log 3

 
 x = e log x
 3 −4 x2
dx =  e − 4 x 2 log 3
dx
0 0

dy
Put 2 x log 3 = y  dx =
2 log 3

  
dy 1
3 dx =  e e
−4 x2 − y2 − y2
= dy
0 0 2 log 3 2 log 3 0


1   
3
−4 x2
dx = = =
0 2 log 3 2 4 log 3 16 log 3

1
10. When n is a positive integer , Prove that 2 n (n + ) = 1.3.5......( 2n − 1) 
2

Proof: We know that (n + 1) = n(n) → (1)

1 1 1 1
(n + ) = (n − + 1) = (n − )(n − )
2 2 2 2
Page | 169
1 3 1 3 3
= (n − )(n − + 1) = (n − )(n − )(n − ), by(1)
2 2 2 2 2

1 3 5 5
= (n − )(n − )(n − )(n − )
2 2 2 2

2n − 1 2n − 3 2n − 5 2n − 5
= . . ( )
2 2 2 2

2n − 1 2n − 3 2n − 5 3 1 1
= . . .......... . . ( ) (by repeated use of (1) n times)
2 2 2 2 2 2

(2n − 1)(2n − 3)(2n − 5)......... 3.1


= 
2n

1
Hence 2 n (n + ) = 1.3.5......( 2n − 1) 
2

1
11. Show that 2 2 n −1 (n)(n + ) = (2n) 
2

1
Proof : Since we have  (m, ) = 2 2 m−1  (m, m)
2

Writing the above result in terms of Gamma function we have


1 1
 ( m) ( ) ( )
2 = 2 2 m −1 (m)(m) or 2 = 2 2 m −1
 ( m)
1  ( m + m) 1  ( 2 m)
( m + ) ( m + )
2 2

  ( m) 1 
= 2 2 m −1 or(m)(m + ) = 2 m −1 (2m)
1  ( 2m) 2 2
(m + )
2

1
Hence 2 2 n −1 (n)(n + ) = (2n) 
2


x 8 (1 − x 6 )
12. Prove that 0 (1 + x) 24 dx = 0 using  -  functions.

Proof:
 
x 8 (1 − x 6 ) x 8 − x 14
0 (1 + x) 24 dx = 0 (1 + x) 24 dx
 
x8 x14
= dx −  dx
0 (1 + x ) 24
0 (1 + x ) 24

Page | 170
  
x 9−1 x15−1 x m −1
= 9 +15
dx −  15+ 9
dx Since  (m, n) =  dx
0
(1 + x ) 0
(1 + x ) 0 (1 + x )
m+ n

=  (9,15) −  (15,9)
=  (9,15) −  (9,15)
=0

(−1) n n!
1

 x (log x) dx =
m n
13. Prove that Where n is a positive integer and m > -1.
0 (m + 1) n +1

Proof: Let  x m (log x) n dx


0

−t
Put log x = −t  x = e  dx = −e −t dt

Also x = 0, t =  & x = 1, t = 0

1 0

 x (log x) dx =  (e ) (−t ) (−e dt)


m n −t m n −t

0 

 
= (−1) n  e −( m+1)t t n dt = (−1) n  e −( m+1)t t ( n+1)−1 dt
0 0

(n + 1)
= (−1) n 
(m + 1) n +1  ( n)
  e − kx x n −1 dx = , n  0, k  0
(−1) n!
n
kn
= 0

(m + 1) n +1

Page | 171

You might also like